Sie sind auf Seite 1von 190

V.T.U.

Web-Based Education

Engineering Mathematics – I (MAT-11)

By Dr. K.S. Chandrashekhar


Professor & Head,
Dept. of Mathematics, N.I.E.,
Mysore-8

Lesson 1 Differential Equations


Session – 1
Introduction
Many problems in all branches of science and engineering when analysed for putting in a
mathematical form assumes the form of a differential equation.
An engineer or an applied mathematician will be mostly interested in obtaining a solution
for the associated equation without bothering much on the rigorous aspects. Accordingly
the study of differential equations at various levels is focused on the methods of solving
the equations.
Preliminaries
Ordinary Differential Equation (O.D.E)
If y = f (x) is an unknown function, an equation which involves atleast one derivative of
y, w.r.t. x is called an ordinary differential equation which in future will be simply
referred to as Differential Equation (D.E).
The order of D.E is the order of the highest derivative present in the equation and the
degree of the D.E. is the degree of the highest order derivative after clearing the
fractional powers.
Finding y as a function of x explicitly [y = f (x)] or a relationship in x and y satisfying the
D.E. [f (x, y)= c] constitutes the solution of the D.E.
Observe the following equations along with their order and degree.

dy
1 = 2x
dx [order = 1, degree = 1]

F
Gdy I
J
2
Fdy I
+ 3GJ+ 2 = 0
2
Hdx K Hdx K [order = 1, degree = 2]

1
General solution and particular solution
A solution of a D.E. is a relation between the dependent and independent variables
satisfying the given equation identically.
The general solution will involve arbitrary constants equal to the order of the D.E.
If the arbitrary constants present in the solution are evaluated by using a set of given
conditions then the solution so obtained is called a particular solution. In many physical
problems these conditions can be formulated from the problem itself.
Note : Basic integration and integration methods are essential prerequisites for this
chapter.
Solution of differential equations of first order and first degree
Recollecting the definition of the order and the degree of a D.E., a first order and first
degree equation will be the form
dy
dx
bg bg bg
= f x, y or M x, y dx + N x, y dy = 0

We discuss mainly classified four types of differential equations of first order and first
degree. They as are as follows :
• Variables separable equations
• Homogenous equations
• Exact equations
• Linear equations

Variables separable Equations


If the given D.E. can be put in the form such that the coefficient of dx is a function of the
variable x only and the coefficient of dy is a function of y only then the given equation is
said to be in the separable form.
The modified form of such an equation will be,
P (x) dx + Q (y) dy = 0
Integrating we have
zP bg Qb
x dx + z yg
dy = c
This is the general solution of the equation.

2
Example –1
Solve :
b g
e x y − 1 dx + 2  e x + 4 dy= 0

>> e b
y − 1g x
dx + 2  e x
+ 4 dy= 0
Dividing throughout by (y-1) (ex + 4) we get,
ex dy
dx + 2 =0  Variables are separated  
x
e +4  y − 1

⇒ zx
ex
e +4
dx + 2
dy
y −1
=c z
i.e. log  e x + 4 + 2 log  y − 1 = c
or log  e x + 4 + log  y − 1 2 = c

i.e. log  e x + 4  y − 1 2
bg
= log k say

or  e x + 4  y − 1 2
= k is the required solution

Example 2

Solve 
dy
dx
= x e y − x given that y 0 = 0
2
bg
dy dy
= x e y − x or = x ey  e− x
2 2
>>
dx dx
dy
i.e. y = x e − x dx by separating the variables.*
2

e
z z
⇒ e − y dy − x e − x dx = c
2

z
i.e. − e − y dy − x e − x dx = c
2

Put − x 2 = t ∴− 2 x dx = dt or − x dx = dt  2
Hence we have, − e − y + e t dt  2 = c z
et
i e − e− y + =c
2
e−x
2

or − e − y = c is the general solution.


2
Now we consider y (0) = 0 That is y = 0 when x = 0,

* For animation

3
The general solution becomes
1 1
− 1 = c or c = −
2 2
Now the general solution becomes
e− x
2
1
− e− y = −
2 2

e j
or e − x + 1 = 2e − y is the particular solution.
2

Example –3
dy
Solve : x y = 1 + x + y + xy
dx
dy
>> x y = 1 + x + y + xy
dx
dy
i.e., xy =  1 + x  + y 1 + x 
dx
dy
i.e., xy =  1 + x   1 + y
dx
ydy 1 + x
or = dx by separating the variables.*
1+ y x

⇒ z y
1+ y
dy − z 1+ x
x
dx = c

or z  1 + y − 1
1+ y
z 1
z
dy − dx − 1 dx = c
x

i.e., z1 dy − z 1
1+ y
dy − log x − x = c

i.e., y − log  1 + y − log x − x = c


b g
or y − x − log x 1 + y = c is the required solution.

Example – 4 :
dy dy
Solve : y − x = y2 +
dx dx
>> Rearranging the given equation we have,

4
dy
 y − y2  =  x + 1
dx
dx dy
or = by separating the variables.*
x + 1 y − y2

⇒ z dx
= z dy
x + 1 y 1 − y
=c

b gz
i.e. log x + 1 −
dy
y 1 − y
=c     1

We have to employ the method of partial fractions for the second term of the above.
1 A B
Let + +
y(1 − y ) y 1 − y
b g
⇒ 1= A 1 − y + B y
Put y = 0  1 = A
Put y =1  1 = B

∴ zb g z z
dy
y 1− y
=
1
y
dy +
1
1− y
dy

or − z = − z dy + z
dy 1 −1
yb
1 − yg y
dy
1− y

i e − zyb1dy− yg= −log y + log b1 − yg= log F


1 − yI
G
Hy JK
Using this result in (1) we get,

b g F
log x + 1 + log G1 − yI
Hy JK= c
L( x + 1) (1 − y) O
or log M
N y P = log k  say
Q
∴  x + 1) (1 − y )= ky, is the required solution.

Example – 5 :
dy
Solve : tan y = cos( x + y ) + cos ( x − y )
dx
>> The given equation on expanding terms in the R. H.S. becomes
dy
tan y =  cos x cos y − sin x sin y +  cos x cos y + sin x sin y
dx
dy
i e tan y = 2 cos x cos y
dx

5
tan y
or dy= 2 cos x dx by separating the variables.*
cos y
z z
⇒ tan y. sec y dy − 2 cos x dx = c
∴ sec y − 2 sin x = c is the required solution.

Session – 2
Equations reducible to the variables separable form
Some differential equations can be reduced to the variables separable from by taking a
suitable substitution. We present a few examples.

Example – 6
Solve : cos (x + y + 1) dx – dy = 0

dy
>> We have = cos ( x + y +1)
dx … (1)
dt dy dt dy
Put t = x + y +1 ∴ =1+ or −1=
dx dx dx dx
dt dt
Now (1) becomes − 1 = cos t or = 1 + cos t
dx dx
i.e.
dt
1 + cos t
= dx ⇒
dt
z
1 + cos t
− dx =c z
i.e. z2 cosdt t  2
2
− x = c or z12 sec  t  2 dt − x = c
2

i.e. tan (t / 2) − x = c
F
Gx + y + 1I
i.e. tan
H 2 JK− x = c is the required solution.
Example – 7
dy
Solve : = x tan  y − x  + 1
dx
dy
>> = x tan ( y − x ) + 1 .... (1)
dx
dt dy dt dy
Put t = y − x ∴ = − 1 or + 1=
dx dx dx dx

6
dt dt
Now (1) becomes + 1 = x tan t + 1 or = x tan t.
dx dx
dt
Hence = x dx by separating the variables.*
tan t
z z
∴ cot t dt − x dx =c
x2
i.e. log (sin t ) − =c
2
x2
Thus log sin ( y − x ) − is the required solution.
2

Example – 8
Solve :  x + 2 y  dx − dy =  dx + dy
>> Rearranging the terms in the given equation we have,
 x + 2 y − 1 dx =  x + 2 y + 1 dy
dy  x + 2 y − 1
or =     1
dx  x + 2 y + 1
Put t = x + 2 y
dt dy F
G IJ
1 dt dy

dx
=1 + 2
dx H K
or
2 dx
−1 =
dx
1 Fdt I t − 1
The d.e becomes G − 1J =
2H dx K t + 1
dt  t − 1
∴ =2 +1
dx  t + 1
dt 3t − 1
i.e., =
dx t + 1

or
t +1
3t − 1
dt =dx ⇒ z t +1
3t − 1
z
dt − dx = c

i. e., z t +1
3t − 1
dt − x = c
… (2)
Let t + 1 = l(3t −1) + m
or t + 1 = ( 3l ) t + ( −l+m)
⇒ 3 l =1 and − l+ m = 1
Hence l = 1  3 and − 1  3 + m = 1 or m = 4  3
∴t + 1 = 1  3 .  3t − 1 + 4  3
Thus (2) can be written as

7
i.e. z1  3  33t t−−11 + 4  3 dt − x = c
i.e. z 1 dt + z
1 4 dt
− x=c
3 3 3t − 1
t 4
i.e. + log (3t − 1) − x = c
3 9
x + 2y 4
i.e. + log (3 x + 6 y − 1) − x = c
3 9
2 4
i.e.  y − x  + log  3x + 6 y − 1 = c is the required solution.
3 9
Example – 9 :
dy  x + y + 1
Solve : =
dx 2 x + 2 y + 3
dy  x + y + 1
>> We have =
dx 2( x + y + 3
dt dy dt dy
Put t = x + y ∴ =1 + or −1 =
dx dx dx dx
The given equation becomes,
dt t +1 dt t +1
−1= or =1 +
dx 2t + 3 dx 2t + 3
dt 2t + 3 + t + 1 dt 3t + 4
i.e. = or =
dx 2t + 3 dx 2t + 3
2t + 3
i.e., dt = dx
3t + 4

⇒ z32tt ++ 43 dt − zdx = c
Let 2t + 3 = l (3t + 4) + m
or 2t + 3 = (3l) t + (4l + m)
⇒ 3 l = 2 and 4 l+m= 3
∴ l= 2  3 and 8 / 3 + m= 3 or m = 1  3
Thus 2 t + 3 = 2/3 . (3t+ 4) + 1/3

Hence (1) becomes z2 / 3.(33tt ++4)4 + 1 / 3 dt − zdx = c


i.e.,
2
3
z
1 dt +
1
zdt
3 3t + 4
− x=c

8
2t 1
i.e., + log  3t + 4 − x = c, where t = x + y
3 9
2  x + y 1
∴ + log  3 x + 3 y + 4 − x = c
3 9
1 1
i.e., (2 y − x ) + log (3x + 3 y + 4) = c is the required solution.
3 9

Example – 10
dy
Solve : x 3 + x 2 y + sec 2  xy = 0
dx
The given d.e. can be written as,
F
G I
Hdx + yJK+ sec  xy = 0
dy
x2 x 2
      1

dt dy
Put t = xy ∴ =x +y
dx dx
dt
Now (1) becomes, x 2 + sec 2 t = 0
dx
dt dt dx
i.e., x 2 = − sec 2 t or 2
=− 2
dx sec t x

z
∴ cos2 t dt + zdxx = c
2

i.e. z1+ cos


2
2t 1
dt − = c
x
1 F sin 2t I 1
i.e., G J− = c, where t = xy.
2 H 2 Kx
t+

1 1
i.e., 2xy + sin (2 xy ) − = c, is the required solution.
4 x

9
EXERCISES :
Solve the following differential equations

1  xy 2 + x  dx +  x 2 y + y dy = 0
2  x 2 − yx 2  dy +  y 2 + x 2 y 2  dx = 0
dy F
G dyIJ
3 y − x
dx H
= a y2 +
dx K
4  x + y  dx − dy = dx + dy
dy
5 x 4 + x 3 y+ cosec ( xy) = 0
dx

ANSWERS :
1  1 + x  2  1 + y 2 = c
2 log y + 1  y + 1  x − x = c
3  1 − ay  a + x  = cy
4 y − x + log  x + y = c
5 cos ( xy) + 1 / 2 x 2 = c

Session –3

A function u = f (x, y) is said to be homogeneous function of degree n if


b g b g
u = x n g y / x or u = y n g x / y
A D.E. of the form M (x, y) dx+ N (x, y) dy = 0 is said to be a homogeneous differential
equation if both M (x, y) and N (x, y) are homogeneous functions of the same degree.

Solution of a homogeneous equation


bg
dy f x, y
We prefer to have the differential equation in the form =
bg
dx g x, y
after recognizing that the D.E. is a homogeneous one.
We take the substitution y = v x so that,
dy dv
= v.1 + x  by product rule.
dx dx
With these the given d.e. can be solved by separating the variables.

10
Example –1

Solve : x 2 y dx −  x 3 + y 3  dy = 0
>> (Observe that the coefficient of dx and dy are homogeneous functions of degree 3)
The given equation can be written as,
dy x2 y
= 3      1
dx x + y 3
dy dv
Put y = v x ∴ =v+ x
dx dx
dv x 2 . vx
Now (1) becomes, v + x =
dx x 3 + v 3 x 3
dv x3 v dv v
i. e., v + x = 3 or x = −v
dx x  1 + v 3  dx 1 + v 3
dv v − v − v 4 dv − v 4
i. e., x = or x =
dx 1 + v3 dx 1 + v 3
1 + v3 − dx
∴ 4
dv =  by separating the variables.*
v x
(* for animation)

⇒ zv1 dv +z1v dv + zdxx = c


4

v −3
i.e., + log v + log x = c
−3
1 y
i.e., − 3 + log (vx ) = c, where v =
3v x
3
x
Thus − 3 + log y = c, is the required solution.
3y

Example –2

Solve : x dy − y dx = x 2 + y 2 dx
L
M O
P
N
>> We have x dy = y + x 2 + y 2 dx
Q
dy y + x 2 + y 2
or =      1
dx x

11
dy dv
Put y = vx ∴ = v+ x
dx dx
dv vx + x 2 + v 2 x 2
Now (1) becomes v + x =
dx x
F
H
dv x v + 1 + v
2 IK dv
i.e., v + x = or x = 1 + v2
dx x dx
dv dx
∴ = by separating the variables. *
1 + v2 x

⇒ z 1dv+ v − zdxx = c
2

i. e., sin h −1 v − log x = c, where v = y / x


Thus sin h −1  y / x  − log x = c, is the required solution.

Example –3
dy y 2
Solve : x + =y
dx x
dy y2
>> We have x = y−
dx x
2
dy xy − y
or =      1
dx x2
dy dv
Put y = v x ∴ = v + x
dx dx
dv x .v x − v 2 x 2
Now (1) becomes v + x =
dx x2
dv x 2  v − v 2  dv
i.e., v + x = 2
or x = −v 2
dx x dx
dv dx
∴ 2
=− by separating the variables.*
v x

z z
dv
Hence 2 +
v
dx
x
1
= c i.e., − + log x = c, where v =
v
y
x
x
Thus − + log x = c, is the required solution.
y

12
Example – 4
Solve : x tan ( y / x ) − y sec 2  y / x  dx + x sec 2  y / x  dy = 0
>> The given equation can be written as,
dy y sec 2  y / x  − x tan ( y / x )
=       1
dx x sec 2  y / x 
y dy dv
Put = v or v = v x ∴ = v + x
x dx dx
2
dv v x sec v − x tan v
Now (1) becomes, v + x =
dx x sec 2 v
dv x  v sec 2 v − tan v
i.e., v + x =
dx x sec 2 v
dv v sec 2 v − tan v
i.e., x = −v
dx sec 2 v
dv v sec 2 v − tan v − v sec 2 v
i.e., x =
dx sec 2 v
dv − tan v sec 2 v dx
i.e., x = 2
or dv = −
dx sec v tan v x

Hence z
sec 2 v
tan v
dv+ z
dx
x
=c

i.e., log (tan v ) + log x = c


or log (tan v. x ) = c = log k (say),
⇒ x tan v = k where v = y / x
Thus x tan ( y / x ) = k, is the required solution.

Example – 5
dy
Solve : x = y log y − log x + 1
dx
>> The given equation can be written in the form
dy y
= log ( y / x ) + 1      1
dx x

13
dy dv
Put y = v x ∴ = v+ x
dx dx
dv
Now (1) becomes v + x = v  log v + 1
dx
dv dv dx
i.e., x = v log v or =
dx v log v x

Hence z1/ v
log v
dv −
dx
x
=c z
i.e., log  log v − log x = c = log k  say
i.e., log  log v = log k + log x
i.e., log  log v = log  kx 
⇒ log v = kx where v = y / x
Thus l g ( y / x ) = k x, is the required solution.

Example – 6

e j F
G xI
Solve : 1 + e x/y dx + e x/y 1 −
H y JKdy = 0
>> (As we observe terms with x / y , we need to express the
equation relating to dx / dy and the terms are homogeneous functions of degree 0.

e j F
Gx I
We have 1 + e x/y dx = e x/y
Hy − 1JKdy
Fx I
e G− 1J
Hy K
x/y
dx
or =         1
dy e1 + e j x /y

dx dv
Put x / y = v or x = v y ∴ = v+y
dy dy
dv e v  v − 1
Now (1) becomes, v + y =
dy  1 + e v 
dv e v  v − 1 dv e v v − e v − v − e v v
i.e., y = − v i.e., y =
dy  1 + e v  dy  1 + ev 
dv  e v + v (1 + e v  dv dy
i.e., y =− or =−
dy  1 + ev  v
e +v y

z(1e+ e+ )vdv + zdyy = c


v
Hence v

14
i.e., log (e v + v ) + log y = c
or log  e v + v y = log k (say)

⇒  e v + v y = k, where v = x / y.
Thus y e x/y + x = k, is the required solution.

Session – 4

Equations reducible to the homogeneous form


Consider the differential equation in the form :
 ax + by + c dx ±  a' x + b' y + c'  dy = 0
We first express the equation in respect of dy / dx and the procedure is narrated by taking
dy a x + b y + c a b
= where ≠      1
dx a' x +b' y + c' a' b'
This condition implies that there are no common factors for the x and y terms in the
numerator as well as in the denominator.
Put x = X + h, y = Y + k where h and k are constants to be chosen appropriately later.
dy dy dY dX
Now =
dx dY dX dx
dY dy dY
= 1 .1 Hence =
dX dx dX
As a consequence of these (1) becomes
dY b gb g
a X +h + b Y +k +c
=
b g b g
dX a' X + h + b' Y + k + c'
dY baX + bY gb + ah + bk + cg
dX b a' X + b' Y gb
+ a' h + b' k + c' g
i.e., =      2

Now, let us choose h and k such that :


ah + bk + c = 0 and a' h + b' k + c' = 0
Solving these equations we get the value for h and k.
Thus (2) now assumes the form
dY aX + bY
=       3
dX a' X + b' Y
It is evident that (3) is a homogeneous equation in the variables X and Y. This equation
can be solved by putting Y = VX as discussed already. Finally we substitute for X and
Y where X = x −h, Y = y −k.

15
Example –1
Solve : ( x − 4 y − 9) dx +  4 x + y − 2 dy = 0
>> We have  4 x + y − 2 dy = −  x − 4 y − 9 dx
dy − x + 4 y + 9
∴ =
dx 4 x + y − 2
Put x = X + h and y = Y + k where h and k are constants to be choosen suitably later.
dy dy dY dX
Now =
dx dY dX dx
dY dy dY
= 1  1 Hence =
dX dx dX
Thus (1) becomes
dY − X + h + 4 Y + k  + 9
=
dX 4  X + h +  Y + k  − 2
dY  − X + 4Y  +  − h + 4 k + 9
i.e., =         2
dX  4 X + Y  +  4 h + k − 2
Let us choose h and k such that
− h + 4 k + 9 = 0 and 4 h + k − 2 = 0
Solving these equations we get, h = 1 and k = −2
dY − X + 4Y
Thus (2) becomes =        3
dX 4 X + Y
dY dV
Put Y = VX ∴ =V + X
dX dx
dV − X + 4VX
Now (3) becomes, V + X =
dX 4 X + VX
dV X  −1 + 4V  dV −1 + 4V
i. e., V + X = or X = −V
dX X 4 + V  dX 4+V
dV −1 + 4V − 4V − V 2 dV − 1 + V 2 
i. e., X = i.e., X =
dX 4+V dX  4 +V
 4 + V  dV dX
∴ 2
=− by separating the variables.*
 1+ V  X

16
⇒4 z1 +dVV + z1V+dVV + zdXX = c
2 2

1
i.e., 4 tan −1 V + log (1 + V 2  + log X = c
2
−1
i.e., 8 tan V + log (1 + V 2  + 2 log X = 2c
i.e., 8 tan −1 V + log (1 + V 2  X 2 = 2c, where V = Y / X

∴ 8 tan −1  Y / X  + log ( X 2 + Y 2  = 2c = k  say


But X = x − h = x − 1 and Y = y − k = y + 2
F
Gy + 2I
8 tan −1
Hx − 1JK+ log  x − 1
2 2
+  y + 2 =k

This is the required solution.

Example – 2
dy x + 2 y − 3
Solve : =
dx 2 x + y − 3
dy x + 2 y − 3
>> We have =       1
dx 2 x + y − 3
Put x = X + h and y = Y + k, where h and k are constants to be chosen suitably later.
dy dy dY dX
∴ =
dx dY dX dx
dY dy dY
= 1  1 Hence =
dX dx dX
dY  X + h + 2 Y + k  − 3
Thus (1) becomes, =
dX 2  X + h +  Y + k  − 3
dY  X + 2Y  +  h + 2 k − 3
i. e., =
dX  2 X + Y  +  2h + k − 3 ……(2)
Let us choose h and k such that,
h+ 2 k − 3 = 0 and 2 h + k − 3 = 0
Solving these equations we get h = 1 and k = 1
dY X + 2Y
Thus (2) becomes =       3
dX 2 X + Y

17
dY dV
Put Y = VX ∴ =V+X
dX dX
dV X + 2VX
Now (3) becomes V + X =
dX 2 X + VX
dV X  1 + 2V 
i.e., V + X =
dX X 2 + V 
dV 1 + 2V dV 1 − V 2
or X = − V or X =
dX 2 + V dX 2 + V
2+V dX
∴ 2
dv = by separating the variables.*
1− V X

⇒2 z dV
1− V 2 z z
+
V dV
1− V 2

dX
X
=c

1 F
G 1+ VIJ 1
i. e., 2
2
log
H K
1− V
− log (1 − V 2  − log X = c
2
F1 + V I
i. e., 2 log G J − log (1 − V  − 2 log X = 2c
H1− VK
2

F1 + V I 2
i.e., log G J − log (1 − V  X = 2c
H1− VK
2 2

or log M
L (1 + V  2 O
P
= log k  say
N(1 − V 2
(1 − V  2
X Q 2

(1 + V  (X +Y) Y
⇒ = k or = k since V =
3 2
(1 − V  X X −Y
3
b g X
ut X = x − h = x − 1 and Y = y − k = y − 1
3
Thus  x + y − 2 = k  x − y is the required solution.

Exercises :
Solve the following equations:
1  x 3 + y 3  dx = xy  x + y dy
2  x + y log x − y log y dx − x  log x − log y dy = 0
3 Sin 2 ( x / y) y dx − x dy = y dy
4  y − x − 4 dx =  y + x − 2 dy
5  3 x − y − 1 dy + 2 x − y dx = 0

18
Answers :
1 y / x + log y − x  = c
2 log x +  y / x  log ( y / x ) − 1 = c
x 1
3. − sin (2 x  y) − log y = c
2y 4
4 log ( x + 1)2 +  y − 3 2
+ 2 tan −1  y − 3   x + 1 = c

5  x + y − 1 4 = c  4 x − 2 y − 1
Session - 5

We say that M (x, y) dx+N (x, y) dy = 0 is an exact differential equation if there exists a
function f (x, y) such that
df = M (x, y) dx+N (x, y) dy.
The necessary and the sufficient condition for the D.E. M (x, y) dx + N (x, y) dy = 0 to be
an exact equation is
M N
=
y x
Further the solution of the exact equation is given by
zMdx + zNbg
y dy = c
where, in the first term we integrate M (x, y) w.r.t x keeping y fixed and N (y) indicate
the terms in N not containing x.

Example –1
b g
Solve : [ y 1 + 1  x + cos y dx + x + log x − x sin y dy = 0
>> Let M = y b
1 + 1  xg
+ cos y and N = x + log x − x sin y
M 1 N 1
∴ = 1 + − sin y and = 1 + − sin y
y x x x
M N
Since =  the given equation is exact.
y x
z z bg
The solution is M dx+ N y dy = c

L
i.e., z
MyG
F1 + 1 IJ + cos yO dx + z
NH x K P Q 0 dy= c

Thus y bx + log x g + x cos y = c, is the required solution.

19
Example –2
dy y cos x + sin y+ y
Solve : + =0
dx sin x + x cos y + x
>> The given equation is put in the form,
by cos x +sin y+ ygdx + bsin x + x cos y + xgdy = 0
Let M = y cos x + sin y+ y and N = sin x + x cos y + x
M N
∴ = cos x + cos y+ 1 and = cos x + cos y+ 1
y x
M N
Since =  the given equation is exact.
y x
z bg
z
The solution is M dx + N y dy = c
i. e., zby cos x +sin y+ ygdx = z0 dy= c
Thus y sin x + x sin y + xy = c, is the required solution.

Example – 3

e 2

j e 2

j
Solve : y 2 e xy + 4 x 3 dx + 2 xye xy − 3 y 2 dy = 0
2 2
>> Let M = y 2 e xy + 4 x 3 and N = 2 x y e xy − 3 y 2
M 2 2 N 2 2
∴ = y 2  e xy  2 xy +e xy  2 y , = 2 x y e xy  y 2 + 2 ye xy
y x
M 2 2 N 2 2
i.e., = 2 xy 3 e xy + 2 ye xy and = 2 xy 3 e xy + 2 ye xy
y x
M N
Since =  the given equation is exact.
y x
z z
The solution is M dx + N  y dy= c

i.e., zey e + 4 x jdx + z−3y dy = c


2 xy 2 3 2

2
e xy
i.e., y  2 + x 4 − y 3 = c
2
y
2
Thus e xy + x 4 − y 3 = c, is the required solution.

Equations reducible to the exact form


Sometimes the given differential equation which is not an exact equation can be
transformed into an exact equation by multiplying with some function (factor) known as
the integrating factor (I.F.)

20
The procedure to find such a factor is as follows.
Suppose that, for the equation M dx + N dy = 0
M N
≠ , then we take their difference.
y x
1 F
GM NI
J = f bg
1 FM
G NI
J = gbyg
If
N Hy x K
− x or
M Hy

xK

then e z bg or e zbg
f x dx − g y dy
is an integrating factor.

Example – 4
b g
Solve : (4 xy + 3 y 2 − x ) dx + x x + 2 y dy = 0

Let M = 4 xy + 3 y 2 − x and N = x  x + 2 y = x 2 + 2 xy
M N
= 4 x + 6 y and = 2 x + 2 y. (The equation is not exact.)
y x
M N
Consider − = 2 x + 4 y = 2 x + 2 y     close to N.
y x
F
G N I 2 x + 2 y
bg
Hy x JK= x x + 2 y
1 M 2
Now − = = f x
N x
Hence e z bg is an integrating factor.
f x dx

z dx 2
e z bg = e x = e z2 log x = e zlog  x 
2
f x dx
= x2
Multiplying the given equation by x 2 we now have,
M = 4 x 3 y + 3 x 2 y 2 − x 3 and N = x 4 + 2 x 3 y
z
Solution is given by M dx + N y dy = c z
Thus z(4 x y + 3x y
3 2 2
− x ) dx + z
3
0 dy = c

4 x4 3 2
i.e., x y + x y − = c, is the required solution.
4

21
Example – 5
b g b g
Solve : y 2 x − y + 1 dx + x 3 x − 4 y + 3 dy = 0
>> Let M = y b
2 x − y + 1g
and N = xb
3 x − 4 y + 3g
i.e., M = 2 xy − y 2 + y and N = 3 x 2 − 4 xy + 3 x
M N
= 2 x − 2 y + 1 = 6x − 4y + 3
y x
M
y

N
x
b g
= −4 x + 2 y − 2 = −2 2 x − y + 1     near to M.

F N I −2 b2 x − y + 1g 2
G J = − = gbg
1 M
Now,
M Hy − x K yb
=
2 x − y + 1g y
y

z
− g y  dy
z2y dy y2 
Hence I. F = e e = e 2 log y = e log  = y2
Multiplying the given equation with y 2 we now have,
M = 2 x y 3 − y 4 + y 3 and N = 3 x 2 y 2 − 4 xy 3 + 3 xy 2
z
The solution is M dx + N y dy = c z bg
i.e., ze2 x y 3
− y4 + y jdx + z
3
0 dy = c

Thus x 2 y 3 − xy 4 + xy 3 = c, is the required solution.

EXERCISES:
Solve the following differential equations
1. cos x (ey +1)dx + sin x eydy = 0
2. [4x3 y2+ y cos (xy)] dx+[2x4y+x cos (xy)] dy =0
3  xy 2 + x − 2 y + 3 dx + x 2 y dy = 2 x + y dy ; y 1 = 1
b g
4 y x + y + 1 dx + x x + 3 y + 2 dy = 0

5 2 3 x 2 + 2 y 3 + 6 y dx + 3 x + xy 2  dy = 0 given that y (1) = 2

ANSWERS:

22
1 sin x e y + 1 = c
bg
2 x 4 y 2 + sin xy = c
x 2 y2 x 2
3 + − 2 xy + 3 x − y 2 = 1
2 2
2 2
x y
4 + xy 3 + xy 2 = c
2
5 x 6 + x 4 y 3 + 3 x 4 y = 15

Session – 6
Linear Equations
A differential equation of the form
dy
+ Py = Q
dx …(1)
where P and Q are functions of x only is called a linear equation in ‘y’.
z
y e zP dx = Q e zP dx dx + c
is the solution of the linear equation (1).
An equation of the form :
dx
+ Px = Q ...  2
dy
where P and Q are functions of y is called a linear equation in x.
The solution can simply be written by interchanging the role of x and y.
z
i.e.. x e zP dy = Q e zP dy dy + c
is the solution for the linear equation (2)
Working procedure for problems
• The given equation must be first put in the form conformal to the standard
form of the linear equation in x or y.
• The expression for P and Q is to be written by simple comparison.

• We equip with the I. F. e zP dx or e zP dy 

• We assume the associated solution and we only need to tackle the R.H.S. part
of the solution to finally arrive at the required solution.

23
Example – 1
dy
Solve : + y cot x = cos x
dx
dy
>> + y cot x = cos x is of the form
dx
dy
+ Py = Q, where P = cot x and Q = cos x
dx
∴ e zP dx = e zcot x dx = e zlog (sin x  = sin x

z
The solution is y e zP dx = Q e zP dx dx + c
z
i.e., y sin x = cos x. sin x dx + c

y sin x = z
sin 2 x
i.e., dx + c
2
− cos 2 x
Thus y sin x = + c , is the required solution.
4
Example – 2
Solve : 2 y' cos x + 4 y sin x = sin 2 x given that y b 3g= 0
>> Dividing the given equation by 2cos x, we have
dy
+  2 tan x  y = sin x sin 2 x = 2 cos x sin x
dx
dy
This is of the form + Py = Q, where P = 2 tan x and Q = sin x
dx
e zpdx = e z2 tan x dx = e z2 log (sec x ) = sec 2 x

z
The solution is y e zP dx = Q e zP dx dx + c

z
i.e., y sec 2 x = sin x. sec 2 x dx + c
i.e., y sec 2 x =ztan x sec x + c
Thus y sec 2 x = sec x dx + c, is the general solution. ..... (1)
Consider y b 3g= 0 That is y = 0 when x =  3
Hence (1) becomes 0 = 2 + c or c = −2
Thus y sec 2 x = sec x − 2, is the required particular solution.

24
Example – 3
Solve : (1 + y 2 )dx +  x − tan −1 y dy = 0
dx tan −1 y − x
>> We have =
dy 1 + y2
dx x tan −1 y
or + =
dy 1 + y 2 1 + y 2
dx
This is of the form + Px = Q
dy
1 tan −1 y zP dy = e tan −1y
Here P = and Q = ; e
1 + y2 1 + y2
The solution is given by x e zP dy = Q e zP dy dy + c z
−1
i.e., x e tan
−1
y
= ztan
1+ y
y
e 2
tan −1 y
dy + c

1
By putting tan −1 y = t, we obtain dy = dt
1 + y2
∴ x e tan
−1
y
z
= t e t dt + c
−1
i.e., x e tan y
= t e t − e t + c, by parts.
−1 −1
Thus x e tan y
= e tan y
 tan −1 y − 1 + c, is the required solution.

Equations reducible to linear form


dy
Form (i): f '  y + P f  y = Q, where P and Q are function of x.
dx
dy dt
We put f  y = t ∴ f '  y =
dx dx
dt
The given equation becomes + Pt = Q which is a linear equation in t.
dx
Similarly, f '  x 
dx
dy
bg
+ P f x = Q, where P and Q are function of y can be

reduced to the linear form by putting f  x  = t

25
dy
Form (ii) : + P y = Q y n  where P and Q are functions of x.
dx
This equation is called as Bernoulli' s equation y.
We first divide the equation throughout by y n to obtain
1 dy
+ P y1− n = Q .....  1
y n dx
dy dt 1 dy 1 dt
Put y1− n = t ∴  1 − n y − n = or n =
dx dx y dx  1 − n dx
1 dt
Hence (1) becomes, + P t=Q
 1 − n dx
dt
or +  1 − n P . t =  1 − n Q which is a linear equation in t.
dx
dx
Similarly + P x = Q x n , where P and Q are functions of y is called
dy
Bernoulli' s equation x. We first divide by x n and later put x 1− n = t
to obtain a linear equation in t.
Example - 4
dy
Solve : tan y + tan x = cos y cos2 x
dx
>> Dividing the given equation by cos y we have
dy
sec y tan y + sec y tan x = cos2 x       1
dx
dy dt
Now, put sec y = t ∴ sec y tan y =
dx dx
dt
Hence (1) becomes + t tan x = cos2 x
dx
dt
This equation is of the form + Pt = Q, where we have,
dx
P = tan x and Q = cos 2 x
∴ e zP dx = e ztan x dx = e zlog (sec x  = sec x
z
The solution is t e zP dx = Q e zP dx dx + c
z 2
i.e., sec y sec x = cos x . sec x dx + c
i.e., sec y . sec x = z
cos x dx + c
Thus sec y sec x = sin x + c, is the required solution.

26
Example – 5
dy y
Solve : + = y2 x
dx x
>> Dividing the given equation by y 2 we have,
1 dy 1
2
+ =x      1
y dx y x
1 −1 dy dt
Put =t ∴ 2 =
y y dx dx
− dt t
Hence (1) becomes + =x
dx x
dt t
or − = −x
dx x
dt
This equation is a linear equation of the + Pt = Q,
dx
−1
where P = and Q = − x
x
z1x dx
∴ e zP dx = e
− 1
= e − log x =
x
The solution is t e zP dx z
= Q e zP dx dx + c

i.e., t .
1
x
z1
= − x . dx + c
x
1
Thus = − x + c, is the required solution.
xy

Example – 6
dy
Solve : xy  1 + xy 2  =1
dx
dx
>> Consider = x y + x 2 y3
dy
dx
or − xy = x 2 y 3  Dividing by x 2 we get,
dy
1 dx 1
2
− y = y3  1
x dy x
1 −1 dx dt
Put =t ∴ 2 =
x x dy dy

27
dt dt
Hence (1) becomes − − t y = y 3 or + t y = − y3
dy dy
dt
This equation is of the form + P t = Q where,
dy
P = y and Q = − y 3 ∴ e zP dy = e zy dy = e y
2
 2

The solution is t e zP dy = Q e zP dy dy + c z
i. e., t e y
2
 2
z
= − y3 e
y2  2
dy + c
2
Put y  2 = u ∴ y dy = du
Also y 2  y dy = y 2 du or y 3 dy = 2u du
∴ t ey
2
 2
z
= −2 u e u du+c
y2  2
i. e., t e = −2 u e u − e u  + c , on integration by parts.
ey  2
2
y2 F I
G
H JK
2
Thus = 2 ey  2 1 − + c , is the required solution.
x 2

EXERCISES
Solve the following equations
dy
1 + y cot x = 4 x cosec x ; y  2 = 0
dx
dy
2 sin 2 x − 2 y = tan x
dx
b
3 x dy + y − x − xy tan x dx = 0 g
dy y
4 + = x 2 y6
dx x
5  y 4 − 2 xy dx + 3 x 2 dy = 0

Answers:
1 y sin x = 2 x 2 −  2
 2
2 y cot x = log tan x + c
3 x y cos x = cos x + x sin x + c
1 5
4 5 5 − 2 = c
x y 2x
5 x 2 = y 3  x + 2

28
Session – 7

Orthogonal Trajectories

Definition : If two family of curves are such that every member of one family intersects
every member of the other family at right angles then they are said to be orthogonal
trajectories of each other.

Working procedure for problems

Case – i : (Cartesian family)

* Given f (x, y, c) = 0, differentiate w.r.t x and eliminate c.


dy dx
* Replace by − and solve the equation.
dx dy
Case – ii : (Polar family)
* Given an equation in r and θ, we prefer to take logarithms first and then
differentiate w.r.t θ.
dr d
*  fter ensuring that the given parameter is elimiated we replace by − r 2
d dr
and solve the equation.

Example –1

Find the Orthogonal trajectories of the family of parabolas y2 = 4 a x.


y2
>> Consider = 4a
x
Now differentiating (1) w.r.t x we have,
dy
x .2y − y2  1
dx dy
2
= 0 or 2 x y − y2 = 0
x dx
dy
i.e., 2 x − y = 0 is the D. E. of the given family.
dx
dy dx F
G IJdx
Replacing
dx
by −
dy H K
we have, 2 x −
dy
− y=0

or 2 x dx + y dy = 0
z z
⇒ 2 x dx + y dy = c
y2
i.e., x 2 + = c or 2 x 2 + y 2 = 2c = k  say
2
Thus 2 x 2 + y 2 = k is the required O.T.

29
Example – 2

Find the orthogonal trajectories of the family of curves


x2 y2
+ = 1 where is the parameter.
a 2 b2 +
x2 y2
>> We have + =1         1
a 2 b2 +
Differentiating w. r. t x we have,
2 x 2 y y1 dy
2
+ 2 = 0 where y1 =
a b + dx
x −y y
i.e., 2 = 2 1      2
a b +
x2 − y2
Also from (1) 2 − 1 = 2
a b +
2 2 2
x −a −y
or 2
= 2       3
a b +
Now, dividing (2) by (3) we get,
x yy x y
2 2
= 21 or 2 2
= 1
x −a y x −a y
dy − dx
Now, let us replace y1 = by
dx dy
x 1 F
G IJ
dx
∴ 2
x −a 2
=
y

H K
dy
− x 2 − a 2 
or y dy = by separating the variables.
x

z z
⇒ y dy = − x dx + a 2 z dx
x
+c
y2 − x 2
i.e., = + a 2 log x + c
2 2
or x + y − 2 a 2 log x − b = 0 is the required orthogonal trajectories. Here b = 2c
2 2

Example-3

Show that the family of parabolas y2 = 4a (x+a) is a self orthogonal.


>> Consider y2 = 4a (x+a) ….(1)
Differentiating w.r.t x, we have
dy yy dy
2y = 4 a ∴ a = 1 where y1 =
dx 2 dx
Substituting this value of ‘a’ in (1) we have,

30
F
G yy I
y 2 = 2 yy1 x +
H 2 JKor y = 2 xy + yy
1 2
1 1

Thus we have, y = 2 xy1 + yy12      2


This is the D.E. of the given family.
Now replacing y1 by −1 / y1 (2) becomes
F−1I F−1I
y = 2 x G J + yG J or y =
2
−2 x y
Hy K Hy K
1 1 y1
+ 2
y1
i.e., yy12
+ 2 x y1 = y .......  3
(3) the D.E. of the orthogonal family which is same as (2) being the D.E. of the given
family.
Thus the family of parabolas y2 = 4a(x+a) is self orthogonal.

Example –4

Find the O.T. of the family r =a(1+sin θ)


>> We have r = a (1 + sin θ)
⇒ log r = log a + log (1 + sin 
Differentiating w.r.t θ we have,
1 dr cos
=0+
r d 1 + sin
dr d
Replacing by − r 2 we get,
d dr
1 F
G IJ d cos
r H K
−r 2
dr
=
1 + sin
d cos
i.e., − r =
dr 1 + sin
1 + sin − dr
or d = by separating the variables.
cos r
⇒ z dr
r
+ z 1 + sin
cos
d =c

z z
i.e., log r + sec d + tan d = c
i.e., log r + log (sec + tan ) + log (sec  = c
i.e., log r b
sec + tan g
sec = log b (say)
F 1 sin IJ 1 = b
⇒ rG +
Hcos cos Kcos
r 1 + sin  r 1 + sin 
i.e., 2
= b or =b
cos  1 − sin 2 
Thus r = b 1 − sin  is the required O. T.

31
Example – 5
Find the O.T. of the family rn cos nθ = an
>> We have rn cos n θ = an
⇒ n log r + log (cos n ) = n log a
Differentiating w.r.t θ we have,
n dr F
G
− n sin n IJ 1 dr
rd
+
H cos n K
= 0 i. e.
r d
= tan n

dr d
Replacing by − r 2 we have,
d dr
1 F
G IJ = tan n
d d
r H K
−r 2
dr
or − r
dr
= tan n

d dr
∴ = by separating the variables.
tan n −r
⇒ z z
dr
r
+ cot n d = c
1
i.e., log r + log (sin n ) = c
n
or n log r + log (sin n ) = nc
i.e., log(r n sin n ) = log b (say)
∴ r n sin n = b, is the required O.T.

EXERCISES
Find the orthogonal trajectories of the following family of curves
1 y = ax 2
2 x 2 + y 2 + 2 y + c = 0 being the parameter.
3. Show that the family of parabolas x 2 = 4 a  y + a is a self orthogonal.
4. r = a sec 2   2
5 r =a  1 + cos 

ANSWERS
1 x 2 + 2 y 2 = c
2. x 2 + y 2 + 2 kx + c = 0  k is arbitrary.
4. r = b cosec 2   2
5 r = b  1 − cos 

32
LESSON – 2 Part – C Integral Calculus

Session - 1

Introduction :
We are familiar with various methods of integration, definite integrals and the associated
application of finding the area under a curve.
In this chapter we first discuss reduction formulae and later discuss the method of tracing
cartesian and polar curves. By knowing the shape of a given curve we disucss application
of definite integrals such as area, length or perimeter, surface area of plane curves and
volume of solids.
In all these applications reduction formulae plays a vital role in the evaluation of definite
integrals.
Reduction Formulae
Reduction formulae is basically a recurrence relation which reduces integral of functions
of higher degree in the form of zf  x n
dx, zf  x m
g x  n
dx (where m and n are non negative
Reduction formulae is basically a recurrence relation which reduces integral of functions
of higher degree in the form of zf  x n
dx, zf  x m
g x  n
dx (where m and n are non negative
integers) to lower degree. The successive application of the recurrence relation finally
end up with a function of degree 0 or 1 so that we can easily complete the integration
process.
We discuss certain standard reduction formulae in the form of indefinite integrals and the
evaluation of these with standard limits of integration.

z zsin x dx dx where n is a positive integer


π /2
Reduction formula for sin n x dx and n

z n
Let In = sin x dx
=z
sin n −1
z
x . sin x dx = u v dx  say
We have the rule of integration by parts,
zu v dx = uzvdx − zzv dx.u' dx
∴ I = sin x − cos x  − z
n
n −1
 − cos x   n − 1 sin x  cos x dxn−2

= − sin x  cos x +  n − 1 z
n −1 n−2
sin x  cos x dx 2

= − sin x  cos x +  n − 1 z
n −1 n−2
sin x  1 − sin x ) dx 2

= −sin x  cos x +  n − 1 z
n −1
sin x dx −  n − 1 z
n−2
sin x dx n

33
i.e. In = − sin n −1 x cos x +  n − 1 In − 2 −  n − 1 In
i.e. In 1 +  n − 1 = − sin n −1 x . cos x +  n − 1 In − 2
−sin n −1 x . cos x n − 1
z
∴ In = sin n x dx =
n
+
n
In − 2       1

This is the required reduction formula.


Illustration
z
 i To find sin 4 x dx
>> Comparing with L.H.S. of (1), we need to take n = 4 and use the established result.

z
∴ I4 = sin 4 x dx =
−sin 3 x cos x 3
4
+ I2
4
We need to apply the result (1) again by taking n = 2

i.e., I4 =
−sin 3 x cos x 3 −sin x cos x 1
+ + I0
R
S U
V
4 4 2 2 T W
We cannot find I0 from (1). But basically we have
z z
I0 = sin 0 x dx = 1 dx = x

=z
3
−sin x cos x 3
4 3x
Thus I4 sin x dx = − sin x cos x + +c
4 8 8
Corollary :

zsin
2
n
To evaluate x dx
0

zsin
 2
n
>> Let In = x dx
0

Equation (1) must be established first.

∴ from (1) In
Lsin
= −M
n −1
x . cos x O
P +
 2
n −1
I
N n Q n
0
n−2

But cos (π / 2) = 0 = sin 0


n −1
Thus In = In − 2 .     2
n
We use this recurrence relation to find In − 2 by simply replacing n by (n − 2).

34
n−3
i.e., In − 2 = In − 4
n−2
n −1 n − 3
Hence In =  In − 4
n n−2
n−5
Similarly from (2) In − 4 = In −6
n−4
n −1 n − 3 n − 5
Hence In =   In −6  again by back substitution.
n n−2 n−4
Continuing like this the reduction process will end up as below.
n −1 n − 3 n − 5 2
In =      I1 if n is odd
n n−2 n−4 3
n −1 n − 3 n − 5 1
=      I0 if n is even
n n−2 n−4 2

zsin x dx = − cos x = − 0 − 1 = 1
 2
 2
But I1 = 0
0

= zsin x dx = zdx = x
 2  2
0  2
and I0 = 0
0 20

Thus we have,
R
|
n −1 n − 3
 
n−5 2
 if n is odd
zsin x dx = S
 2
n n n − 2 n−4 3
0 |Tn n− 1  nn −− 23  n−5 1
    if n is even
n−4 2 2
Note :
z
The reduction formula for cos n x dx can be established in a similar way.
The result is as follows.
n −1
zcos x dx = cos xn . sin x + n n− 1 I
n
n−2

zsin x dx = zcos x dx by the property zf  x dx =zf  a − x dx


 2  2 a a
n n

0 0 0 0

35
Session – 2
 
Reduction formula for tan n x dx and z ztan
0
n
x dx, where n is a positive integer.

z
Let In = tan n x dx
=ztan n−2
z
x . tan 2 x dx = tan n − 2 x  sec 2 x − 1 dx
=ztan n−2
x sec 2
x dx − z
tan n−2
x dx
=z
tan n−2
x sec 2 x dx − In − 2
For the first term, put tan x = t ∴ sec 2 x dx = dt
t n −1
z
Now In = t n − 2 dt −In − 2 =
n −1
− In − 2

tan n −1 x
Thus In = − In − 2     1
n −1

ztan x dx
 4
n
Next, let In =
0

Ltan x O
=M
n −1  4

Nn − 1 P
∴ In − I  by using (1)
Q 0
n−2

But tan ( / 4) = 1 and tan 0 = 0

ztan
 4 1
n
Thus In = x dx = − In − 2      2
0 n −1
1
or In + In − 2 = or n In +1 + In −1  = 1
n −1

Illustration
z
(i) To find tan 5 x dx

z
>> I5 = tan x dx =
tan 4 x
4
5
− I3  by using (1).

i.e., =
tan 4 x R
S

tan 2 x U
V
− I1
4 T 2 W
But I = z
1 tan x dx = z 1
tan x dx = log (sec x )

∴z
4 2
5 tan x tan x
tan x dx = − + log (sec x ) + c
4 2
Note : The reduction formula for z cot x dx can be established in a similar wayn

and the result is as follows.

36
− cot n −1 x
z
In = cot n x dx =
n −1
− In − 2

z
3. Reduction formula for sec n x dx where n is a positive integer

z
>> Let In = sec n x dx
=z
sec n−2
x  sec 2 dx
Integrating by parts we have,
z
In = sec n −2 x  tan x − tan x .  n − 2 sec n −3 x sec x tan x dx
= sec n−2
z
x tan x −  n − 2 sec n − 2 x tan 2 x dx
= sec n−2
x tan x −  n − 2 z
sec n−2
x  sec 2 x − 1 dx
= sec n − 2 x tan x −  n − 2 z
sec n
z
x dx +  n − 2 sec n − 2 x dx
In = sec n −2 x tan x −  n − 2 In +  n − 2 In − 2
i. e., In 1 +  n − 2 = sec n−2 x tan x +  n − 2 In − 2

z sec n− 2 x tan x n−2 F


G IJ
Thus In = sec n x dx =
n −1
+
n −1
In − 2
H K
zsec
 4
n
Next let In = x dx
0

Lsecn− x tan x O
=M
2
P Fn − 2 I
+ G JI
 4

N n − 1 Q Hn − 1 K
∴ In n−2
0 …(1)
But sec ( / 4) = 2  tan ( / 4) = 1, sec 0 = 1, tan 0 = 0

z
/4 2  n−2  n−2 F
G IJ
H K
n
Thus sec x dx = + In − 2     2
0 n −1 n −1

Illustrations
z
(i) To find sec 5 x dx

z
>> I5 = sec 5 x dx =
sec 3 x tan x 3
4
+ I3  by using (1).
4
3
sec x tan x 3 sec x tan x 1
+ + I1
R
S U
V
i.e., =
4 4 2 2 T W
z
But I1 = sec x dx = log (sec x + tan x )

37
z 5
∴ sec x dx =
sec 3 x tan x 3
4
3
+ sec x tan x + log (sec x + tan x ) + c
8 8

zsec
 4
4
(ii) To evaluate x dx
0

e2 j + 2 I
2 R
| e 2j U
|= 2 + 2 = 4
0

>> I 4 =
3 3
S + 0V
2 2
= +
|T 1 |W3 3 3
2
3 3

Note : The reduction formula for z n


cosec x dx can be established in
a similar way and the result is as follows.

z − cosec n − 2 x cot x n−2 F


G IJ
H K
n
In = cosec x dx = + In − 2
n −1 n −1

Session – 3
z zsin
/2
Reduction formula for sin m x cos n x dx and m
x cos n x dx
0
where m and n are positive integers.
z
Let I m, n = sin m x  cos n x dx 
=zsin x  sin x cos x  dx = z
m −1 n
uv dx  say
We have zuv dx = u z v dx − z
zv dx.u' dx
n +1
Here z
v dx = z
− cos x n
sin x cos dx = by putting cos x = t
n +1
Now I = (sin x  G
F−cos x I− z−cos x  m − 1 sin x cos x dx
n +1 n +1

Hn + 1 JK n + 1
m −1 m−2
m, n

sin m −1 x cos n +1 x m − 1
i. e., = −
n +1
+
n +1
z
sin m − 2 x cos n + 2 x dx
m −1 n +1
=−
sin x cos x m − 1
n +1
+
n +1
z
sin m − 2 x cos n x cos 2 x dx
m −1 n +1
=−
sin x cos x m − 1
n +1
+
n +1
z
sin m − 2 x cos n x (1 − sin 2 x  dx

sin m −1 x cos n +1 x m − 1
=−
n +1
+
n +1
z
sin m − 2 x cos n x dx −
m −1
n +1
z
sin m x cos n x dx

38
−sin m −1 x cos n +1 x m − 1 m −1
Im, n = + I m − 2 n − Im, n
n +1 n +1 n +1
L
M
m −1 O 1
N P
i. e., Im, n 1 +
n +1 Q =
n +1
−sin m −1 x cos n +1 x +  m − 1 Im − 2 n

L
M
m + nO 1
Nn + 1 P
m −1 n +1
Im, n
Q= n + 1 −sin x cos x +  m − 1 I m − 2 n

m −1 n +1
=z m sinn x cos x m − 1
∴ Im, n sin x cos x dx = − + I m −2 n
m+n m+n …(1)
m n m n −1
Note : If we decompose sin x cos x =  sin x cos x  cos x and integrate by parts taking
u = cos n −1 x, v = sin m x cos x we can obtain
−sin m +1 x . cos n −1 x n − 1
Im, n = + Im, n −2     2
m+n m+n

Note:
 m − 1  m − 3     n − 1  n − 3   
zsin
 2
m
x cos n x dx = ×k
0  m + n  m + n − 2   m + n − 4    
where k = π / 2 when m and n are even. This is known as Walli’s rule.

Illustrations

z  4  2  3  1
 2 8
 i sin 5 x cos 4 x dx = =
0 9 × 7 × 5 × 3 × 1 315

zsin  6  4  2   4  2
 2 1
7
 ii x cos 5 x dx = =
0 12 × 10 × 8 × 6 × 4 × 2 120

zsin  5  3  1  4  2
 2 8
6
 iii x cos 5 x dx = =
0 11 × 9 × 7 × 5 × 3 × 1 693

zsin  7  5  3  1  5  3  1
 2 5
8
 iv x cos6 x dx =  =
0 14 × 12 × 10 × 8 × 6 × 4 × 2 2 4096

Session – 4

Note for problems : Basic properties of definite integrals are the prerequisites for
working problems on reduction formulae.

Example –1

Evaluate zx sin
0
8
x dx

z
Let I = x sin 8 x dx
0

39
z z
a a
We have the property f  x  dx = f  a − x  dx
0 0

∴I =  z − x sin  − x
0
8
dx =z
0
 − x sin 8 x dx

= zsin x dx − zx sin x dx
0
8

0
8

zsin x dx − I or 2 I =  2 zsin
 2
8 8
I= x dx by a property.
0 0
7 5 3 1
Hence I =       by reduction formula.
8 6 4 2 2
35 2
Thus I =
256

Example –2

zcos 3x sin 6 x dx using reduction formula.


/6
4 2
Evaluate
0

>> Let I = zcos 3 x sin 6 x dx


/6
4 2

0
sin 6 x = 2 sin 3 x cos 3 x sin = 2 sin   2 cos (  2

zcos 3x  2 sin 3x cos 3x


/6
4 2
∴I= dx
0

i. e., I = 4 zsin 3 x cos 3 x dx


/6
2 6

0
Put 3 x = y ∴ dx = dy  3
If x = 0 y = 0  If x =  6 y =  2

zsin y cos y dy3 = 43 zsin y cos y dy


/2 /2
2 6 2 6
∴I=4
0 0

4L 1   5  3  1 O
I= M  Pby reduction formula.
3N 8×6× 4 ×2 2Q
5
Thus I =
192

40
Example – 3

Using reduction formula find the value of

zx  1 − x  dx
1
2 2 3 2

>> Let I = z
1
2 2 3 2
x  1− x  dx
0
Put x = sin ∴ dx = cos d and varies from 0 to  2
3 2 3 2
 1 − x2  =  cos 2  = cos3

zsin zsin
 2  2
2
∴ I= cos3 cos d = 2
cos 4 d
0 0
 1   3  1
Hence I =   by reduction formula.
6×4×2 2
Thus I =
32

Example – 4

zx z 2ax − x
x 2 dx
2a 2a
2
Evaluate (i) 2 ax − x 2 dx (ii)
0 0 2

zx
2a
2
>> Let I1 = 2 ax − x 2 dx
0

Put x = 2 a sin 2 ∴dx = 4 a sin cos d 


varies from 0 to  2
Also 2 ax − x 2 = 4 a 2 sin 2 − 4 a 2 sin 4
i.e., = 4 a 2 sin 2  1 − sin 2  = 4 a 2 sin 2 cos2 = 2 a sin cos

z4a
 2
2
∴ I1 = sin 4  2 a sin cos  4 a sin cos d
=0

zsin
 2
= 32 a 4 6
cos 2 d
0

 5  3  1
= 32 a 4   by reduction formula.
8×6×4×2 2
5 a4
Thus I1 =
8

41
z 4 a 2 sin 4
 2
ii) I2 =  4 a sin cos d
0 2 a sin cos

zsin
 2 3 1
= 8 a2 4
d = 8a 2    by reduction formula.
0 4 2 2
3 a2
Thus I2 =
2
Example – 5

z 1 +xx 
∞ 4
Evaluate 2 4
dx
0

z x4

>> Let I = 2 4
dx
0 1+ x 

Put x = tan ∴ dx = sec 2 d


If x = 0 = 0  If x = ∞ , =  2
Also (1 + x 2 ) 4 =  1 + tan 2  4
=  sec 2  4 = sec8

z z
/ 2 tan 4 / 2 tan 4
∴ I= 8
sec 2 d = 6
d
= 0 sec = 0 sec

zcos
sin 4
z
 2 /2
6
= 4
d = sin 4 cos 2 d
0 cos 0

 3  1   1
Hence I =  by reduction formula.
6×4×2 2
Thus I =
32
Exercises :

Evaluate the following integrals


zsin
 4
4
1 4 x cos3 2 x dx
0

2
0
zx sin 7
x cos 2 x dx

za x4
a
3 dx
2
0 − x2

z 1 +xx 
1 3
4 2 4
dx
0

5 z
∞ 6
x
2 9 2
dx
 1+ x 
0

Answers
1 128  1155 2 16  315 3 3 a 4  16 4 1  24 5 1  7

42
Session –5
Tracing of Curves
Introduction :
This topic gives an insight to the process of finding the shape of a plane curve based on
its equation by examining certain features. Based on these features we can draw a rough
sketch of the curve. It is highly essential to known the shape of the curve to find its
area, length, surface area and volume of solids.
List of important points to be examined for tracing a cartesian curve f (x, y) = 0
1. Symmetry : If the given equation has even powers of x only then the curve is
symmetrical about the y axis and if the given equation has even powers of y only
then the curve is symmetrical about the x-axis.
If f (x, y) = f (y, x) then the curve is symmetrical about the line y = x. Also if
f (x, y) = f ( −x, − y) then the curve is symmetrical about the origin.
2. Special points on the curve : If f (0, 0) = 0 then the curve passes through the
origin. In such a case we can find the equations of tangents at the origin by
equating the groups of lowest degree terms in x and y to zero.
The points of intersection of the curve with the x-axis is got by putting y = 0 and
that with the y-axis is got by putting x = 0.
3. Asymptotes : Asymptote of a given curve is defined to be the tangent to the given
curve at infinity. In otherwords these are lines touching the curve at infinity.
Equating the coefficient of highest degree terms in x to zero we get asymptotes
parallel to the x-axis and equating the coefficient of highest degree terms in y to
zero we get asymptotes parallel to the y-axis.
4. Region of existence : Region of existence can be determined by finding out the set
of permissible (real) values x and y. The curve does not lie in the region whenever
x or y is imaginary.
By examining these features we can draw a rough sketch of the curve.

Note : In the case of a parametric curve : x = x (t) and y = y (t), we need to vary the
parameter t suitably to take a note of the variations in x and y so that the curve can be
drawn accordingly.
List of important points to be examined for tracing a polar curve f ( r, θ) = 0
1. Symmetry : f ( r, θ) = f ( r, −θ) then the curve is symmetrical about the initial line
θ = 0 and θ = π.
If f ( r, θ) = f ( r, π −θ) then the curve is symmetrical about the line θ = π / 2 (positive
y-axis)

43
If f ( r, θ) = f ( r, π / 2 −θ) then the curve is symmetrical about the line θ = π / 4 (the line
y = x)
If f ( r, θ) = f ( r, 3 π / 2 −θ) then the curve is symmetrical about the line θ = 3 π / 4 (the
line y = −x)
If f ( r, θ) = f ( −r, θ) then the curve is symmetrical about the pole. (origin)
2. Curve passing through the pole : If r = 0 gives a single value of θ say θ1 between 0
and 2π then the curve passes through the pole once. θ = θ1 is a tangent to the curve at
the pole.
If it gives two values then the curve passes through the pole twice.
3 Asymptote : If r → ∞ as → 0 then the line = 0 is an asymptote.
4. Region of existence : If r is imaginary for θ∈ (α, β) i.e, α< θ<β then the curve does
not exist in the region between θ = α and θ = β.
5. Special points : We can tabulate a set of values of r for convenient values of θ. These
give some specific points through which the curve passes.
By examining these features we can draw a rough sketch of the curve

Example – 1
Trace the curve y2 (a −x) = x3, a > 0
>> We have y2 (a −x) = x3. (This curve is known as cissoid)
We observe the following features of the curve.
1. Symmetry : The equation contain even powers of y.
⇒ the curve is symmetrical about x- axis.
2. Special points : The curve passes through (0, 0).
The given equation is ay2 −xy2=x3.
The lowest degree term is ay2 and ay2 = 0 ⇒ y = 0 which is the equation of x-axis. Hence
x-axis is the tangent to the curve at the origin.
Putting y = 0 we get x = 0 and vice versa. This means that the curve meets the x-axis and
y-axis at the origin.
3. Asymptotes : Equating the co-efficient of the highest degree term in y i.e. coefficient
of y2 being a −x to zero we get x=a which is a line parallel to the y-axis. Hence x=a is an
asymptote. Also coefficient of the highest degree term in x is x3 whose coefficient is 1 ≠ 0.
This implies that there is no asymptote parallel to the x-axis.
4. Region of existence : y2 = x3 / (a −x)

∴ y = x 3  a − x.

44
This is positive if x > 0, a −x > 0 or x < 0, a −x < 0 i.e. x > 0, x < a, x < 0, x > a. Since
a > 0 the second case is not possible. Hence y is real if x > 0 and x < a which implies that
the curve lies in the interval 0 < x < a. Further as x increases y also increases.

The shape of the curve is as follows

Note : Since the curve meets the coordinate axes at the origin only, the origin called a
‘cusp’ with x-axis as the common tangent.

Example – 2
Trace the curve y2 (a −x) = x2 (a + x), a > 0
>> y2 (a −x) = x2 (a + x) (This curve is known as ‘Strophoid’.)
We observe the following features of the curve.
1. Symmetry : The equation contain even powers of y.
⇒ the curve is symmetrical about x- axis.
2. Special points : The curve passes through the origin. The equation of the curve can be
put in the form
a (y2 −x2) −x y2 −x3 = 0
Equating the lowest degree terms to zero we have a (y2 −x2) = 0
Hence y = + x which are the tangents to the curve at the origin. Since there are two
tangents, the origin is called a ‘node’.
Next putting y =0 we get x2 (a + x)= 0 x=0, x = −a.
The points are (0, 0) and ( −a, 0)
Also putting x = 0 we get ay2 = 0 or y = 0 and the point is (0, 0)
Hence we say that the curve intersects x-axis at (0,0) ( −a, 0) and intersects the y-axis at
(0, 0) only.
3. Asymptotes : The co-efficient of the highest degree term in x being x3 is −1 and hence
there is no asymptote parallel to the x-axis. Also the coefficient of the highest degree in

45
y being a −x, a −x = 0 gives x = a. Hence x = a is the only asymptote which is a line
parallel to the y-axis.

4 Region of existence : y = x 2  a + x   a − x.
When a+x < 0 and also when a −x < 0 y is imaginary.
Hence we can say that the curve lies between the lines x = −a and x = + a
The shape of the curve is as follows.

Example – 3
Trace the curve a y2 = x2 (a −x), a > 0
We observe the following features of the curve.
1. Symmetry : The equation contain even powers of y and hence the curve is
symmetrical about x- axis.
2. Special points : The curve passes through (0, 0). The equation of the curve is
ay2 −ax2 + x3 = 0. Equating the lowest degree terms to zero we have
a (y2 −x2) = 0 y=+x
These are the equations of the tangents to the curve at the origin.
If y = 0 then x2 (a − x) = 0, or x = 0, x = a and if x = 0, y = 0. Hence the curve meets
the x-axis at (a, 0) and meets the y-axis at (0,0) only.
3. Asymptotes : The coefficient of the highest degree terms in x and y are
respectively 1 and a. Since these are constants, there are no asymptotes.
4. Region of existence : y = x  a − x   a
y is positive if x > 0 and a −x > 0. x > 0 and x < a or 0 < x < a.
The curve lies between x = 0 and x = a.

46
The shape of the curve is as follows.

Session – 6
Example – 4
Trace the curve r = a sin 3  (Three leaved rose)
We observe the following features of the curve.
f (r, ) f (r, − ) the curve is not symmetrical about the initial line.
f (r, ) f ( −r, ) the curve is not symmetrical about the pole.
f (r, ) f (r,  − ) the curve is symmetrical about the line    .
r = 0 gives sin 3 0
   
 n or   
n  
Taking values for n = 0, 1, 2, …..6 we get the corresponding values of   
   

    
        and the curve passes through the pole for these values.
If 0<   
 r is positive and r = a if 
    
If <
  
   r is positive and r = 0 if 
  
  
If   
<    r is negative and r = −a if 
  
  
These observations implies that r increases from 0 to a as varies from 0 to r
  decreases
from a to 0 as  varies from to
  .
r increases numerically from 0 to a as varies from to
  
 
Further f(r 
   −  ) = f(r, ) implies that the curve is symmetrical about the line
= / 6 so that we conclude that there is a loop between the lines and
    

Similarly we can examine the path of the curve as moves from  to and also from
 to 2 .

47
Let us tabulate a set of values of r corresponding to some values of

           


r 0 a 0 −a 0 a 0 −a 0 a 0 −a

The curve is symmetrical about 


    and 3 2
The shape of the curve is as follows.

Example – 5
Trace the curve r2 = a2 cos 2 (Lemniscate of Bernoulli)
>> We observe the following features of the curve.
f (r, ) = f (r, − ) the curve is symmetrical about the initial line.
f (r, ) = f ( −r, ) the curve is symmetrical about the pole.
r = 0 gives a2 cos 2 = 0 i.e., cos2 = 0 2 
   and  

   and 
       are the tangents to the curve at the pole.
When = 0, r2 = a2 or r = + a.
Hence the curve meets the initial line at the points (+a, 0) and ( − a, 0).
Since the curve is symmetrical about the initial line it is composed of two loops. r is real
for          and    . Also r doesnot tend to infinity for any 
and hence there are no asymptotes.

The shape of the curve is as follows.

48
Application to find area, length and volume of solids of revolution

The relevent formulae for finding these are as follows.


1. Area : The area (A) bounded by a curve y = f (x) , the x-axis and the ordinates x = a
and x = b is given by

zy dx
b
A=
x=a

The area (A) between the curves y = f (x) and y = g (x) between x = a and x = b is given
by

z z
b b
A = f  x  dx − g  x  dx.
a a

The area (A) called the sectorial area, bounded by a polar curve r = f (  and the lines 
 
and 
  is given by

A=
1
2
zr d
2

1
2

2. Length : The length (s) of the arc of a curve between two specified points on it for
various types of the curves are given by the following formulae.
Such a process is called rectification and the entire length of the curve is called as the
perimeter of the curve.
(i) Cartesian curve y = f (x) or x = f (y)
b
Fdy I Fdx I
2
s = z 1 + G J dx or z 1 + G J dy
d
2

x=a Hdx K Hdy K y=c

(ii) Parametric curve x = x (t), y = y (t)

Fdx I Fdy I
2
s = z G J + G J dt
t2 2

t1Hdt K Hdt K
(iii) Polar curve r = f ( 

49
z r +Fdr I Fd I
2 2

G
Hd JKd or s = z 1 + r G J dr
2 r2

Hd r K
2 2
s=
= 1 r = r1

3. Surface area : When a curve revolves about the x-axis a surface is generated and
the same is called a surface of revolution. If a curve is bounded by the ordinates
x = a and x = b revolves once completely about the x-axis, the area of the surface
(S) generated is given by

z z ds
b b
S = 2 y ds= 2 y dx
x=a a dx

ds Fdy I
1 + GJ
2
where
dx
=
Hdx K
Similarly the surface area of revolution about the y-axis is given by

z2 z ds
d d
S= x ds = 2 x dy ,
y=c c dy

ds dx F
G IJ 2
where
dy
= 1+
dy HK
In the case of a polar curve the surface area of revolution about the initial line is given by

S= z2
=
2

1
r sin ds= 2 zr sin
2

1
ds
d
d

ds dr F
G IJ 2
where
d
= r2 +
d HK
4. Volume of revolution : The volume (V) of the solid generated by the revolution of the
curve y = f (x) between the ordinates x = a and x = b, about the x-axis is given by

zy dx
b
2
V=
x=a

Similarly if the axis of revolution is the y-axis, the volume of the solid is given by

zx dy
d
2
V=
y=c

Also in the case of a polar curve r = f ( ) the volume (V) of the solid generated is given
by

V=
2
3
z
r 3 sin d (revolution about the initial line)

V=
2
3
z
r 3 cos d (revolution about the line =  2)

50
Session – 7

Application related to standard curves

Example –1
The Astroid : Astroid is the curve represented by the equation
x 2/3 + y 2/3 = a 2/3
Its parametric equation is x = a cos3 and y = a sin3 .
We shall find its shape first and then determine the associated area, perimeter, surface
area and the volume.
(a) Trace curve x 2/3 + y 2/3 = a 2/3
>> Let us consider its parametric equation :
x = a cos3 y = a sin3 .
We tabulate x, y corresponding to certain angles of in the interval [0, 2 

    
 
x a 0 −a 0 a
y 0 a 0 −a 0

From the table we conclude that the curve meets the x-axis at the points (a, 0) and (
− a, 0). Also it meets the y-axis at the points (0, a) and (0, − a). Since | cos | < | and |
sin | < 1, we have | x | < a and | y | < a. Hence we infer that the entire curve lies within a
circle of radius ‘a’ having origin as the centre.
Also we have from the cartesian equation of the curve,
f (x, y) = f ( −x, y) ; f (x, y) = f (x, − y) ; f (x, y) = f (y, x)
Hence the curve is symmetrical about the coordinate axes and also about the line y = x.
Taking a note of the values of x and y as advances from one quadrant to the other the
shape of the curve is as follows.

51
(b) Find the area enclosed by astroid x 2/3 + y 2/3 = a 2/3
>> Note : In any problem on applications we need to draw the curve first by briefly
examining the important features.
The curve astroid is symmetrical about the coordinate axes and hence the required area
(A) is equal to four times the area in the first quadrant.

z z ddx d
a a
i.e., A = 4 y dx = 4 y
0 0

dx
We have x = a cos3  y = a sin 3 ∴ = −3a cos 2 sin
d
When x = 0 : a cos3 = 0 or cos3 = 0  
   
x = 1 : a cos3 = a or cos3 = 1  
 

za sin
0
3
∴ A=4  −3 a cos2 sin  d
0=  2

zsin
 2
= 12 a 2 4
cos 2 d
0

 3  1   1
= 12 a 2   by reduction formula.
6×4×2 2
Thus the area ( A) enclosed is 3 a 2  8 sq. units.

(c) Find the parameter of the astroid x 2/3 + y 2/3 = a 2/3


>> Since the curve is symmetrical about the coordinate axes, the perimeter (entire length)
of the curve is four times its length in the first quadrant.

Fdx I Fdy I
l = 4 z G J +G J d
 2
2 2

Hd K Hd K
=0

52
z 9a cos
 2
2 4
=4 sin 2 + 9a 2sin 4 cos 2 d
0

z 9a cos
 2
2 2
=4 sin 2  cos 2 + sin 2  d
0

z
 2
= 4 3a cos sin d
0

z
/2
= 6 a sin 2 d
0

L−cos 2 O
= 6a M
 2

N2 P Q = − 3a  cos 0
− cos 0 = −3a −1 − 1 = 6 a

Thus the perimeter of the curve is 6 a units.


(d) Find the surface area of revolution of the astroid x 2/3 + y 2/3 = a 2/3
>> Because of symmetry the required surface area is equal to twice the surface area by
the revolution of the first quadrant of the curve.

z zy dds d
a  2
∴ S = 2 × 2 y ds = 4
0 =0

ds Fdx I Fdy I
= G J + G J = 3a cos
2 2
But
d Hd K Hd K sin 

Hence S = 4 za sin  3a cos sin


 2
3
d
0

zsin
2
= 12 a 2 4
cos d
0
 3
= 12 a 2  by reduction formula.
5 × 3×1
Thus the required surface area = 12 a 2 / 5 sq. units.
(e) Find the volume of the solid generated by the revolution of the astroid x 2/3+y 2/3= a 2/3
>> Because of the symmetry the required volume (V) is equal to twice the volume of the
solid generated by the curve in the first quadrant about the x-axis.

z y dx = 2 zy ddx d
a a
2 2
∴ V =2×
0 0

za sin  −3a cos sin


0
2 6 2
=2  d
2

a zsin cos d
 2
3 7 2
=6
0

53
 6  4  2  1
= 6 a3 by reduction formula.
9 × 7 × 5 × 3 ×1
Thus the required volume of the solid is 32 a 3 / 105 cubic units.
Example –2
Cycloid : Cycloid is a curve generated by a point on the circumference of a circle which
rolls on a fixed straight line known as the base. Imagine a wheel rolling on a straight line
without slipping. A fixed point on the rim of the wheel traces the cycloid. The parametric
equation of the cycloid can be in the following forms :
 i x = a  − sin   y = a  1 − cos 
 ii x = a  + sin   y = a  1 − cos 
 iii x = a  − sin   y = a  1 + cos 
 iv x = a  + sin   y = a  1 + cos 

(a) Trace the cycloid :


x = a − sin   y = a  1 − cos 
>> Let us tabulate x, y for certain values of in the interval [0, 2 ] where is inradians.

    
 
x 0 a( 
 ) a a(3 
  ) 2a
y 0 a 2a A 0

From the table we can conclude that the curve intersects the x-axis at x = 0 and 2a .
Also, we have y = a (1cos ) and since | cos | < 1 y is non negative. Hence the curve
lies above the x-axis.
Taking a note of the values of x and y as advances in the interval [0, 2 ] the shape of
the curve is as follows. It is called an arch of the curve.

54
(b) Find the area of an arch of the cycloid :
x = a − sin   y = a  1 − cos 

zy ddx d
2
>> Area  A =
=0

z
2
i.e., A = a 1 − cos   a  1 − cos  d
0

z 1 − cos  d
2
= a2 2

z4 sin (  2 d
2
= a2 4

Put  2 = t ∴ d = 2 dt, t varies from 0 to 

z zsin
 2
∴ A = 8a 2 sin 4 t dt = 8a 2  2 4
t dt
t =0 t =0

3 1
i.e., A = 16 a 2    by reduction formula.
4 2 2
Thus the area enclosed by an arch of the curve on its base is 3 a2 sq.units.
(c) Find the length of an arch the cycloid :
x = a − sin   y = a  1 − cos 

zFdx I Fdy I
2 2

G
Hd JK+ G
Hd JKd
2
>> Length (l ) =
=0

z a  1 − cos  + a sin d
2
2 2 2 2
i.e., l =
0

= za  1 − 2 cos + cos + sin  d


2
2 2

= a z 2  1 − cos  d = a z 2 2 sin (
2 2
2
 2 d
0 0

= 2a z
2
L2a cos (  2 O
sin (  2 d = − M
2

0 N 1 2 P Q 0
= −4 a  cos − cos 0 = −4 a  −1 − 1 = 8a
Thus the required length is 8a

55
(d) Find the surface area generated by the revolution of an arch of the cycloid
x = a ( −sin ), y = a (1 −cos ) about the x-axis.

zy dds d
2
>> Surface area ( S = 2
0

ds Fdx I Fdy I 2
= G J + G J = 2 a sin ( / 2)
2
But
d Hd K Hd K
∴ S=2 z
2
a 1 − cos   2 a sin ( / 2) d
0

z zsin
2 2
= 4 a2 2 sin 3 ( / 2) d = 8 a 2 3
( / 2) d
0 0
Put  2 = t ∴ d = 2 dt and t varies from 0 to 

zsin zsin
2
Hence S = 8 a 2 3
t . 2 dt = 16 a 2  2 3
t dt
t=0 t=0
2
i.e., S = 32 a 2  by applying reduction formula.
3
Thus the required surface area is 64 a 2  3 sq. units.
(e) Find the volume of the solid generated by the revolution of the cycloid
x = a − sin   y = a  1 − cos 

zy ddx d
2
2
>> V =
0

= za  1 − cos
2
2
 2  a  1 − cos  d
0

= a zo t z
2 3 2
3 2
2 sin   2 d = 8 a 3 sin 6   2 d
0 0

= 8 a  2z 3
sin  6
 2 d = 16 a z
3
sin  6
 2 d
0 0

Put  2 = t ∴ d = 2 dt and t varies from 0 to  2

zsin
 2
∴ V = 16 a 3 6
t . 2 dt
0
5 3 1
= 32 a 3 
   , by reduction formula.
6 4 2 2
Thus the required volume is 5 2 a 3 cubic units.

56
Session –8

Example –3

Cardiode : r = a (1+cos )
(a) Trace the curve r = a (1+cos )
>> r = a (1+cos )
We observe the following features of the curve.
(i) f (r, ) = f (r, − ) and hence the curve is symmetrical about the initial line.
(ii) r = 0 when = and hence the curve passes through the pole. = is a
tangent to the curve at the pole.
(iii) Since | cos | < |, | r |< 2a and hence the curve lies with in the circle of radius
2a having its centre at the pole.
Let us tabulate r for certain angles of .
     
r 2a 3a/2 a a/2 0

It is evident thatas increases from 0 to , r decreases from 2a to 0. The shape of the


curve is as follows.

(b) Find the area of the cardioide r = a (1 + cos )


>> Since the curve is symmetrical about the initial line, the total area (A) is twice the area
above the initial line.

57
i.e., A = 2 z = za  1 + cos  d
1 2
20
r d
0
2 2

= a2 z2 cos (  2)] d = 4a zcos


0
2 2 2

0
4
(  2) d

Put  2 = t ∴ d = 2 dt and t varies from 0 to  2

zcos zcos
 2  2
∴ A = 4a 2 4
t. 2 dt = 8 a 2 4
t dt
t=0 t=0
3 1
= 8 a2   by reduction formula.
4 2 2
Thus the area enclosed is 3 a 2 / 2 sq. units.

(c) Find the perimeter of cardioide r = a (1 + cos )


>> Perimeter (length) = 2 (length of the upper half of the curve)

zds ds dr F
G IJ 2
i.e., =2
0d
where
d
= r2 +
d HK
ds
Now = a 2  1 + cos  2 + a 2 sin 2 = a 2  1 + cos 
d
= 2 a cos (  2

z L
sin ( / 2) O
M
∴ perimeter = 2 2 a cos( / 2) d
0
= 4a
N1  2 P Q= 8a 0
Thus the perimeter of the curve is 8a units.

(d) Find the surface area of the revolution of the curve about the initial line.

>> Surface area  S = 2 r sin z ds


d
d

ds dr F
G IJ = 2a cos (
2
But
d
= r2 +
d HK  2)

∴ S=2 za 1 + cos


=0
 sin  2 a cos (  2) d

=4 a z2 cos 
2 2
 2  2 sin (  2 cos(  2) cos(  2) d
=0

58
z
= 16 a 2 cos 4 
0
 2 sin (  2 d

Put  2 = t ∴ d = 2 dt and t varies from 0 to  2.

zcos
 2
Hence S = 16 a 2 4
t sin t . 2 dt
t=0
 3  1
= 32 a 2  by reduction formula.
5×3
Thus the required surface area is 32 a 2  5 sq. units.

(e) Find the volume generated by the revolution of the curve r = a (1 + cos ) about
the initial line.

>> V =
2
3 0
z
r 3 sin d =
2
3 0
z
a 3  1 + cos  3 sin d

Put t = 1 + cos ∴ dt = −sin d


If = 0 t = 2 and if =  t = 0

∴V =
3 2
z
2 a3 0 3
t  − dt  = z
2 a3 2 3
3 0
t dt

=
2 a3 L
Mt O 2 a
P
4 2
=  4 − 0 =
8 a 3 3

3 N4 Q 3 0
3
Thus the required volume is 8 a 3  3 cubic units.

Example – 4
Regarding sphere as the solid generated by revolving a circle about a diameter, find the
volume of a sphere of radius ‘a’
>> Let x2 + y2 = a2 be the equation of the circle and when the semicircle revolves about
the diameter (x-axis) a sphere of radius ‘a’ is generated.
Volume V = zy dx 2

z z a − x  dx
a a
V = y 2 dx = 2 2

−a −a
a
= a 2 x −  x 3  3 = 4 a3  3
−a

Thus the required volume is 4 a 3  3 cubic units.

59
Example – 5
Find the volume generated by the parabola y2 = 4 ax when revolved about the y-axis
between y = 0 and y = 2a
>> Required volume (V  = zx dy
2

z y  4a dy
2a
2 2
i.e., V =
y=0

=
L
My O
P
5 2a

16 a N 5Q
2
0

32 a 5
=  2 a 5 =
80 a 2 80 a 2
Thus the required volume is 2 a 3  5 cubic units.

Example – 6
Find the perimeter of the curve r = a sin3 ( / 3)
>> Let us tabulate r for certain angles of where r = a sin3 ( / 3)

    
   

r 0 a/8 3 3a 8 A 3 3a 8 a/8 0

zr
3
2 2
∴ perimeter  l  = +  dr / d  d
=0

Fdr I 2
+ G J = a sin  o t 2

Hd K
2 2 6
r  3 + a. 3 sin 2   3  cos   3  1  3

= a 2 sin 6   3 + a 2 sin 4   3 cos 2   3


2 4 2 2
= a sin   3 sin   3 + cos   3
2 4
= a sin   3
∴ r 2 +  dr / d  2
= a sin 2   3

60
za sin
3
2
Hence, perimeter  l  =   3 d
0
Put  3= ∴ d = 3d and varies from 0 to 

z zsin
2
∴ l =a sin 2  3d = 3 a 2 2
d
=0 0

1
i.e., l = 6 a .  by reduction formula.
2 2
Thus the required perimeter is 3a / 2 units.

Exercises
1. Show that the length of the arc from = 0 to = along the curve

x = a (cos + sin ), y = = a (sin − cos ) is a /2
2. Find the surface area of a sphere of radius a.
3. Show that the surface area generated by the revolution of the loops of the curve
r2 = a2 cos 2 about the initial line 2 a2 ( 2 − 2 ).
4. Find the volume of the sphere of radius ‘a’.
5. Find the volume of the solid generated by the revolution of a loop of the curve
r2= a2 cos 2 about the initial line.

Answers
3
2. 4 a2 4. 4 a3/3 5. 2 a  15

61
Infinite Series
Session - 1

Introduction:
Infinite series is basically a summation of infinite number of terms. The summation will
be meaningful if there is no significant change in the sum as more and more terms gets
added up which is the concept of convergence. In this topic we discuss the aspect of
convergence, divergence and oscillation of an infinite series which are attributed as the
nature or the behaviour of an infinite series. In this connection we make use of well
established tests so as to draw a valid conclusion on the nature of an infinite series.
‘Progression’ is the pre-requisite for this topic
Definitions
If un is a function of n defined for all integral values of n, an expression of the form
u1 + u2 + u3 +... + un containing infinite number of terms is called an infinite series


usually denoted by ∑ un or simply ∑ un
n =1

un is called the nth term or the general term of the infinite series. The sum of the first n
terms of the series is denoted by sn
i.e., sn = u1 + u2 + u3 +    +u n
Convergence, Divergence and Oscillation
A series ∑ un is said to be convergent if lim sn = l where l is a finite quantity.
n→∞

∑ un is said to be divergent if lim sn = ± ∞


n→∞
A series ∑ un is said to be Oscillatory if sn tends to more than one limit as n → ∞
Geometric series as an example for convergence, divergence and oscillation.
Let us consider the geometric series a + ar + ar2 + …to discuss its nature for various
values of r. The sum to n terms of this series is given by

62
a 1 − r n 
sn = if r < 1     1
1− r
a r n − 1
and sn = if r > 1     2
r −1
Now if r|< 1 r n → 0 as n → ∞ and from (1), we have
a a
lim sn =  1 − 0 = which is a finite quantity.
n→∞ 1− r 1− r
Hence we conclude that the geometric series is convergent for | r |< 1.
a r n − 1
Next if r > 1 from (2) lim sn = lim =∞
n→∞ n→∞ r −1
n
If r > 1 r → ∞ as n → ∞
Hence we conclude that the geometric series is divergent if r > 1
Also if r = 1 the series becomes a + a + a +…
sn = a + a + a… to n terms = na
∴ lim sn = lim n a = ∞
n→∞ n→∞

Hence the series is divergent if r = 1.


When r = −1, the series become a −a + a … where the sum of the first n terms sn is 0 or
' a' according as n is even or odd and we can say that lim sn = 0 or a. sn tends to more
n→∞

than one limit as n ∞

Hence we conclude that the series is oscillatory if r = −1.


Further if r < −1, rn  ∞ or − ∞ according as n is even or odd. sn tends to more than one
limt as n ∞

Hence we conclude that the series is oscillatory if r < −1.


Note : An infinite series of positive terms is either convergent or divergent.
Property : If ∑ un is convergent, then lim un = 0, but not conversely.
n→∞

An example to show that the converse is not true.

63

Consider the series ∑ n + 2 − n +1
n=1

Here un = n + 2 − n + 1 =
en + 2 − n +1jen + 2 + n +1 j
en + 2 + n +1j
 n + 2 −  n + 1 1
i.e., un = =
n + 2 + n +1 en + 2 + n +1 j
Clearly lim un = 0
n→∞
Next sn = u1 + u2 + u3     +un −1 + un
i. e., sn = e3 − 2 j+ e4 − 3 j+ e5 − 4 j+    + en + 1 − n j+ en + 2 − n +1 j
i. e., sn = − 2 + n + 2
∴ lim sn = lim − 2 + n + 2 = ∞
n→∞ n→∞

Hence we conclude that un is divergent.


This example shows that the converse of the property is not true.

Remarks :

1. The contrapositive of the statement of this theorem provides a fundamental test


for the divergence of an infinite series which can be stated as follows :
If un does not tend to zero as n tends to infinity, then un is not convergent.
Thus lim un ≠ 0 ⇒ ∑ un is divergent.
n→∞

2. Addition or deletion or multiplication of each term of infinite series by a nonzero


constant doesnot alter the nature of infinite series.

Tests for convergence for series of positive terms


Comparison Test
If ∑ un and ∑ vn be any two series of positive terms such that
un
lim is a non zero finite quantity. Then ∑ un and ∑ vn behave alike.
n→∞ v n

That is ∑ un and ∑ vn both will converge or both will diverge.

64
Remark : In this test the given series un is to be compared with another series vn
whose nature is to be known to us. We usually compare with the harmonic series

∞ 1
∑ ( p − series) which is convergent for p > 1 and divergent for p ≤ 1
n =1 np
(The proof is established later)

Working procedure for problems

• Given an infinite series, we first write the nth term (general term) un.
[The nth term of the A.P; a, a+ d, a+2 d, … is a+(n − 1)d ]

• Comparison test is usually applicable when un involves expression of n in the


numerator and the denominator like (n+1), (2n+1), (n2+1), n , n3 + 4 etc.
1
• We choose vn = p − q where p and q are the highest power n in the denominator
n
and numerator of un respectively.
• However if un is of the form f  n − g  n or f  n − g n  irrational factors connected
by a negative sign), we must rationalize by multiplying with f  n + g n or
f  n + g n as the case may be and then chose vn as stated.
Order Test
1
We say that p− q
is the order of un . If ( p − q ) = k (say) then we say that un is of the order
n
1
and conclude that ∑ un is converngent if k > 1 and divergent if k ≤ 1.
nk
Sometimes this is referred to as the order test.
Session – 2

Example – 1

65
1 2 3
Examine the series + + +     for convegence.
1.3.5 3 5 7 5 7 9
>> Numerators are 1, 2, 3, .... Hence the general term is n.
In the denominator we have,
First factors : 1, 3, 5, ... ∴ n th term = 1 +  n − 1 2 = 2 n − 1
Second factors : 3, 5, 7, ... ∴ n th term = 3 +  n − 1 2 = 2 n + 1
Third factors : 5, 7, 9 ... ∴ n th term = 5 +  n − 1 2 = 2 n + 3
Thus n th term of the given series is given by
n n 1
un = Choose vn = 3 = 2
 2n − 1  2 n + 1  2n + 3 n n
un n n2
Thus = ×
vn  2 n − 1  2n + 1  2 n + 3 1
un n3 1
Now lim = lim 3 =
n→∞ vn n→∞ n  2 − 1  n  2 + 1  n  2 + 3  n 8
By comparision test, ∑ un and ∑ vn behave alike.
1 1
Here ∑ vn = ∑ 2
is of the form ∑ p  p = 2 > 1 is convergent.
n n
Hence ∑ un is also convergent.

Example – 2
Find the nature of the series
1 1+ 2 1+ 2 + 3
2
+ 2 2
+ 2 +  
1 1 +2 1 + 22 + 32
>> It should be observed that there will be n terms in the numerator and the
denominator of the general term un.

66
1 + 2 + 3+    + n ∑n
∴ un = 2 2 2 2
= 2
1 + 2 + 3 +   + n ∑n
n n + 1 n n + 1  2n + 1
i.e., un = 
2 6
n n + 1 6 3 1
i.e., un =  = Choose vn =
2 n n + 1  2n + 1 2n + 1 n
u 3 n
Now lim n = lim 
n→∞ vn n→∞ 2 n + 1 1

F
G 3 I 3
= lim
n→∞ H2 + 1  n JK= 2
By comparison test, ∑ un and ∑ vn behave alike.
1 1
But ∑ vn = ∑ = ∑ 1  p = 1 is divergent.
n n
Hence ∑ un is also divergent.

Example – 3
∞ 1
Test for convegence ∑
n =1 n +1+ n
1 1
>> By data un = Choose vn =
n +1 + n n
un 1 n
Now lim = lim 
n→∞ vn n→∞ n +1+ n 1
n n 1
= lim = lim =
n→∞ n  1 + 1  n + n n→∞ n 1 + 1 n + 1 2

By comparison test ∑ un and ∑ vn behave alike.


1
But ∑ vn = ∑ ( p = 1 / 2 < 1) is divergent.
n1/2
Hence ∑ un is also divergent.
Note : Alternative version of the problem.

67
en + 1 − n j

Discuss the convergence of the infinite series ∑
n =1

>> un = n + 1 − n and on rationalizing we have

en + 1 − n j en + 1 + n j =
n +1+ n n + 1− n
un =
n +1+ n
1
i.e., un = which is the same as the example discussed.
n +1+ n

Example – 4
L
Mn + 1 − n O

P
N Q
3 3
Test for convergence ∑
n =1

=L
Mn + 1 − n O
P
N Q
3 3
>> By data, un

We have an elementary formula


a3 − b3
a 3 − b 3 =  a − b  a 2 + ab + b 2  or  a − b =
a 2 + ab + b 2
If we take a = 3 n 3 + 1 and b = n, un assumes the form  a − b 
 n 3 + 1 − n 3
∴ un = by above.
 n 3 + 1 2 3
+ 3 n3 + 1  n + n2
1 1
i.e., un = Choose vn =
3
 n + 1 2 3 3 3
+ n +1 n+ n 2 n2

un 1 n2
Now lim = lim ×
n→∞ vn n→∞  n 3 + 1 2 3
+ 3 n3 + 1  n + n2 1
n2
= lim 2 3
n→∞
n3  1 + 1  n3  + 3 n3  1 + 1  n3   n + n 2

n2
= lim
2 3 3
n→∞ n2  1 + 1  n3  + n2 1 + 1  n3 + n2
n2 1 1
= lim
L
M O
P
= =
1+1+1 3
N + 3 1 + 1  n3 + 1
Q
2 3
n→∞ n2  1 + 1  n3 

By comparison test ∑ un and ∑ vn behave alike.


Hence ∑ un is also convergent.

68
Exercises :
Find the nature of the following series
1 4 7
1 + + +  
1 2 3 2 5 7 3 8 11
12 12 + 22 12 + 2 2 + 32
2 3 + 3 + +  
1 1 + 2 3 13 + 2 3 + 33
1 1 1
3 + + 
3+ 5 4+ 6 5+ 7
L
Nn + 3 − n O
M P
Q
4 2
4 ∑

Answers :
1. Convergent 2. Divergent 3. Divergent 4. Convergent

Session – 3

D’Alembert’s ratio test and Raabe’s test


If un is a series of positive terms and if
un +1
lim = l (finite qty)
n→∞ un

then un is convergent if l < 1, divergent if l > 1 and the test fails if l = 1.


When the ratio test fails, we can apply the comparison test (order test) if possible or
Raabe’s test stated as follows.
If un is a series of positive terms and if
F
G u I
− 1J = l (finite qty)
Hu K
n
lim n
n→∞ n +1

then un is convergent if l > 1, divergent if l < 1 and the test fails if l = 1.


Remark :
Usually we try ratio test in the following cases.

(i) nth term of the series is not of the order 1/nk.


(ii) nth term of the series involves variables like xn, x2n etc.,

69
(iii) nth term of the series involves n !, (n+1) !, (n !)2 etc.
(iv) The number of factors in the numerator and denominator increase steadily from term

1 1 2 1 2 3
to term like + + +     (carry over series)
3 3 5 3 5 7

Working procedure for problems


We first write the general term un for the given series and find un+1 by replacing n
by (n+1)
un+1
• We find the ratio  simplify and find its limit as n → ∞ to decide the nature of the series.
un

When the ratio test fails we can try to apply the order test in the first instance
or
apply Raabe’s test.
Raabe’s test becomes inevitable when the ratio test fails in the case of carry over
series.

Example – 1
x x2 x3
+ + +   
1.2 2 3 3 4
xn
>> un =
n n + 1
x n +1
∴ un +1 =
 n + 1  n + 2 
un +1 x n +1 n n + 1 n
Now = = x
un  n + 1  n + 2 xn n+2
u n 1
∴ lim n +1 = lim x = lim x=x
n→∞ un n→∞ n + 2 n→∞  1 + 2  n

R
S
convergent if x <1
∴ by D' Alembert' s ratio test ∑ un is
T
divergent if x > 1
and the test fails if x = 1
1n 1 1
But when x = 1 un = = = 2
n n + 1 n n + 1 n +n

70
un is of order 1 / n 2 ( p = 2 > 1) and hence ∑ un is convergent.
Hence we conclude that
∑ un is convergent if x ≤ 1 and divergent if x > 1

Example – 2
Find the nature of the series
1 1 3 2 1 3 5 3
x+ x + x +  
2 2 4 2 4 6
1 3 5    2n − 1 n
>> un = x     1
2 4 6   2n
1 3 5   2 n + 1 − 1 1 3 5   2 n + 1 n +1
un +1 = x n +1 = x
2 4 6   2 n + 1 2 4 6   2 n + 2
1 3 5    2 n − 1  2 n + 1 n+1
i.e., un +1 = x     2
2 4 6    2 n  2 n + 2
From (1) and (2) we have,
un +1 1 3 5    2 n − 1  2n + 1 n +1 2 4 6   2 n
= x ×
un 2 4 6    2 n  2 n + 2 1 3 5    2 n − 1 x n
u 2n + 1
i.e., n +1 =  x ...  3
un 2n + 2
un +1 2n + 1 n  2 + 1  n
∴ lim = lim  x = lim  x=x
n→∞ un n→∞ 2 n + 2 n→∞ n  2 + 2  n

Hence by ratio test


R
S
convergent if x < 1
T
∑ un is divergent if x > 1 and the test fails if x = 1

Now when x = 1 we shall apply Raabe' s test.


un +1 2 n + 1 u 2n + 2
When x = 1 from (3) = ∴ n =
un 2n + 2 un +1 2 n + 1
F
G u IJ F
G2n + 2 I F2n + 2 − 2n − 1IJ
− 1J = lim n G
Hu K
− 1 = lim n
H2n + 1 K H 2n + 1 K
n
Now lim n
n→∞ n +1 n→∞ n→∞

n 1
= lim
= <1
n→∞ n  2 + 1  n 2
∴ ∑ un is divergent by Raabe' s test Hence we conclude that the given series
∑ un is convergent if x < 1 and divergent if x ≥ 1

71
Example –3
∞ 3 6 9   3n 5n
Test for convergence ∑ 
n =1 4 7 10    3n + 1 3n + 2
3 6 9   3n 5n
>> By data un = 
4 7 10    3n + 1 3n + 2
3 6 9   3 n + 1 5n +1
∴ un +1 = 
4 7 10   3 n + 1 + 1 3 n + 1 + 2
3 6 9   3n 3n + 3 5n +1
i. e., un +1 = 
4 7 10    3n + 1  3n + 4 3n + 5

un +1 3 6 9    3n  3n + 3 5n +1 4 7 10    3n + 1  3n + 2
Now =  ×
un 4 7 10    3n + 1  3n + 4 3n + 5 3 6 9   3n 5n
u  3n + 3  3n + 2
i.e., n +1 =  5
un  3n + 4  3n + 5
un +1  3n + 3  3n + 2
∴ lim = lim  5
n→∞ un n→∞  3n + 4  3n + 5
 3 + 3  n   3 + 2  n
= lim  5 = 5 >1
n→∞  3 + 4  n   3 + 5  n

Hence by the ratio test ∑ un is divergent.

Example – 4

72
∞ 4 7 10    3n + 1 n
Find the nature of the series ∑ x
n =1 n
4 7 10    3n + 1 n
>> By data, un = x     1
n
4 7 10   3 n + 1 + 1 n +1 4 7 10    3n + 4 n +1
un +1 = x =  x
 n + 1   n + 1 
4 7 10    3n + 1  3n + 4 n +1
i.e., un +1 =  x     2
 n + 1  n 
From (1) and (2) we have,
un +1 4 7 10    3n + 1  3n + 4 n +1 n
=  x ×
un  n + 1  n  4 7 10    3n + 1  x n
u 3n + 4
i.e., n +1 =  x
un n +1
un +1 3n + 4 3+ 4 n
∴ lim = lim  x = lim  x = 3x
n→∞ un n→∞ n + 1 n→∞ 1 + 1  n

R
S
convergent if 3 x < 1 or x < 1  3
Hence by ratio test ∑ un is
T
divergent if 3 x > 1 or x > 1  3
and the test fails if 3 x = 1 or x = 1 / 3
u  3n + 4 1 u 3n + 3
If x = 1  3 we have n +1 = or n =
un  n + 1 3 un+1 3n + 4
F I F3n + 3 − 1IJ = lim n F −1 I
Hu K H3n + 4 K H3n + 4 JK= lim
− 1J = lim n G G −1 −1
Now lim n
n→∞
G u n

n+1 n→∞ n→∞ n→∞  3 + 4  n


=
3
<1

Hence ∑ un is divergent when 3 x = 1 by Raabe' s test.


Thus ∑ un is convergent if x < 1 / 3 and divergent if x ≥ 1 / 3.

Example – 5
Test for convergence of the infinite series
2 3 4
1+ 2
+ 3 + 4 +  
2 3 4
>> The first term of the given series can be written as 1!/11 so that we have,
n  n + 1   n + 1  n  n
un = n
∴ un +1 = n +1
= n +1
= n
n  n + 1  n + 1  n + 1

73
un +1 n nn nn
Now =  = n
un  n + 1 n n  n  1 + 1  n n

u 1 1
∴ lim n +1 = lim n
= <1
n→∞ un n→∞  1 + 1  n e
Hence by ratio test ∑ un is convergent.

Exercises
3 32 33
1 1+ + + +  
1 2  3
2 3 2 4 3
2 x+ x + x +  
3 4 4 5 5 6
1 1 2 2 1 2 3 3
3 x+ x + x +  
3 3 5 3 5 7
3 x 3 6 x 2 3 6 9 x 3
4  +  + +  
4 5 4 7 8 4 7 10 11
∞ 3 5 7    2 n + 1
5 ∑ xn
n =1 3 6 9   3n

Answers

1. Convergent
2. Convergent if x < 1, divergent if x > 1
1. Convergent if x < 2, divergent if x > 2
2. Convergent if x < 1, divergent if x > 1
3. Convergent if x < 3/2, divergent if x > 3/2

Session - 4
Cauchy’s root test
If un is a series of positive terms and if
1 n
lim  un  = l (finite)
n→∞

then the series is convergent if l < 1, divergent if l > 1 and the test fails if l = 1.
When the test fails we can try the order test or the fundamental test.
Note : 1. We try to apply this test if un is of the form [f (n)]g (n)
2. The following standard limits will be useful

74
F
G 1I
n
F xI
n
(i) lim n1  n = 1
H n JK= e; G
H n JK= e
x
 ii lim 1 +  iii lim 1 +
n→∞ n→∞ n→∞

Example – 1
∞ L1 + 1 O
∑ M
− n3  2

Test for convergence


n =1 N nP Q
>> u = M
L1 + 1 O − n3  2

N nP
n
Q by data.
R
| L 1 O U |V − n3  2 1 n

∴u  =S M
1 n
1+ P
n
|TN n Q |W
L1 + 1 O
=M
L1 + 1 O − n1  2
− n

N nQ N nP P =M
1 n
u 
n
Q
L1 + 1 O
= lim M
− n

N nP
1 n
Now lim  u 
n→∞
n
Q n→∞

1 1
= lim = < 1
n→∞
L
M 1 O
n e
N nP
1+
Q
 as n → ∞, n also → ∞
Hence by Cauchy' s root test ∑ un is convergent.

Example – 2
 n + 1 n x n ∞
Discuss the convergence of ∑
n =1 n n +1
 n + 1 n x n
>> un = by data.
n n +1

∴ (un ) 1/ n
=
o n + 1 t n 1 n
 xn 1 n
 n + 1 x  n + 1 x
= =
n  n+1 1  n
n1+1  n n n1  n
1/ n  n + 1 x n 1 + 1  n x  1 + 1  n x
lim (un ) = lim 1 n
= lim 1 n
= lim
n→∞ n→∞ nn n→∞ nn n→∞ n1  n

75
lim  1 + 1  n x  1 + 0 x
1/ n n→∞ 1 n
lim (un ) = 1 n
= = x, lim n = 1
n→∞ lim n 1 n→∞
n→∞

convergent if x < 1 R
S
Hence by Cauchy' s root test ∑ un is
divergent if x >1 T
and the test fails if x = 1
 n + 1 n
When x = 1,un =
n n +1
un is of order n n / n n +1 = 1 / n and hence ∑ un is divergent.
Thus ∑ un is convergent if x < 1 and divergent if x ≥ 1.

Example – 3
Find the nature of the series

F
G I 2

HJKx
2 3 2
1+ x+ +    x > 0
3 4
Fn + 1 I
>> Omitting the first term, u = G J x
n

Hn + 2 K
n
n

R
|F n + 1I U |V  x  = F n + 1I
n 1 n
∴ u n =S G J
|THn + 2 K|W
1 n
G
Hn + 2 JKx
n 1 n

1 n  1 + 1  n
Now lim  un  = lim x=x
n→∞ n→∞  1 + 2  n

convergent if x < 1 R
S
Hence by Cauchy' s root test ∑ un is
divergent if x > 1 T
and the test fails if x = 1

76
Fn + 1 I Fn + 1 I L 1 + 1  n O
But when x = 1 u = G J 1 = G J = M
n n n

Hn + 2 K Hn + 2 K N  1 + 2  n P
n
n
Q
 1 + 1  n e 1 n
F1 + x IJ = e
= using lim G
n

H nK
x
lim u = lim
n = n 2
n→∞  1 + 2  n
n→∞ e e n→∞

1
Since lim un = ≠ 0 ∑ un is divergent when x = 1
n→∞ e
Thus ∑ un is convergent if x < 1 and divergent if ≥ 1.

Example – 4
1 1 1
Test for convergence + 2 + 2 +  
2 2 3
>> The first term of the series can be put in the form 1 / 11 so that we have,
1
un = n
n
F1 I 1 n

lim G J = lim = 0 < 1


1
Hn K
1 n
∴ lim  un  = n
n→∞ n→∞ n n→∞

Hence by Cauchy' s root test ∑ un is convergent.

Exercises :
Tests for convergence the following series

F1 + 3 IJ
∑ G
− n2
1
H nK
Fn + 1I F3 I
∑ G J GJ
n2 n
2
Hn K H4 K
4 F 7I F 10 I F
13 I
2 3 4
+ GJ + GJ + GJ +   
3 H 5K H 7K H 9K
3

x x2 x3
4 1 + + + +    x > 0
2 32 4 3

Answers :

77
(1) Convergent (2) Divergent (3) Divergent (4) Convergent

Cauchy’s integral test


If ∑ un is a series of positive terms and if ux = f ( x ) be such that
(i) f ( x ) is continuous in 1 < x < ∞
(ii) f ( x ) decreases as x increases [ f ( x ) is monotonically decreasing] then the series ∑ un is

zf ( x) dx is finite or infinite.

convergent or divergent according as the integral
1
Note : The condition f ' ( x ) < 0 ensures that f ( x ) decreases as x increases where f ' ( x )
is the derivative of f ( x ).
Example :
Apply Cauchy’s integral test to discuss the nature of the harmonic series (p- series)
∞ 1

n =1 np
1
>> By data un = = f  n
np
1 −p
f  x =
p
= x − p  ∴ f '  x  = − px − p −1 = p +1 < 0
x x
Hence f  x  is decreasing.

z z z L
x1− p
M O ∞

N P
∞ ∞ 1 ∞
Now f  x  dx = p dx = x − p dx = if p ≠ 1
1 1 x 1 1− p Q 1
Case - i : Let p > 1 or ( p − 1) > 0
1
Then x 1− p = p −1 → 0 as x → ∞  p − 1 > 0
x
Lx O 1 = 1 which is finite.
1− p
∴ zf  x  dx =M P= 0 −

1 N1− pQ 1− p p −1
1
Thus ∑ un is convergent if p > 1.

78
Case - ii : Let p < 1 or 1 > p i.e., (1 − p) > 0
Lx O
As above zf  x  dx =M P
∞ 1− p ∞

1 N1− pQ 1
1− p
But x → ∞ as x → ∞. (1 − p) > 0

zf  x dx = ∞ − 1 −1 p = ∞

Hence
1

Thus ∑ un is divergent if p < 1.


Case - iii : Let p = 1

zf  x dx = zx1 dx = log x
∞ ∞

1 1 =∞−0=∞
1 1
Thus ∑ un is divergent if p = 1.
∞ 1
Conclusion : ∑ p is convergent if p > 1 and divergent if p ≤ 1.
n=1 n
Remark : We have used the behaviour of this series while solving problems on comparison test
∞ 1
to discuss the nature of a given series in comparison with a series of the form ∑ p
n=1 n

Session – 5

Alternating series
If u1 , u2 , u3 , u4 ...are all positive, then a series of the form u1 − u2 + u3 − u4 + ... is called an

n −1
alternating series represented by ∑  −1 un
n =1

Leibnitz test for convergence of an alternating series


If u1 , u2 , u3 , u4 ...are all positive and if
(i) un > un+1 , for all n i.e., u1 > u2 > u3 > u4 ...
(ii) lim un = 0
n→∞

then the alternating series u1 − u2 + u3 − u4 +... is convergent.


Remark : If lim un ≠ 0 then the series is oscillatory.
n→∞

79
Absolute convergence and conditional convergence
The alternating series u1 − u2 + u3 − u4 +... is said to be absolutely convergent if the series of
positive terms u1 + u2 + u3 + u4 +... is convergent.
If the given alternating series is convergent and the absolute series (series of positive
terms) is divergent then the alternating series is said to be conditionally convergent.
Property : Every absolutely convergent series is convergent but the converse is not true.

Generalised D’Alembert’s test


un +1
If ∑ un is a general series and if lim =| l | then ∑ un is absolutely convergent if | l | < 1
n→∞ un
not convergent if | l | > 1 and the test fails if | l | = 1.

Example – 1
1 1 1
Test for convergence the series, 1 − + − +  
2 3 4
1
>> Here un = We shall apply Leibnitz test.
n
1 1 n +1− n 1
Now un − un +1 = − = = >0
n n + 1 n n + 1 n n + 1
i.e., un − un +1 > 0 ⇒ un > un +1
1
Also lim un = lim =0
n→∞ n→∞ n
Hence the given series is convergent by Leibnitz test.

Remark : We have mentioned a property that “every absolutely convergent series is

1 1 1
convergent”, but the converse is not true. The series 1 − + − +   serves as an
2 3 4
example for the converse of the statement not being true. That is a convergent series not
being absolutely convergent.

80
1 1
The alternating series is convergent by Leibnitz test, but the absolute series 1 + + +  
2 3
∞ 1
being ∑ is divergent . ( p - series where p = 1
n =1 n
Thus the alternating series is not absolutely convergent.

Example –2
1 1 1
Test the series 1 − + −
+... for
2 2 3 3 4 4
(a) convergence (b) absolute convergence (c) conditional convergence
1 1 1
>> We have un = = 3  2 ∴ un +1 =
n n n  n + 1 3  2
1 1
Now un − un +1 = 3  2 −
n  n + 1 3  2
 n + 1 3  2 − n 3  2
= > 0 since (n + 1) > n.
n n + 1 3  2
i.e., un − un +1 > 0 ⇒ un > un +1
1
Also lim un = lim =0
n→∞ n n
n→∞

Hence the given series is convergent by Leibnitz test.


1 1 1
The absolute series is given by 1 + + + +...
2 2 3 3 4 4
1 1
We have un = = 3 2
n n n
∞ ∞
3 2 p
∑ 1  n is a harmonic series of the type ∑ 1  n  p = 3  2 > 1 is convergent.
n =1 n =1

Thus the alternating series is absolutely convergent and it is not conditionally


convergent.

Example – 3

81
3 5 7 9
Find the nature of the series − + − +  
4 7 10 13
2n + 1 2 n + 1 + 1 2 n + 3
>> We have un = ∴ un +1 = =
3n + 1 3 n + 1 + 1 3n + 4
2n + 1 2n + 3 1
Now un − un +1 = − = >0
3n + 1 3n + 4  3n + 1  3n + 4
i.e., un > un +1
2 +1/ n 2
Also lim un = lim = i.e., lim un ≠ 0
n→∞ n →∞ 3 + 1  n 3 n →∞

Hence the given series is oscillatory.

Example – 4
Show that the series
1 1 1 1 1 1 1
1+ − − + + − − +   is convergent.
2 2 32 4 2 5 2 6 2 7 2 8 2
>> Consider the absolute series,
1 1 1 1 1 1 1
1 + 2 + 2 + 2 + 2 + 2 + 2 + 2 +  
2 3 4 5 6 7 8
1
The general term of this series is given by un = 2
n
1
∑ 2 is a harmonic series ( p = 2 > 1) and it is convergent.
n
Hence we conclude that the given series is absolutely convergent.
But every absolutely convergent series is convergent.
Hence the given alternating series is convergent.
Remark : The problem can also be done by applying Leibnitz test.

Example – 5

82
Discuss the nature of the series
x2 x3 x4
x− + − +  
2 3 4
>> The general term of the series is given by
n −1
 −1 xn  −1 n x n +1
un = ∴ un +1 =
n n +1
We shall apply generalised D' Alembert' s test.
un +1  −1 n x n +1 n
Now lim = lim  n −1 n
n→∞ un n→∞ n + 1  −1 x
n
= lim −1 x = x
n→∞ n  1 + 1  n

Hence the series is absolutely convergent if | x | < 1, not convergent if | x | > 1 and the test
fails if | x | = 1.
Since absolute convergence implies convergence, the given series is convergent if |x|<1,
that is –1< x <1,not convergent when | x | > 1 and the test fails if | x | = 1.
If | x | = 1 then x = 1 or –1.
1 1 1
If x = 1, the series becomes 1 − + − +  
2 3 4
1 1
We have un = ∴ un +1 = Clearly un > un +1
n n +1
1
Also lim un = lim =0
n→∞ n→∞ n

Hence the series is convergent by Leibnitz test


If x = −1 the series becomes
1 1 1 F
G1 IJ
−1−
2

3
−   = − 1 +
2
+
H 3
+  
K
1 1
Here the series of positive terms ∑ = ∑ 1  2 is divergent.
n n
∴ the given series is divergent (to − ∞) when x = −1
Thus the given series is convergent if − 1 < x ≤ 1 divergent when x = −1

83
Exercises :
Find the nature of the following series :
1 1 1
 1 1 − + − +  
4 7 10
1 1 1 1
 2 Examine the series − + − +   for
7 12 17 22
(a) convergence for (b) absolute convergence
(c) conditional convergence

(3) Show that the seies


1 1 1 1 1 1 1
1+ − − + + − − +  
2 2 3 3 4 4 5 5 6 6 7 7 8 8
is convergent.
1 1 1
(4) Show the series 1 − x + x 2 − x 3 +  
4 7 10
is absolutely convergent if | x | < 1, conditionally convergent if x = 1 and divergent
if x = 1.

Answers :

1. Convergent
2. Convergent, conditionally convergent, not absolutely convergent.

84
V.T.U. Web-Based Education

Engineering Mathematics – I (MAT-11)

By Prof.M.G.Geetha
Dept. of Mathematics, PESIT
Bangalore

ANALYTICAL GEOMETRY IN 3
DIMENSIONS

Introduction
Any point in a plane can be located by means of two real numbers, called the
coordinates. These numbers could either be distances from two fixed straight lines or a
distance form a fixed point and an angle with reference to a straight line.

The former is called the Cartesian System of coordinates and the latter the polar system
of coordinates. If the two reference lines in the Cartesian system are perpendicular to
each other, the system is called the rectangular Cartesian coordinate system.

Any point in a plane is represented by P(x, y) in Cartesian system and P(r, θ ) in polar
system of coordinates.

In space, we need three real numbers to locate a point and the reference lines, called the
coordinate axes, which are three concurrent mutually perpendicular lines. The system of
coordinates is called the rectangular Cartesian coordinate system in three dimension.
F I G 1

In the above figure, the lines OX,OY and OZ are the three axes with O as the
origin of the coordinate system.

85
If P(x,y,z) is any point in this system, then ON = x, NM = y and MP = z where
M is the foot of the perpendicular from P to the XOY plane and N is the foot of
the perpendicular from M to the axis OX.

Distance formula :
Then from the right angled triangle OMP,

OP2 = OM2 + MP2


= ON2 + MN2 + MP2, because ONM is a right angled triangle.
= x2 + y2 + z2

Or, the distance OP = x2 + y2 + z 2

Distance between two given points :


Let P (x1, y1, z1) and Q (x2, y2, z2) be the two given points.
Shift the origin to P keeping the axes parallel to the original axes.
Then A≡ (x2- x1, y2- y1, z2- z1) with reference to the new axes.

∴Distance PQ = ( x 2 − x1 ) 2 + ( y 2 − y1 ) 2 + ( z 2 − z1 ) 2

Section Formula
If a point R divides the line segment joining the points P(x1, y1, z1) and Q(x2, y2, z2)

 px + qx1 py 2 + qy1 pz 2 + qz1 


in the ratio p : q , then R≡ 2 , ,
 p + q p + q p + q 

 x + x1 y 2 + y1 z 2 + z1 
If R is the middle point of PQ, then R≡ 2 , ,
 2 2 2 

Example:
Find the coordinates of the centroid of the triangle whose vertices are
A (x1, y1, z1), B (x2, y2, z2) and C (x3, y3, z3).

Answer : Fig. 1.2

86
If G is the centroid of the triangle ABC, then it divides AD, the median from A,
in the ratio 2:1.

 x 2 + x3 y 2 + y 3 z 2 + z 3 
Since D is the mid point of the side BC, D ≡  , ,
 2 2 2 
 x + x2 + x3 y1 + y 2 + y 3 z1 + z 2 + z 3 
∴G ≡  1 , , 
 3 3 3

Direction Cosines:
If a straight line makes angles α, β and γ with the coordinate axes, then cosα, cosβ and
cosγ are defined as the Direction Cosines (D.C.s) of the straight line. They are denoted
by l, m, n.

Numbers proportional to the D.C.s are called Direction Ratios (D.R.s), denoted by a. b.
c.

For instance,
1, 0, 0 are the D.C.s of the X-axis and s,0,0 are its D.R.s for any real number s.

Note
1. Two parallel lines have the same D.C.s
2. If A(x, y, z) is any point on the line OA that makes angles α, β and γ with the
three axes, then
x y z
3. = cos α , = cos β , = cos γ where OA = r
r r r
∴cos2 α + cos2 β + cos2 γ = 1, using the distance formula r =

x2 + y2 + z 2

87
4. If P is any point at a distance r from the origin, then P is given by P ≡ (r l, r m,
r n) where l, m, n are the D.C.s of the line OP.
5. l2 + m2 + n2 =1 for any line whose D.C.s are l, m, n
6. If a, b, c are D.R.s of a line, then a, b, c are proportional to the D.C.s l, m, n

a b c a2 + b2 + c2
i.e., = = = = a2 + b2 + c2
l m n l +m +n
2 2 2

∴The D.C.s of the line are


a b c
, ,
a2 + b2 + c2 a2 + b2 + c2 a2 + b2 + c2

D.C.s of the line joining two points: fig 1.3


Let A (x1, y1, z1) and B (x2, y2, z2) be the two points.
Shifting the origin to A (x1, y1, z1) with the axes parallel to the original axes,
We get B ≡ (x2 - x1, y2 - y1, z2 - z1)
If l, m, n are the D.C.s of the line AB with length r units, we have B ≡ (r l, r m, r
n) since A is the origin in the new system of coordinates.
l m n
∴ = =
x2 − x1 y 2 − y1 z 2 − z1
∴ The D.C.s of AB are proportional to x2 – x1, y2 - y1, z2 – z1.
or one set of D.R.s can be taken as x2 – x1, y2 - y1, z2 – z1.

Angle between two lines with known D.C.s :

Fig 1.4
Let l1, m1, n1 and l2, m2, n2 be the D.C.s of two given straight lines.
Draw OP and OQ parallel to the given lines passing through the origin.
Let OP = r1 and OQ = r2.
Then P ≡ (r1l1, r1m1, r1n1) and Q ≡ (r2l2, r2m2, r2n2)
∴ PQ2 = (r2l2 – r1l1)2 + (r2m2 – r1m1)2 + (r2n2 – r1n1)2
= r12 + r22 – 2r1r2 (l1l2 + m1m2 + n1n2), using the distance formula
Remember : l12 + m12 + n12 = 1 and l22 + m22 + n22 = 1 .

88
Also, using Cosine formula for triangle OPQ,

We have PQ2 = OP2 + OQ2 – 2OP.OQ cos θ, where θ = P O Q .

Equating the two expressions for PQ2, we get cos θ = l1l2 + m1m2 + n1n2
Note:
1. The expression for the angle between two lines in terms of their D.R.s (a 1, b1, c1 and
a2, b2, c2 )
a1 a 2 + b1b2 + c1 c 2
is cos θ =
a12 + b12 + c12 a 22 + b22 + c 22

2. The condition for perpendicularity of two lines is


l1l2 + m1m2 + n1n2 = 0 or a1a2 + b1b2 + c1c2 = 0.

Projection of a line joining two points on a straight line with known D.C.s :
Fig. 1.5
Let A ≡ (x1, y1, z1), B ≡ (x2, y2, z2) and let l, m, n be the D.C.s of the line MN.
MN is the projection of AB on MN.
If θ is the angle between AB and MN, then MN = AB cos θ.

l (( x2 − x1 ) + m( y 2 − y1 ) + n( z 2 − z1 )
∴MN = ( x 2 − x1 ) 2 + ( y 2 − y1 ) 2 + ( z 2 − z1 ) 2
( x 2 − x1 ) 2 + ( y 2 − y1 ) 2 + ( z 2 − z1 ) 2

∴MN = l(x2 – x1) + m(y2 - y1) +n(z2 - z1).


Remember: The D.C.s of a line joining the points (x1, y1, z1) and (x2, y2, z2) are

x 2 − x1
etc.
( x 2 − x1 ) 2 + ( y 2 − y1 ) 2 + ( z 2 − z1 ) 2

Exercise:
1. The distance between the points (0,0,3) and (-4, 0, 0) is
a)* 5 unit b) √5 units c) 25 units d) 4 units

89
2. If A ≡ (-1, 0, 4) and B ≡ (1, -6, 2), then the mid-point of AB is
a) (1, -3, 1) b)* (0, -3, 3) c) (1, -3, -1) d) (0, 3, -1)
3. The line which is perpendicular to the line with D.R.s 1,2,3 has D.C.s
proportional to
a) 3, 2, 1 b) *0, 3, -2 c) 1, 3, -2 d) 1, -2, 3.
4. The vertices (5, -1, 1), (7, -4, 7), (1, -6, 10) and (-1, -3, 4) form a
a) rectangle b) Square c)* Rhombus d) None of the others
5. The point A (5,8,-1) is joined to the mid-point M of the line joining P(1,2,-3) and
Q(3,4,5). The angle between these two lines is
a)* 90o b) 30o c) 60o d) 45o

Planes:
A Plane is a surface such that the line joining any two points on the surface lies entirely
on the surface.

Theorem:
A linear equation ax + by + cz +d = 0 in x, y and z represents a plane.

Proof :
If A(x1, y1, z1) and B(x2, y2, z2) are two points on the surface represented by the above

 px + qx1 py 2 + qy1 pz 2 + qz1 


equation, then the point C≡ 2 , , is a point lying
 p + q p + q p + q 
on the line AB dividing it in the ratio p : q.
Since A and B are points lying on the surface ax + by + cz +d = 0 -------------------------
(1)
we have ax1 + by1 + cz1 + d = 0 and ax2 + by2 + cz2 + d = 0
∴ a ( px 2 + qx1 ) + b ( py 2 + qy1 ) + c ( pz 2 + qz1 ) + d ( p + q) = 0

90
 px + qx1   py 2 + qy1   pz 2 + qz1 
Or a 2  + b  + c  + d = 0
 p + q   p + q   p + q 
which implies that the point C lies on the surface given by equation (1).
Since C is any arbitrary point lying on the line AB, the entire line AB lies on the surface
(1).
∴By the definition of a plane, equation (1) represents a plane.

Normal Form of equation to a plane :


Fig. 1.6
Let OL be drawn perpendicular (normal) to the plane from the origin.
Let its length be p and the D.C.s be l, m, n.
If P (x, y, z) is any point on the plane, then OL is the projection of OP on the normal
OL.
By the projection formula, we have p = lx + my + nz.
or lx + my + nz = p is the normal form of equation to a plane.

Note: Comparing the general and the normal forms of equations of a plane, we get
a b c d
= = = from which we observe the following:
l m n −p
1. a, b and c, the coefficients of x, y and z in the equation of a plane are proportional
to the D.C.s of the normal to the plane.
In other words, the coefficients of x, y, z are a set of D.R.s of the normal.

a2 + b2 + c2
2. Since each ratio is equal to = a2 + b2 + c2 ,
l +m +n
2 2 2

−d
we get p=
a + b2 + c2
2

d
∴ Length of the perpendicular from the origin to the plane is
a2 + b2 + c2

3. To find the length of the perpendicular from any point P ≡ (x1, y1, z1) to the plane
ax + by + cz + d = 0, shift the origin to the point P keeping the directions of the axes

91
unchanged.
Then any general point (X, Y, Z) in the new system of coordinates changes to
(x – x1, y – y1, z –z1)
∴The equation of the plane in the new system of coordinates is
a (x + x1) + b (y + y1) + c (z + z1) + d = 0
or ax + by + cz + (ax1 + by1 + cz1+ d) = 0
so that the perpendicular distance of the plane from P, the origin in the new system, is

ax1 + by1 + cz1 + d


a2 + b2 + c2

Three Point form of Equation:


Given the three points (xi, yi, zi); i =1, 2, 3.
If the plane given by ax + by + cz +_d = 0 passes through these three points, then they
satisfy the equation of the plane.
Eliminating ‘d’, we get
a(x - x1) + b(y – y1) + c(z – z1) = 0
a(x1 – x2) + b(y1 – y2) + c( z1 – z2) = 0
and a(x2 – x3) + b(y2 – y3) + c(z2 – z3) = 0
x − x1 y − y1 z −z1
This system of three equations has a non-trivial solution if x1 − x 2 y1 − y 2 z1 − z 2 =
x2 − x 3 y2 − y 3 z2 −z 3

0.
Note:
The three-point form of equation to a plane gives the condition for coplanarity of four
points
(xi, yi, zi) ; i = 1, 2, 3, 4 as

x1 − x 2 y1 − y 2 z1 − z 2
x2 − x 3 y2 − y 3 z2 − z 3 = 0.
x3 − x 4 y3 − y 4 z3 − z 4

92
Intercept form of equation:
If a plane makes intercepts A, B, and C with the coordinate axes, then the points
(A,0,0), (0,B,0) and (0,0,C) lie on the plane.
If ax + by + cz + d = 0 is the equation of the plane, then
−d −d −d
a= , b= & c=
A B C
x y z
∴The equation of the plane reduces to + + = 1.
A B C
This is known as the intercept form of equation to the plane.

Note:
1. The angle between the two planes aix + biy + ciz + di = 0 ; i = 1, 2 ; is equal to the
angle between their normals and hence is

 a1 a 2 + b1b2 + c1c 2 
cos −1

 1 + + + + 2 
2 2 2 2 2 2
a b1
c1
a 2
b 2
c 
2. If the two planes are perpendicular, then a1a2 + b1b2 + c1c2 = 0.
3. Equation of a plane passing through the line of intersection of two planes
aix + biy + ciz + di = 0, i =1,2 ; is
(a1x + b1y + c1z +d1) + k(a2x + b2y + c2z +d2) = 0 where k is a constant.

Exercise:
1. The planes 2x + 3y - z = 5 and 6x +9y – 3z -7 = 0 are
a) Perpendicular b)* Parallel c) coinciding d) inclined at an angle of 45o
2. The distance of the point (1,1,1) from the plane 2x – y +2z =3 is
a)* 0 b) 1/3 c) 1 d) 2
3. The D.C.s of the normal to the XY-plane are
a) 1,1,1 b) 0,1,0 c)* 0,0,1 d) 1,0,0
4. Which of the following points lie on the same side of the plane 3x – y + 2x – 8 = 0

93
a) (3,0,0) b) (3,0,2) c) (2,0,1) d)* (2,0,0)
5. One of the following planes is perpendicular to the plane 2x – 3y +5z = 1
a) x – y – z = 4 b) 5y - 3z = 1 c) x + y – z = 14 d) 3x – 2y + 5 = 0

True of False
1. The plane 4x – 3y +5 = 12 passes through the intersection of the planes
2x - 5y + z = 6 and x + y + 2z = 3. (T)
2. The point (1, -5,2) lies on the plane 3x + y + z = 1. (F)
3. The Plane 2x + 3y – z + 6 = 0 and 10x - 15y - 5z + 24 = 0 are parallel (F)

Straight Lines:
A curve in space is defined as the set of points common to two intersecting surfaces
and it is represented by two equations - equations of the intersecting surfaces.
A straight line is the curve of intersection of two planes.

Two plane form of equation to a straight line:


Two planes interest in a straight line and therefore the equations of the intersecting
planes together represent a straight line.
If a1x + b1y + c1z + d1 = 0 and a2x + b2y + c2z + d2 = 0 represent two intersecting
planes, then the equations of the straight line of intersection are
a1x + b1y + c1z + d1 = 0 = a2x + b2y + c2z + d2
This is called two-plane form of equations .

Symmetric form of equations:


If P(x, y, z) is any point on a straight line that passes through the point (x 1, y1, z1),
then D.C.s of the straight line l, m, n are proportional to x - x1, y - y1, z - z1.
x − x1 y − y1 z − z1
∴The equations of the straight line are = = ,
l m n
called the symmetric form of equations.

Note:

94
1. Any point lying on the straight line whose equations are

x − x1 y − y1 z − z1
= = ,
l m n
is given by (lt + x1, mt + y1, nt + z1) where t is a parameter .
2. The symmetric form of equations to a straight line can be converted to its two-
plane form of equations and vice versa.
3. When the two-plane form of equations to a straight line are given, the D.C.s of the
straight line are proportional to b1c2 – b2c1, c1a2 – c2a1 and a1b2 – a2b1,
since the straight line is perpendicular to the normal to the two intersecting planes.

Coplanarity of two straight lines:


Consider two straight lines
x − x1 y − y1 z − z1 x − x2 y − y 2 z − z 2
= = and = = .
l1 m1 n1 l2 m2 n2
If these lines are coplanar, let the equation of the plane containing them be
ax + by + cz + d = 0.
Then the two points (x1, y1, z1) and (x2, y2, z2) lie on this plane. Also the normal to the
plane is perpendicular to the two lines.
∴we have the following conditions :
ax1 + by1 + cz1 + d = 0 --------------------------- (1)
ax2 + by2 + cz2 + d = 0 --------------------------- (2)
al1 + bm1 + cn1 = 0 --------------------------- (3)
al2 + bm2 + cn1 = 0 --------------------------- (4)
Eliminating d from equations (1), (2) and ax + by + cz + d = 0, we get
a (x – x1) + b ( y – y1) + c (z –z1) = 0 and
a (x1 – x2) + b ( y1 – y2) + c (z1 –z2) = 0

95
Eliminating a, b, c from this equation, (3) and (4) , we get the condition for
coplanarity of straight lines in the form

x1 − x 2 y1 − y 2 z1 − z 2
l1 m1 n1 = 0
l2 m2 n2

x − x1 y − y1 z −z1
and the equation of the plane containing them is given by l1 m1 n1
l2 m2 n2

= 0.

Note:
If two lines are coplanar, then the lines are either parallel or intersecting. When they
intersect, the point of intersection can be found by solving the equations of the two lines.

Skew Lines:
Two lines are said to be skew when they are non-coplanar. This is so, when they are
neither parallel nor intersecting.

Shortest distance between skew lines:


When two lines are skew, there exists a line perpendicular to both called the line of
shortest distance. The length of the common perpendicular intercepted between the
skew lines is the shortest distance between them.
Fig. 1.7
x − x1 y − y1 z − z1
Let the two skew lines be given by = = -------------- (1)
l1 m1 n1

x − x2 y − y 2 z − z 2
and = = -------------(2)
l2 m2 n2

with P ≡ (x1, y1, z1) and Q ≡ (x2, y2, z2) .


AB, the line of shortest distance is perpendicular to both the lines.

96
Then the D.C.s λ, µ, ν of the line of shortest distance are proportional to
m 1n2 – m2n1, n1l2 – n2l1, l1m2 – l2m1
We observe that the length AB is the projection of the line PQ on the line of shortest
distance.
∴ By the projection formula,
AB = (x2 – x1)λ + (y2 – y1)µ + (z2 – z1)ν

( x 2 − x1 )(m1 n 2 − m 2 n1 ) + ( y 2 − y1 )(n1l1 − n 2 l1 ) + ( z 2 − z1 )(l1 m 2 − l 2 m1 )


=
(m1 n 2 − m 2 n1 ) 2 + ( n1l 2 − n 2 l1 ) 2 + (l1 m 2 − l 2 m1 ) 2

This gives the shortest distance between the two skew lines.

Equations of the line of shortest distance:


The shortest distance line is coplanar with line (1) since it intersects line (1) at the point
A. Point B is the point of intersection of the line of shortest distance and line (2).
∴ We have

x − x1 y − y1 z −z1 x −x 2 y −y2 z −z 2
λ µ υ =0 and λ µ υ =0
l1 m1 n1 l2 m2 n2

These two equations together represent the line of shortest distance between the skew
lines given by (1) and (2).

Exercise:
x − x1 y − y1 z − z1
1. The straight line = = is
a b c
a)* Perpendicular b) Parallel to c) lies entirely on d) at an angle of 45o
with
the plane ax + by + cz + d = 0 .
x y z x y
2. The point of intersection of the lines = = and = = z is
7 3 −2 3 −4
a) (-5, -13, -37) b) (4, 7, -3) c)* (0, 0, 0) d) (1, -1, 2)

97
x−2 y −5 z +3 x − 4 6 − y 2z + 3
3. The lines = = and = = are
3 −4 2 3 4 4
a) Perpendicular b)* Parallel c) coincident d) Skew
4. The image of the point (1, 1, 1) in the plane x + y + z = 0 is
1 1 1
a) (0, 0, 0) b) (1, -1, -1) c)* (-1, -1, -1) d) ( , , )
3 3 3

True or False
2− y z−3 x−2 y −3 4− z
1. The lines x −1= = and = = are perpendicular
2 2 4 5 2
.
x−4 y−6
2. The point (1, 2, 3) lies on the line = =2− z.
3 4
x −1 z
3. The line = y−2= and the z – axis are skew lines.
0 0
x −1 y − 2
4. The equations = =z−4 and 3x –2y +1 = 0 = y – 3z +10
2 3
represent the same straight line.
x − 2 y +1 z −1
5. The plane 2x + 3y + 3z –1 = 0 contains the line = = .
3 2 −4
6. The shortest distance between the lines
x −1 y − 2 z − 4 x −1 y − 2 z − 4
= = and = = is 0.
2 3 −4 −3 4 5
7. The straight lines which do not intersect are called skew lines.

Right Circular cone and Right Circular Cylinder:


The solid generated when a straight line rotates about a fixed line is either a right
circular cone or a right circular cylinder. The rotating line is a generator of the solid.
The fixed line is called the axis.

98
In the case of a cone, the generators are inclined at an angle α with the axis while in a
cylinder, the generators are parallel to the axis.

Note:
The angle α is called the semi-vertical angle of the cone and the point of intersection
of the generators with the axis is called the vertex of the cone.

Equation of a right circular Cone:


Fig. 1.8
x − x1 y − y1 z − z1
Let the axis of the cone be given by = = with its vertex as V≡ (x1, y1, z1).
l m n
If P (x, y, z) be any point on the surface of the Cone, then the D.C.s of the generator
passing through V are proportional to x – x1, y – y1, z – z1.
If α is the semi-vertical angle of the cone, then

( x − x1 )l + ( y − y1 ) m + ( z − z1 )n
cos α =
( x − x1 ) 2 + ( y − y1 ) 2 + ( z − z1 ) 2 l 2 + m 2 + n 2
or [ ( x − x ) l + ( y − y ) m + ( z − z ) n]
1 1 1
2

[
cos 2 α = (l 2 + m 2 + n 2 ) ( x − x1 ) + ( z − z1 ) 2
2
]
is the equation of a right circular cone with its vertex at (x1, y1, z1), semi-vertical angle
as α and the D.C.s of its axis proportional to l, m, n.

Equation of a right circular cylinder:


Fig/ 1.8
x − x1 y − y1 z − z1
= = be the axis of the right circular cylinder with A ≡ (x1, y1,
l m n
z1) and r as its radius.
If P ≡ (x, y, z) is any point on the surface of the cylinder, then AM is the projection of
AP on the axis with M as the foot of the perpendicular from P to the axis.

or AM = AP cos θ, where θ = M AP

99
l ( x − x1 ) + m( y − y 1 ) + n( z − z1 )
∴ AM = ( x − x1 ) 2 + ( y − y1 ) 2 + ( z − z1 ) 2 .
∑ (x − x ) l + m + n
1
2 2 2 2

Also AM 2 = AP2 – MP2 with MP = r

∴ ( x − x1 ) + ( y − y 1 ) + ( z − z1 ) = r
2 2 2 2
+
[ l ( x − x ) + m( y − y ) + n ( z − z )]
1 1 1
2

l 2 + m2 + n2
is the equation of the right circular cylinder.

Exercise
Fill in the blanks :
1. The semi-vertical angle of the right circular cone passing through the origin, vertex at

x y −1 z +1
(1,2,1) with its axis parallel to the line = = is
2 2 −1
____________. π /3

y−7 z +7
2. The vertex of the right circular cone whose axis is the line x= = with the line
−3 2
x +1 y − 3
= =z+2 as one of its generators is _______________ (2,1,-3)
−3 2
3. The radius of the right circular cylinder passing through the origin with its axis lying

x+9 y−4 z −5
along the line = = is _______ units.
10 −2 −3
3.

Questions
1. Find the distance of the point (1, 2, -3) from the middle point of the line segment
joining (-3, 4, -2) and (1, 2, 6)
The mid-point of the given line segment is (-1, 3, 2).
∴ The distance of (1, 2, -3) from this point is 4 + 1 + 25 = 30 units

2. Show that the points A (2, 3, 4), B (3, 4, 2) and C(-1, 0, 10) are collinear.

100
D.R.s of the line AB are 1, 1, -2 and those of the line BC are 4, 4, -8 or 1, 1, -2
∴ The lines AB and BC have proportional D.R.s. The lines have a common point B.
Hence the points A, B and C are collinear.

3. Find the centroid of the triangle ABC where


A ≡ (-1, 2, 1), B ≡ (-3, 4, -4) and C ≡ (-1, 0, 3).

 −1+ 3 +1 2 + 4 + 0 1− 4 + 3 
The centroid G is given by  , ,  or (1, 2, 0).
 3 3 3 
4. Find the angles made by a line with the coordinate axes if it is equally inclined with
the axes.
We know that cos2 α + cos2 β + cos2 γ = 1
1
∴ angle = α = cos
−1
But α = β = γ is given .
3
5. Find the angles of the triangle ABC formed by the vertices
A (3, 0, -1), B(-1, 4, 1) and C(3, 4, 2) .
D.R.s of AB are –4, 4, 2 and D.R.s. of AC are 0, 4, 3.
0 + 16 + 6 11
∴ cos A = =
36 25 15
D.R.s of BA are 4, -4, -2 and D.R.s of BC are 4, 0, 1
7
∴ cos B =
3 17
D.R.s of CA are 0, -4, -3 and D.R.s of CB are –4, 0, -1
3
∴ cos C =
5 17

11 7 3
∴The angles are cos −1 = , cos −1 = −1
and cos =
15 3 17 5 17

6. Find the angle between any two diagonals of a cube.

101
Fig. 1.10
Let the cube be as shown in the figure with one of its vertices O as the origin of the
coordinate system and the axes along the three sides OA, OB, OC of the cube
respectively.
If d is the length of each side of the cube, then its vertices are given by
O ≡ (0, 0, 0), O’ ≡ (d, d, d),
A ≡ (d, 0, 0), A’ ≡ (0, d, d),
B ≡ (0, d, 0), B’ ≡ (d, 0, d),
C ≡ (0, 0, d), C’ ≡ (d, d, 0),
We can see that the four diagonals of the cube are OO’, AA’, BB’, and CC’.
D.R.s of OO’ and AA’ are d, d, d and –d, d, d respectively
d2 + d2 + d2 1
∴ Angle between these two diagonals is given by cos −1 = = cos −1
3d 2
3d 2 3

7. Find the angle between a diagonal of a cube and


(i) an intersecting diagonal of a face (ii) a non-intersecting diagonal of a face.

(i) The diagonal AA’ of the cube and the diagonal O’A of a face of the cube
intersect.
D.R.s of these two lines are respectively –d, d, d, and 0, d, d .
2d 2 2
∴Angle between them is given by cos −1 = = cos −1
3d 2 2d 2 3

(ii) The diagonal AA’ of the cube and the diagonal B’C’ of a face of the cube do not
intersect. Their D.C.s are respectively proportional to –d, d, d and 0, d, -d. .

−1 0+d2 −d2 π
∴Angle between them is given by cos = =
3d 2
2d 2 2

8. Given a cube as in Fig. 1.10, the diagonals passing through O, of the faces meeting at
O, make angles α, β, γ with the diagonal OO’ of the cube, prove that
cos2 α + cos2 β + cos2 γ = 2.

102
The diagonals of the faces meeting at O and passing through O are OA’, OB’, OC’.
whose D.R.s are 0, d, d; d, 0, d and d, d, 0.
If these lines are inclined at angles α, β, γ with OO’, whose D.R.s are d, d, d ;

2d 2 2 2
then the angles are given by cos α = = or cos α =
2

6d 2
6 3
2 2
Also cos β = and cos γ = .
2 2
Hence the required result.
3 3

9. Find the angle between the two lines whose D.C.s are given by the relations
l+m+n=0 and 2lm + 2nl – mn = 0.
2
m m
Eliminating l from the given relations, we get 2  + 2 + 1 = 0
n n

m   m 
⇒  + 2  +  2 + 1 = 0.
n   n 
⇒ m + 2n = 0 or

2m +n = 0
These with l + m + n = 0 give us two sets of l,m,n proportional to –1, -1, 2 and 1,
-2, 1.
−1 −1+ 2 + 2 π
The angle between these two lines is cos = .
6 3

10. Show that the straight liens whose D.C.s are given by ul + vm + wn = 0 and
pl2+qm2+rn2 = 0
are (i) perpendicular if u2(q+r) + v2(r+p) + w2(p+q) = 0
u 2 v 2 w2
and (ii) parallel if + + =0
p q r

Eliminating l from the two given relations,

103
m2 m
We get ( pv + qu ) 2 + 2 pvw + ( pw 2 + ru 2 ) = 0
2 2
-----------------( 1)
n n
m m1 m2
(i) Equation (1) is quadratic in and has two roots, say, and
n n1 n2

m1 m2 pw 2 + ru 2
Then the product of the 2 roots is given by =
n1 n2 qu 2 + pv 2
m1 m 2 n1 n 2
or =
pw 2 + ru 2 qu 2 + pv 2
Similarly, by eliminating m and n from the given relations and then getting the
product of the roots and simplifying, we get
n1 n 2 ll l1l 2 m1 m 2
= 212 2 and =
qu + pv
2 2
rv + qw rv + qw
2 2
pw 2 + ru 2

m1 m 2 nn ll
∴ = 2 1 2 2 = 2 1 2 2 ---------------(2)
pw + ru
2 2
qu + pv rv + qw
where li, mi, ni ; i= 1, 2 are the two straight lines given by the relations.
When these lines are perpendicular, we have l1l2 + m1m2 + n1n2 = 0
⇒ pw2 + ru2 + qu2 + pv2 + rv2 + qw2 = 0 from (2)
⇒ u2 (q+r) + v2(r+p) + w2(p+q) = 0
(ii) When the two lines are parallel, they have the same D.C.s.
This is true when equation (1) has two equal roots.
⇒ 4p2v2w2 - 4(pv2 + qu2) (pw2 + ru2 ) = 0
⇒ qru2 + rpv2 + pqw2 = 0

u 2 v 2 w2
⇒ + + =0
p q r

Planes:
11. Find the equation of the plane passing through the points (1, 2, 3), (2, 0, -1) and
(-1, 4, 5).
Also find the distance of the point (0, 2, -4) from the plane.
What is the angle made by this plane with 2x-y+z = 6 ?

104
x −1 y − 2 z − 3
The plane through the three given points is 1 −2 − 4 = 0 or 2x+3y-z
−2 2 2

= 5.
0+6+4−5 5
Perpendicular distance of (0,2,-4) from this plane is = .
14 14
The angle between this plane and 2x-y+z = 6 is given by cosθ = 0.
or the two planes are perpendicular.

12. Show that the points (3, 4, 1), (-1, -2, 5), (1, 7, 1) and (1, 10, 0) are coplanar.
Find the equation of the plane containing them and also the intercept made by it on the
x-axis.

The plane containing (3,4,1), (-1, -2, 5) and (1,7,1) is


x − 3 y − 4 z −1
2 3 − 2 = 0 or 3x +2y + 6z - 23= 0.
2 9 −4

Since the point (1, 10, 0) lies on this plane, the four given points are coplanar.
x y z
+ + =1
The equation of the plane in intercept form is 23 23 23
3 2 6
23
∴The x-intercept by the plane is units.
3

13. Find the equation of the plane bisecting perpendicularly the join of the points (2,1,3)
and (4, 3, -1).

Let the equation of the required plane be ax+by+cz + d = 0


Then the mid-point (3,2,1) lies on it. Also a, b, c, are proportional to 1, 1, -2.
∴The equation is x + y – 2z – 3 = 0.

105
14. Find the equation to the plane perpendicular to x + y – 4z = 0 and through the
intersection of the planes x +3y –z = 4 and 2x + 2y +2z = 1.

The equation of the plane through the intersection of the given planes is
x + 3y – z – 4 + k(2x + 2y + 2z – 1) = 0
or (1 + 2k) x + (3 + 2k) y + (-1 + 2k) z + (-4 – k ) =0
But this is given to be perpendicular to x + y – 4z = 0
∴ 1 + 2k + 3 + 2k + 4 –8k =0 ⇒ k = 2.
∴ The equation of the required plane is 5x + 7y + 3z =6.

15. The plane 4x + 7y + 4z +81 = 0 is rotated through a right angle about its line of
intersection with the plane 5x + 2y + 5z = 25. Find the equation of the plane in its
new position.

The plane in its new position passes through the line of intersection of the two given
planes.
Fig. 1.11
∴Its equation is given by 4x + 7y + 4z +81 + k (5x + 2y +5z – 25) = 0
or (4 + 5k) x + (7 + 2k) y + (4 + 5k) z + (81 –25k) = 0
But this is perpendicular to the plane 4x + 7y +4z +81 =0
3
∴We have (4 + 5k) 4 + (7 + 2k) 7 + (4 + 5k) 4 = 0. ∴k = − .
2
∴The equation of the required plane is 7x-8y+7z-237 =0.

Straight lines:
16. Find the equations of the straight line of intersection of the planes 2x – y + z = 5
and
x +3y – 2z +1=0

The line of intersection is perpendicular to both the planes.

106
If l, m, n are its D.C.s, we have 2l – m + n = 0 and l + 3m – 2n = 0 .
∴l : m : n = -1 : 5 : 7
Choosing z = 0 in the equations of the planes and solving the resulting equations,
we get (2, -1, 0) as a point lying on both the planes.
x − 2 y +1 z
∴ Equations of the straight line are = =
−1 5 7

17. Find the image of the point P(1, 3, 4) in the plane 2x – y +z + 3 = 0 .


Fig. 1.12
If P’ is the image of the point P in the given plane, then the line PP’ is perpendicular
to the plane and the plane bisects the line at the point, say, M .
x−2 y −3 z −4
∴ the equations of PP’ are = = = k (say)
2 −1 1
M is given by M ≡ (2k+1, -k+3, k+4) and lies on the given plane. ∴k = 1.
∴M ≡ (-1, 4, 3)
∴ P’≡ (-2, 5, 2)

y z −3
18. Find the image of the point (1, -1, 2) in the line x −1= = .
3 −2

Let P ≡ (1, -1, 2) and P’ be its image.


Then PP’ is perpendicular to the given line and its mid-point, say, M lies on the
given line.
∴M can be written as (k + 1, 3k, -2k + 3) .
∴D.R.s if PP’ are k + 1 –1, 3k+1, -2k +3 –2 or k, 3k + 1, -2k+1.
y z −3
Since PP’ is perpendicular to the line x −1= = ,
3 −2
1
we have k + 3 (3k + 1) –2(-2k + 1) = 0 ⇒ k = −
14

 13 − 3 44 
∴ M ≡  , , .
 14 14 14 

107
 6 4 30 
∴ P' ≡  , , .
7 7 7 

x −1 y − 2 z − 3
19. Find the image of the line = = in the plane x + y + z + 3 =0 .
2 3 4
The given line is not parallel to the given plane.
∴It must intersect the plane.
Any point on the given line is (2k + 1, 3k +2, 4k + 3). If this lies on the plane, then k
= -1.
∴(-1, -1, -1) is the point of intersection of the given line and the given plane and
hence is a point lying on the image of the line in the plane.
(1,2, 3) is a point on the given line. Its image in the given plane is (-5, -4, -3) and is
another point on the image of the line.
∴D.R.s of the image line are 4, 3, 2 .
x+5 y+4 z+3
∴Its equations are = = .
4 3 2

20. Find the equation of the plane through the points (1, 0, -1) and (3, 2, 2,) and parallel
to the line
1− y z − 2
x −1= = .
2 3

The equation of the required plane is of the form a(x-1) + by + c (z+1) =0 --------
(1)
This plane also passes through the point (3, 2, 2).
∴We have 2a + 2b + 3c = 0 ----------- --- (2)
y −1 z − 2
The plane is parallel to x −1= = .
−2 3
∴ The normal to the plane with D.R.s a, b, c is perpendicular to the line.
∴ a – 2b + 3c = 0 ---------------------(3)
Evaluating a, b, c from the three equations, we get

108
x −1 y z +1
2 2 3 = 0 or 4x-y-2z-6 = 0 is the equation of the plane.
1 −2 3

x z
21. Find the angle between the lines =y= and 4x – y + z –5 =0 = 14x –
2 2
3y + 4z – 20.

D.R.s of the first line are 2, 1, 2 .


D.R.s of the second line are got by solving the equations 4a – b + c = 0 &
14a – 3b + 4c=0
and they are 1, 2, -2 .
−1 2+2−4 π
∴ Angle between the lines is cos =
9 2

x −1 2z + 3
22. Find the angle that the line = y−2 = makes with the plane x – y
2 4
+ z = 0.

π
The required angle is − cos θ , where θ is the angle between the line
2
and the normal to the plane.
2 −1+ 2 1
Here θ = cos
−1
= cos −1
3 3 3
−1 1
∴ Required angle is sin .
3

23. Find the distance of the point (1, -2, 3) from the plane x – y + z =13 measured
x y z
parallel to the line = = .
2 3 −6
Fig. 1.13

109
Let P ≡ (1, -2, 3). The equations of the line through P parallel to the given line are

x −1 y + 2 z − 3
= = . Let this line intersect the given plane at M.
2 3 −6
Then M ≡ (2k + 1, 3k - 2, -6k + 3).
Since M lies on the plane, we have 2k + 1 –3k + 2 – 6k + 3 = 13 ⇒ k = -1.
∴ M ≡ (-1, -5, 9).
∴ The required distance PM = 7 units .

24. Find the distance of the point (-1, -5, -10) from the point of intersection of the line

x − 2 y +1 z − 2
= = and the plane x - y + z =5.
3 4 12

Any point on the given line is (3k + 2, 4k –1, 12k+2).


If this lies on the given plane, then
3k + 2 – 4k +1 +12k +2 = 5 ⇒ k =0.
∴(2, -1, 2) is the point of intersection of the given line with the given plane.

∴ The required distance is (2 + 1) 2 + (5 − 1) 2 + (2 + 10) 2 = 13 units

25. Find the line through (3, -4, 1), parallel to the plane 2x + y –z +8 = 0 and

x − 3 y +1
intersecting the line = = z − 2.
2 −3

x − 3 y + 4 z −1
The equations of the required line are = = , where l, m, n are
l m n
its D.C.s.
This line is parallel to 2x + y – z +8 = 0.
∴We have 2l + m – n = 0 -----------------(1)
x − 3 y +1
The line intersects = =z−2 .
2 −3

110
∴They are coplanar.

l m n
∴ 2 − 3 1 = 0 ⇒ 3l + m − 3n = 0 -----------------------(2)
0 − 3 −1

Solving equations (1) and (2) , we get l : m : n are 2 :-3 :1 .


x − 3 y + 4 z −1
∴ Equations of the required line are = = .
2 −3 1

26. Find the equations of the line through (2, 1, 3) and perpendicular to the line

y+6 z−2
x−5= = .
2 3
Also find the length of the perpendicular from (2, 1, 3).

If M is the foot of the perpendicular from (2, 1, 3) to the given line, M ≡ (k+5, 2k-
6,3k+2).
∴ D.R.s of PM are k + 3, 2k – 7, 3k –1 with P ≡ (2, 1, 3) .
But PM is perpendicular to the given line.
∴ k + 3 + 2 (2k – 7) + 3 ( 3k –1) = 0 ⇒ k =1

∴ M ≡ (6, -4, 5) ∴ Perpendicular distance = 3 3


x − 2 y −1 z − 3
Equations of the perpendicular line are = = .
4 −5 2
27. A line with D.C.s proportional to 7, 4, -1 is drawn to intersect the lines

x −1 y − 7 x +1 y − 3 z − 5
= = z + 2 and = = . Find the points of intersection and
3 −1 −3 2 4
also the length intercepted.

Let P and Q be the points of intersection of the line with D.R.s 7, 4, -1 with the two
given lines.
Then P ≡ (3k1 + 1, k1 + 7, k1- 2)
and Q ≡ (-3k2 – 3, 2k2 + 3, 4k2 + 5) for some k1 & k2

111
∴ D.R.s of PQ are 3k1 + 3k2 +4, -k1 –2k2 + 4, k1 –4k2 – 7
They are also given by 7, 4, -1.
∴These two sets of D.R.s are proportional .
We get the equations 19k1 + 26k2 = 12 and k1 – 6k2 = 8,
which when solved, yield k1 =2, k2 = -1 .
∴Points of intersection are P ≡ (7, 5, 0) and Q ≡ (0, 1, 1) .

∴Length intercepted 66 units

x−4 y −2 z −3
28. Find the shortest distance between the lines = = and
2 −3 4
5x +3y +7z +6 = 0 = 6x +7y -5z +14. Also find the equations of the shortest
distance line.

Line 1 has D.R.s 2, -3, 4 and (4, 2, 3) lying on it.


Line 2 has D.R.s –64, 67, 17 and (0, -2, 0) lying on it.
∴The shortest distance line has D.R.s 11, 10, 2.
But the shortest distance is the projection of the join of (4, 2, 3) and (0, -2, 0) on the
line of shortest distance.
11 10 2
∴ Shortest distance length (4 − 0) + (2 + 2) + (3 − 0) = 6 units
15 15 15
Equations of the shortest distance line are given by

x−4 y−4 z −3 x y+2 z


2 −3 4 = 0 = − 64 67 17
11 10 2 11 10 2

or 46x – 40y – 53z + 55 = 0 = 4x – 35y + 153z – 70.

x −1 y − 2 z − 3
29. Find the shortest distance between the line = = and
2 −4 7

x − 2 y −1 z − 2
= = .
2 5 −6

112
What do you conclude here?

The D.C.s of the shortest distance line are proportional to 11, 9, 2 .


11( 2 − 1) + 9(1 − 2 ) + (2 − 3)
Shortest distance = =0.
206
∴The lines are coplanar. We can see that they are not parallel. Hence they intersect.

30. Find the plane through the line 5x + 3y + 7z +6 = 0 = 6x + 7y – 5z +2 and parallel

x −1 y −1 z − 3
to the line = = . Hence find the length of the shortest distance
2 −3 4
between the given lines.

Any plane through the given line is (5 + 6k) x + (3 + 7k) y + (7 – 5k) z + (6 +2k) =
0.
x −1 y − 2 z − 3
This is parallel to the line = = .
2 −3 4
∴(5 + 6k) 2 + (3+7k) (-3) + (7-5k) 4 = 0 ⇒ k = 1.
∴ The equation of the required plane is 11x +10y + 2z + 8 = 0.
Now the length of the perpendicular from the point (1, 2, 3) on to this plane gives
the shortest distance between the two given lines.
45
∴It is = 3 units.
15

Cones and Cylinders :


31. Find the equation of the right circular cone with vertex at (1, 2, -3), 30o as its semi-

x −1 y −1 z + 3
vertical angle and its axis along the line = = .
2 3 1

cos 2 30 o =
3
=
[ 2( x − 1) + 3( y − 1) + ( z + 3) ] 2

[
4 14 ( x − 1) 2 + ( y − 2) 2 + ( z + 3) 2 ]
or [
21 ( x − 1) + ( y − 2 ) + ( z + 3)
2 2 2
] = 2[ 2 x + 3 y + z − 5] 2
is the equation.

113
32. Show that the equation of the right circular cone with semi-vertical angle as α and its
axis along the z –axis, is given by x2 + y2 = (z-3)2 tan2 α given that the vertex is at
(0, 0, 3).

cos 2
α=
[ 0( x − 0) + 0( y − 0) + ( z − 3)]
2

x 2 + y 2 + ( z − 3) 2

[ ]
⇒ x 2 + y 2 + ( z − 3) cos 2 α = ( z − 3) 2
2

⇒ x 2 + y 2 = ( z − 3) 2 tan 2 α
33. Given that the vertex of a right circular cone passing through the origin is at the point

x −1 y − 2 z −1
(1,2,1) whose axis is along the line = = . Find its equation.
2 1 −2

cos 2 θ =
[ 2(0 − 1) + 1(0 − 2) − 2(0 − 1)] 2 =
2
9 x6 27
If (x, y, z) is any general point on the surface of the cone, then

2 [ 2( x − 1) + 1( y − 2) − 2( z − 1)]
2

=
[
27 9 ( x − 1) 2 + ( y − 2) 2 + ( z − 1) 2 ]
or [
2 ( x − 1) + ( y − 2 ) + ( z + 1)
2 2 2
] = 3[ 2 x + y − 2 z − 2] 2
is the equation.

34. Find the equation of the right circular cylinder whose axis is along the line

y +1 z −1
x−2= =
−3 2
with the radius of its base as 3 units.

Let A ≡ (2,-1, 1), then AM 2


=
[ ( y − 2 ) − 3( y + 1) + 2( z − 1) ]
2
. (check fig.
14
1.9 )
But AM2 = AP2 – PM2 , by Pythagoras Theorem.
= (x – 2)2 + (y +1)2 + (y - 1)2 –9 .

114
∴(x –3y + 2z -7)2 = 14[(x-2)2 + (y + 1)2 + (z - 1)2] –126 is the equation of the
right circular cylinder.

35. Show that the equation of a right circular cylinder whose axis is along x –2 = 0 = z
–1
of radius unity, is x2 + y2 – 4x – 2z + 4 = 0.

x − 2 y − 0 z −1
The equations of the axis are = = .
0 1 0
Here A = (2, 0, 1) and r = 1.
∴We have x2 + z2 – 4x – 2z +4 = 0 as the equation.

x−2 y −3 z −4
36. Find the equation of a right circular cylinder whose axis is = = and
3 4 5

x−4 y −3 z −2
a generator is = =
3 4 5

A ≡ (2, 3, 4) and Q ≡ (4, 3, 2) is a point on the surface of the cylinder.


If QM perpendicular to the axis, QM = r, radius and r2 = AQ2 – AM2

=4+0+4−
[ 3(2) + 4(0) + 5( −2)]
2

=
192
.
50 25
If P ≡ (x, y, z) is any general point on the surface of the cylinder, then
AL2 = AP2 – PL2, PL perpendicular to the axis.

[ 3( x − 2) + 4( y − 3) + 5( z − 4)] 2

= ( x − 2) 2 + ( y − 3) 2 + ( z − 4) 2 − 192 / 25.
50
or [
50 ( x − 2 ) + ( y − 3) + ( z − 4 )
2 2 2
] − (3x + 4 y + 5z − 38) 2
= 384
is the equation of the right circular cylinder.

115
TEST :
1. If α, β, γ are the angles made by a line with the coordinate axes, the value of sin2α+
sin2 β + sin2 γ is
a) 1 b) 2 c) 3 d) 4
2. A straight line is inclined at angles 45o and 60o with the y and the z-axes respectively.
Its inclination with x-axes is
a) 60o b) 45o c) 75o d) 90o
3. The D.C.s of the line perpendicular to the plane ABC formed by A≡(1, 1, 1), B≡(3, 4,
-1) and C≡(1, 4, 0) are
3 2 6 3 2 6 3 −2 6
a) 3, 2, 6 b) , , , c) , , d) , ,
7 7 7 7 7 7 7 7 7
4. The foot of the perpendicular from the origin to the line joining (1, 2, 3) and (2, 3, 4)
is
a) (1, 0,1) b) (-1, 0, 1) c) (1, 0, -1) d) (-1, 0, -1)
5. The points A(3, 4, 2), B(6, -1, 10) and C(9, 1, 4) are the vertices of a triangle.
Traingle ABC is
a) Isosceles b) Right angled c) equilateral d) Right angled isosceles
6. The perpendicular distance between the planes 2x – 2y + z + 3 = 0 and 4x – 4y +
2y +9 = 0 is
1 1 1 2
a) b) c) d)
2 6 3 3
7. The equation of the plane through the points (2, 3, 4), (-3, 5, 1) and (4, -1, 2) is
a) x-y+z1=0 b) x+y-z+1=0 c) x+y-z-1=0 d) x-y-z-1 =0
8. If the points (1, 1, -1), (7, 1, 2), (λ, -1, 2) and the origin are coplanar, the value of λ is
a) 1 b) 2 c) 3 4) 4
9. The equation of the plane through (1, -2, 4) and parallel to the plane x+2y-3z-7 = 0
is
a) x+2y-2z=0 b) x+2y-3z+7=0 c) x+2y-3z-15=0 d) x+2y-3z+15=0
10. The acute angle between the plane 2x-y+z = 7 and x+y+2z-11 = 0 is
π 2π −1 5 −1 1
a) b) c) cos d) cos
3 3 6 6

116
11. The equation of the plane through the line of intersection of the planes 2x-
3y+4z+1=0 and x+y+z-5=0 and perpendicular to the latter plane is
a) 3x-2y+5z-4=0 b) x-4y+3z+6=0 c) 11x+6y+13z-44=0 d) 12x+7y+14z-
49=0
12. The foot of the perpendicular from (3, 2, 1) to x+3y-2z+7 = 0 is
a) (2, -2, 3) b) (4, 5, -1) c) (2, -1, -1) d) (3, 2, 1)
13. The equations of the straight line joining the points (3, 2, 4) and (4, 5, 2) are
x−3 y−2 z −4 y−2 z−4
a) = = b) x − 3 = =
4 5 2 3 2
x−4 y −5 z −2 y−3 z +2
c) = = d) x − 1 = =
3 2 4 3 −2
14. The equation of the plane through the line of intersection of the planes 3x-4y+5z-10
=0
and 2x+2y-3z-4 = 0 and parallel to the straight line x = 2y = 3z is
a) x+20y –27z=14 b) x-20y-27z+14=0 c) x-20y +27z=14 d) x +20y – 27z
+14=0
15. Equations of the straight line through the point (2, 3,1) and parallel to the line
x-2y-z +5 = 0 = x+y+3z-6 are
x − 2 y − 3 z −1 x − 2 y − 3 z −1
a) = = b) = =
5 −4 −3 5 4 −3
x − 2 y − 3 z −1 x − 2 y − 3 z −1
c) = = d) = =
−5 4 3 5 −4 3
16. Equations of the straight line through the point (2, 1, -3) and perpendicular to the
plane 3x+2y-4y-7=0 are
x − 2 y −1 z + 3 x − 2 y −1 z − 3
a) = = b) = =
3 2 4 3 2 4
x − 2 y −1 z + 3 x − 2 y −1 z −1
c) = = d) = =
3 2 −4 3 2 −4
17. Equations of the straight line through the point (3, 4, -3) and perpendicular to two
lines whose D.C.s are
1:2:2 and 2 : 1 : 3 are

117
x−3 y−4 z +3 x−3 y−4 z −3
a) = = b) = =
4 −1 −3 4 1 −3
x−3 y−4 z −3 x−3 y−4 z +3
c) = = d) = =
−4 1 −3 4 1 −3
18. A(1, 2, 3), B(2, 1, -5) and C(4, -1, 3) are the vertices of a triangle.
The equations of the median through A are
x −1 y − 2 z − 3 x − 2 y −1 z + 5
a) = = b) = =
−1 1 2 −1 −1 2
x − 4 y +1 z − 3 x −1 y − 2 z −1
c) = = d) = =
1 −1 4 1 −1 4
19. The equation of the plane through the point (2, -1, 0) & (3, -4, 5) and parallel to the
line 2x=3y=4z is
a) 29x-27y –22z=85 b) 29x+27y-22z-85=0 c) 29x-27y-22z+85=0 d) 29x-27y+22z-
85=0
x −1 y + 3 z − 4
20. The plane 4x+5y-z+15=0 ______________ the line = =
2 −1 3
a) Perpendicular b) Contains c) Parallel to d) is at 45o with
x −1 y −1 z +1
21. The point of intersection of the straight lines = = and
1 2 3

x+2 y−5 z +2
= = is
2 −1 2
a) (2, 3, 1) b) (2, 3, 2) c) (3, 3, 1) d) (3, 3, 2)
x −1 y − 2 z − 3
22. The straight line = = is non-coplanar with one of the following
2 3 4
straight lines.
x − 2 y − 8 z − 11 3x − 5 y − 3 3z − 13
a) = = b) = =
2 −1 2 2 1 4
x−2 y −4 z −5 y +1 z −1
c) = = d) x = =
3 4 5 3 2
23. The shortest distance between the straight lines x+y = 0 = y+z and x+y = 0 =
x+y+z-a is

118
a 2a 2a 2
a) b) c) d) a
6 3 3 3
24. The equation x2+y2-z2=0 represents a
a) Sphere b) Cone c) Cylinder d) Two planes
25. The equation y2+z2=1, in space, represents a
a) Cylinder b) cone c) Sphere d) none of the three

119
Notes by Prof. A.T.Eshwar, PES, Mandya

Visvesvaraya Technological University, Belgaum

WEB-BASED EDUCATION

Engineering Mathematics – I (I Semester)


(VTU Sub.Code:MAT -11)
Syllabus
PART –A
1. Analytical geometry in 3 dimensions
Direction cosines and direction ratios - planes - straight lines -Angle between
planes/straight lines - coplanar lines-shortest distance between skew lines, right circular
cone and right circular cylinder.
PART - B
2. Differential calculus
Determination of nth derivatives of standard functions, Leibnitz's theorem (without
proof) - problems only. Polar curves - Angle between the radius vector and the tangent -
pedal equations of polar curves only. Partial differentiation: Euler's Theorem. Total
differentiation.
Differentiation of Composite and implicit functions - Jacobins - Errors and
approximations. -Illustrative Engg. Oriented problems.
PART – C
3. Integral calculus
Reduction formulae for the functions sinn x, cosn x, tann x, cotn x, secn x, cosec n x and
sinm x cosn x - Evaluation of these integrals with standard limits - problems. Tracing of
standard curves in Cartesian form, parametric form and Polar form,
Applications to find area, length, volume and surface area.
PART - D
4. Differential equations
Solutions of 1st order & 1st degree equation- variables separable - Homogeneous and
Non- Homogeneous, Exact equations and reducible to exact form, Linear and Bernoulli’s
equations. Orthogonal trajectories of Cartesian and polar forms. (Use of initial condition
should be emphasized).-Illustrative examples from engineering field.

5. Infinite Series
Convergence, divergence and oscillation of an infinite series, Comparison Test, P-series,
D'Alembert's ratio test, Raabe's test, Cauchy's root test, Cauchy's integral test (All tests
without proof) for series of positive terms. Alternating series. Absolute and conditional
convergence, Leibnitz's test (without proof)

120
Text Book:
1. Higher Engineering Mathematics by B.S.Grewal, 36th Edition - July 2001.

Reference book:
2. Advanced Engineering Mathematics: - by E. Kreyszig, John Wiley & Sons, 6th Ed.

• Engineering Applications of MAT-11 at glance

Sl. Engg. Maths -I Applications


No (Code:MAT 11)
1 PART –A Wave Theory in E&C; Filed Theory in E&E;
Drawings in Mech. Engg.; Applied Mechanics in Civil
Analytical
Engg.,
Geometry
in 3 dimensions
2 PART - B Basic of Applied Mechanics, Bending, deflection torsion,
Differential calculus in Civil & Mech. Engg.; Spreading the functions &
Electron beam deflections in E&C and E&E
3 Basic Thermodynamics for Mech.Engg., Area, Volume
Part – C
and Surface area, Tracing of Curves for all branches of
Integral Calculus
Engg.,
4 Mathematical Modeling in circuits theory for E&E,
PART - D
Differential E&C, CS&E, IS&E, Mechanical Vibrations, Deflection
equations of beams,Whilring of shafts in Civil/ Mech.Engg

5 Theory of signals in E&E, E&C; Series solutions of


PART - D
Differential Equations in all branches; Description of
Periodic phenomena
Infinite
Series

• Contents of Part-B

Differential Calculus
(Part-B)
A.T.Eswara, PESCE, Mandya

121
Chapter 1 Chapter 2 Chapter 3
Successive Polar Curves Partial
Differentiation Differentiation

Lesson-1 Lesson-2 Lesson-1 Lesson-2 Lesson-1 Lesson-2 Lesson-3


th
n derivative Leibnitz` Angle Pedal Euler’s Total derivative, Applications to
of standard s theorem between equations theorem differentiation of Jacobians, Errors
functions polar curves composite & & Approximations
implicit functions
Chapter – 1

LESSON -1 : Successive Differentiation

• In this lesson, the idea of differential coefficient of a function and its


successive
derivatives will be discussed. Also, the computation of nth derivatives of some
standard
functions is presented through typical worked examples.
1.0 Introduction:- Differential calculus (DC) deals with problem of calculating
rates of
change.When we have a formula for the distance that a moving body covers as a
function of
time, DC gives us the formulas for calculating the body’s velocity and
acceleration at any
instant.
• Definition of derivative of a function y = f(x):-

122
f ( x + ∆x) − f ( x)
Fig.1. Slope of the line PQ is
∆x
The derivative of a function y = f(x) is the function f ′(x) whose value at each x is
defined as
dy
= f ′(x) = Slope of the line PQ (See Fig.1)
dx
f ( x + ∆x) − f ( x)
= ∆xlim
→0 -------- (1)
∆x
= lim (Average rate change)
∆x → 0

= Instantaneous rate of change of f at x provided the limit exists.

The instantaneous velocity and acceleration of a body (moving along a line) at any instant
x is the derivative of its position co-ordinate y = f(x) w.r.t x, i.e.,
dy
Velocity = = f ′(x ) --------- (2)
dx
And the corresponding acceleration is given by

d2y
Acceleration = 2 = f ′′( x) ---------- (3)
dx
• Session - 1

1.1 Successive Differentiation:-


The process of differentiating a given function again and again is called as
Successive differentiation and the results of such differentiation are called
successive derivatives.
• The higher order differential coefficients will occur more frequently in spreading
a function all fields of scientific and engineering applications.
• Notations:
dy d 2 y d 3 y dny
i. , 2 , 3 ,…….., nth order derivative:
dx dx dx dx n
ii f ′(x ) , f ′′(x ) , f ′′′(x ) ,…..., nth order derivative: f n (x)
iii Dy, D 2 y , D 3 y ,………..., nth order derivative: D n y
iv y ′ , y ′′ , y ′′′ ,……, nth order derivative: y (n )
v. y1 , y 2 , y 3 …, nth order derivative: y n
• Successive differentiation – A flow diagram
df
Input function: y = f (x) Operation Output function y′ = = f ′(x) (first order
d dx
dx
derivative)

123
d2 f
Input function y ′ = f ′(x) Operation Output function y ′′ = = f ′′( x ) (second order
d
dx
dx 2
derivative)

d3 f
Input function y ′′ = f ′′(x) Operation Output function y ′′′ = = f ′′′( x) (third order
d
dx
dx 3
derivative)

-----------------------------------------------------------------------------------------------------------
------
dn f
Input function y n −1 = f n −1 ( x) Operation Output function y n = n
= f n (x) (nth order
d
dx
dx
derivative)

Animation Instruction
(Successive Differention-A flow diagram)
Output functions are to appear after operating

Operation
d
dx

on Input functions, successively.

• 1.1 Solved Examples :


d2y dy
1. If y = sin(sin x) , prove that 2
+ tan x + y cos 2 x = 0
dx dx
Solution: Differentiating y = sin(sin x) --------- (1) w.r.t.x, we get
dy
y1 = = cos(sin x). cos x -------------- (2)
dx
dy
Again differentiating y1 = w.r.t.x gives
dx
d2y
y 2 = 2 = [ cos(sin x)(− sin x ) + cos x (− sin(sin x ) cos x ] Using product rule
dx
d2y
[
y 2 = 2 = − sin x cos(sin x) + cos 2 x sin(sin x)
dx
]
 sin x 
i.e. y 2 = −  cos x cos(sin x) + cos 2 x sin(sin x)
 cos x 

124
[ ]
y 2 = − tan xy1 + cos 2 xy , using Eqs. (1) and (2)
or y 2 + tan xy1 + cos 2 xy = 0
d2y dy
or 2
+ tan x + y cos 2 x = 0
dx dx

(ax + b)
2. If y = , show that 2 y1 y 3 = 3 y 2
2
(cx + d )
(ax + b)
Solution: We rewrite y = , by actual division of ax+b by c x+d, as
(cx + d )
 ad  1 a  ad 
y = a + b − = + k ( cx + d ) ---------- (1)where k =  b −
−1
c   
c  cx + d c  c 
Differentiating (1) successively thrice, we get
dy
= y1 = − kc( cx + d )
−2
---------- (2)
dx
d2y
= y 2 = −2kc 2 ( cx + d )
−3
2
---------- (3)
dx
d3y
= y 3 = −6kc 3 ( cx + d )
−4
3
---------- (4) From (2), (3) and (4) we get
dx
[{ }{
2 y1 y 3 = 2 − kc(cx + d ) −2 − 6kc 3 (cx + d ) −4 }]
2 y1 y 3 = 12k 2 c 4 (cx + d ) −6
[
2 y1 y 3 = 3 − 2kc 2 (cx + d ) −3 ] 2

Therefore 2 y1 y3 = 3 y 2 , as desired.
2

d2y dy
3. If x = sin t , y = sin pt , Prove that (1 − x 2 ) 2
− x + p2 y = 0
dx dx
Solution: Note that the function is given in terms a parameter t. So we find,
dy dy
= cos t and = p cos pt , so that
dt dt
dy dy dt p cos pt
y1 = = = . Squaring on both sides
dx dx dt cos t

( y1 ) 2 = p cos2 pt = p (1 − sin2 pt ) = p (1 − 2y ) (by data)


2 2 2 2 2 2

cos t 1 − sin t 1− x
( )
∴ 1 − x ( y1 ) = p 1 − y .
2 2 2
( 2
)
Differentiating this equation w.r.t x, we get
( )
1 − x 2 2 y1 y 2 + ( y1 ) 2 ( −2 x) = p 2 (−2 yy1 ) .

Canceling 2 y1 throughout, this becomes


( )
1 − x 2 y 2 − xy1 = − p 2 y

125
( )
or 1 − x 2 y 2 − xy1 + p 2 y = 0
d2y dy
i.e. (1 − x 2 ) 2
− x + p2 y = 0
dx dx
d2y
4. If x = a (cos t + t sin t ) , y = a (sin t − t cos t ) , find
dx 2
dy
Solution: = a( − sin t + t cos t + sin t ) = at cos t
dt
dy
= a( cos t + t sin t − cos t ) = at sin t
dt
 dy  dy at sin t
∴   = dt = = tan t
 dx  dx dt at cos t
d2y  dt   1  1
Hence, 2
= sec 2 t   = sec 2 t  = 3
dx  dx   at cos t  at cos t

( )
5. If y = a cosh x a , prove that a 2 y 22 = 1 + y12

Solution: y1 =
dy
dx a a
( )( )
= a sinh x 1 = sinh x , and
a
( )
( )( )
2
d y
y 2 = 2 = cosh x 1 ,
dx a a
( )
∴ ay 2 = cosh x , so that a 2 y 22 = cosh 2 x
a a
( )
i.e. a y 2 = 1 + sinh
2 2 2 x
a
( )
= 1 + y1 , as desired.
2

• Problem Set No. 1.1 for practice.

(
1. If y = e ax sin bx , prove that y 2 − 2ay1 + a 2 + b 2 y = 0 )
2
d y h − ab 2

2. If ax 2 + 2hxy + by 2 = 1 , prove that =


dx 2
( hx + by ) 3
3. If y = Ae − kt cos(l t + c) , show that
d2y
dx 2
+ 2k
dy
dx
+ k2 + l2 y = 0( )
( ) ( )
2
d y dy
4. If y = log x + 1 + x 2 , prove that 1 + x 2 2
+x =0
dx dx
2
d2y  dy 
5. If y = tan −1 ( sinh x ) , prove that + tan y   =0
dx 2  dx 
d2y
6. If x = a ( cos t + log tan t 2 ) , y = a sin t , find Ans:
dx 2
sin t
a cos 4 t

126
d2y
7. Find 2
, when x = a cos 3 θ , y = b sin 3 θ Ans:
dx
b cos ecθ sec 4 θ
3a 2
d3y  x  1 + 2x 2
8. Find 3 , where y = tan  
−1
Ans:
dx  1− x 
2 
(1 − x )
2
5
2

d2y π
9. If x = 2 cos t − cos 2t , x = 2 sin t − sin 2t , Find  2  x = Ans: -3/2
 dx  2
d2y dy
10. If xy = e x + be − x , prove that x 2 + 2 − xy = 0
dx dx

• Session -2

1.2 Calculation of nth derivatives of some standard functions

• Below, we present a table of nth order derivatives of some standard functions for
ready reference.
Table : 1

127
Sl. y = f(x) dny
No yn = n
= Dn y
dx
1 e mx m n e mx
2 a mx m n ( log a ) a mx
n

3 ( ax + b ) m i. m( m − 1)( m − 2 )....( m − n + 1) a n ( ax + b )
m− n
for all m .
ii. 0 if m < n
iii. ( n!) a n if m = n
m!
iv. x m −n if m < n
( m − n )!
4 1 (−1) n n ! n
a
( ax + b ) (ax + b) n +1
5. 1 ( −1) n (m + n − 1) ! n
a
( ax + b ) m (m − 1) !(ax + b) m + n
6. log(ax + b) (−1) n −1 (n − 1) ! n
a
(ax + b) n
7. sin( ax + b) a n sin( ax + b + n π )
2
8. cos(ax + b) a cos(ax + b + n π )
n
2
9. e ax sin(bx + c) r e sin(bx + c + nθ ) , r = a 2 + b 2
n ax
θ = tan −1 ( b a )
10. e ax cos(bx + c ) r n e ax cos(bx + c + nθ ) , r = a 2 + b 2 θ = tan −1 ( b a )

• We proceed to illustrate the proof of some of the above results, as only the above
functions are able to produce a sequential change from one derivative to the
other. Hence, in general we cannot obtain readymade formula for nth derivative
of functions other than the above.

1. Consider e mx . Let y = e mx . Differentiating w.r.t x, we get


y1 = me mx . Again differentiating w.r.t x, we get
( )
y 2 = m me mx = m 2 e mx
Similarly, we get
y3 = m 3 e mx
y4 = m 4 e mx

128
…………….
And hence we get
dn
yn = m n e mx ∴ n e mx = m n e mx .
dx
[ ]
2. ( ax + b ) m (See Sl. No-3 of Table-1 )
let y = ( ax + b ) m Differentiating w.r.t x,
y1 = m ( ax + b ) m −1 a . Again differentiating w.r.t x, we get
y2 = m ( m − 1) ( ax + b ) m− 2 a 2
Similarly, we get
y3 = m ( m − 1) ( m − 2) ( ax + b ) m−3 a 3
………………………………….
And hence we get
yn = m ( m − 1) ( m − 2) ………. ( m − n + 1) ( ax + b ) m − n a n for all m.
Case (i) If m = n (m-positive integer),then the above expression becomes
yn = n ( n − 1) ( n − 2 ) ……….3.2.1 ( ax + b ) n −n a n
i.e. y n = ( n!) a
n

Case (ii) If m<n,(i.e. if n>m) which means if we further differentiate the above
expression, the
right hand site yields zero. Thus D n ( ax + b ) m = 0 if ( m < n ) [ ]
If m>n, then y n = m( m − 1)( m − 2 )......( m − n + 1)( ax + b ) a n becomes
m−n
Case (iii)
m( m − 1)( m − 2 )......( m − n + 1)( m − n ) !
= ( ax + b ) m−n a n
( m − n) !
m!
. i.e yn = ( ax + b ) m−n a n
( m − n) !
1
3. (See Sl. No-5 of Table-1 )
( ax + b ) m
1
= ( ax + b )
−m
Let y=
( ax + b ) m

Differentiating w.r.t x
y1 = −m( ax + b ) a = ( − 1) m( ax + b )
− m −1 − ( m +1)
a
[
y 2 = ( − 1)( m ) − ( m + 1)( ax + b )
− ( m +1) −1
]
a = ( − 1) m( m + 1)( ax + b )
2 −( m+2 )
a2
y 3 = ( − 1) m( m + 1)( m + 2 )( ax + b )
3 − ( m+ 3 )
Similarly, we get a3
y 4 = ( − 1) m( m + 1)( m + 2 )( m + 3)( ax + b )
4 − ( m+ 4 )
a4
……………………………
y n = ( − 1) m( m + 1)( m + 2).....( m + n − 1)( ax + b )
n −( m+n ) n
a

129
This may be rewritten as

yn =
( − 1) n ( m + n − 1)( m + n − 2).....( m + 1) m( m − 1) ! ( ax + b ) −( m+n ) a n
( m − 1) !
yn =
( − 1) ( m + n − 1) ! n
n
a
or
( m − 1) !( ax + b ) m+n

1
4.
( ax + b ) (See Sl. No-4 of Table-1 )
Putting m = 1, in the result
 1  (−1) n (m + n − 1) ! n
Dn  m 
= m+n
a
 ( ax + b)  ( m − 1) !(ax + b)
n 1  (−1) n (1 + n − 1) ! n
we get D  = 1+ n
a
 (ax + b)  (1 − 1) !(ax + b)
n 1  (−1) n n ! n
or D   = 1+ n
a
 ( ax + b)  (ax + b)

• 1.2.1. Worked Examples:-


In each of the following Questions find the nth derivative after reducing them into
standard functions given in the table 1.2.1
(3 x + 5) 2 (2 − 3 x)
1. (a) log(9 x 2 − 1) [
(b) log (4 x + 3)e 5 x +7 ]
(c) log 10
( x + 1) 6
Solution: (a) Let y = log(9 x 2 − 1) = log{ (3 x + 1)(3 x − 1)}
y = log(3 x + 1) + log(3 x − 1) ( log( AB ) = log A + log B )
dn dn
∴ y n = n { log(3x + 1)} + n { log(3 x − 1)}
dx dx
(−1) (n − 1) ! n (−1) n −1 (n − 1) ! n
n −1

i.e y n = (3) + (3)


(3 x + 1) n (3 x − 1) n

[ ]
(b) Let y = log ( 4 x + 3)e 5 x +7 = log(4 x + 3) + log e 5 x +7
= log(4 x + 3) + (5 x + 7) log e e ( log A B = B log A )
∴ y = log(4 x + 3) + (5 x + 7) ( log e e = 1 )
(−1) n −1 (n − 1) ! n
∴ yn = ( 4) + 0 D(5 x + 6) = 5
(4 x + 3) n
D 2 (5 x + 6) = 0
D n (5 x + 1) = 0 (n > 1)

130
(3 x + 5) 2 (2 − 3 x)
(c) Let y = log10
( x + 1) 6
1  (3 x + 5) 2 (2 − 3 x)  log e X
=   log10 X =
log e 10  ( x + 1) 6  log e 10

1 1  (3 x + 5) 2 (2 − 3 x) 
=  log   log A B = B log A
log e 10  2  ( x + 1) 6 

 A
log  = log A − log B
B
1
=
2 log e 10
{
log(3 x + 5) 2 + log(2 − 3x ) − log( x + 1) 6 }
1
∴ y= { 2 log(3x + 5) + log(2 − 3x) − 6 log( x + 1)}
2 log e 10
Hence,

1  ( −1) n −1 ( n − 1) ! n (−1) n −1 (n − 1) ! ( −1) n −1 ( n − 1) ! n 


yn = 2. (3) + (−3) − 6.
n
(1) 
2 log e 10  (3 x + 5) n ( 2 − 3 x) n ( x + 1) n 

2. (a) e 2 x + 4 + 6 2 x + 4 (b) cosh 4 x + cosh 2 4 x


1 1
(c) e − x sinh 3 x cosh 2 x (d) + + (6 x + 8) 5
(4 x + 5) (5 x + 4) 4

Solution: (a) Let y = e 2 x + 4 + 6 2 x + 4


= e2xe4 + 62x64
∴ y = e 4 (e 2 x ) + 1296(6 2 x )
dn dn
hence y n = e 4 n (e 2 x ) + 1296 n (6 2 x )
dx dx
{ } {
= e 4 2 n e 2 x + 1296 2 n (log 6) n 6 2 x }
(b) Let y = cosh 4 x + cosh 2 4 x
2
 e 4 x + e −4 x   e 4 x + e −4 x 
=   +  
 2   2 
1
( ) {
1
}
= e 4 x + e −4 x + (e 4 x ) 2 + (e −4 x ) 2 + 2(e 4 x )(e − 4 x )
2 4
1
( ) {1
y = e 4 x + e − 4 x + e 8 x + e −8 x + 2
2 4
}

131
hence, yn =
2
[
1 n 4x
]
1
[
4 e + (−4) n e − 4 x + 8 n e 8 n + (−8) n e −8 n + 0
4
]
(c) Let y = e − x sinh 3 x cosh 2 x
−x  e 3 x − e −3 x  e 2 x + e −2 x 
=e   
 2  2 
−x
=
e
4
{
(e 3 x − e −3 x )(e 2 x + e − 2 x )}
e −x 5x
=
4
{
e − e − x + e x − e −5 x }
{
1 4x
= e − e − 2 x + 1 − e −6 x
4
}
1
{
y = 1 + e 4 x − e −2 x − e −6 x
4
}
Hence,
1
{
y n = 0 + (4) n e 4 x − (−2) n e − 2 x − (−6) n e −6 x
4
}
1 1
(d) Let y = + + (6 x + 8) 5
(4 x + 5) (5 x + 4) 4

Hence, y n =
dn  1  dn 
  + 
1
dx  (4 x + 5)  dx  (5 x + 4)  dx
n n
 dn
4 
{
+ n ( 6 x + 8)
5
}
(−1) n n ! (−1) n (4 + n − 1) !
= n +1
(4) +
n
4+ n
(5) n + 0
(4 x + 5) (4 − 1) !(5 x + 4)
(−1) n n ! (−1) n (3 + n) ! n
i.e y n = (4) +
n
(5)
(4 x + 5) n +1 3!(5 x + 4) n + 4

• Session - 3
• 1.2.2 Worked examples:-

132
1 1 x2
1. (i) (ii) (iii)
x 2 − 6x + 8 1 − x − x 2 + x3 2x 2 + 7x + 6

 x + 2
(iv)  + 2
1
 x + 1  4 x + 12 x + 9
−1
( ) −1  1 + x 
(v) tan x a (vi) tan −1 x (vii) tan 
1− x 

• In all the above problems, we use the method of partial fractions to reduce
them into standard forms.
1 1
Solutions: (i) Let y = 2 . The function can be rewritten as y =
x − 6x + 8 ( x − 4)( x − 2)
• This is proper fraction containing two distinct linear factors in the denominator.
So, it can be split into partial fractions as
1 A B
y= = + Where the constant A and B are found
( x − 4)( x − 2) ( x − 4) ( x − 2)

as given below.

1 A( x − 2) + B ( x − 4)
=
( x − 4)( x − 2) ( x − 4)( x − 2)

∴ 1 = A( x − 2) + B ( x − 4) -------------(*)
Putting x = 2 in (*), we get the value of B as B = − 1 2

Similarly putting x = 4 in(*), we get the value of A as A = 1 2


1 (1 / 2) (−1 / 2)
∴y= = + Hence
( x − 4)( x − 2) x − 4 x−2
1 dn  1  1 dn  1 
yn =  −  
2 dx n  x − 4  2 dx n  x − 2 

1  (−1) n n ! n 1  (−1) n n ! 
=  n +1
(1)  −  n +1
(1) n 
2  ( x − 4)  2  ( x − 2) 

1  1 1 
= (−1) n n !  n +1
− n +1 
2  ( x − 4) ( x − 2) 

1 1 1
(ii) Let y = = =
1− x − x + x
2 3
(1 − x) − x (1 − x) (1 − x)(1 − x 2 )
2

133
1 1
ie y = =
(1 − x)(1 − x)(1 + x) (1 − x) 2 (1 + x)
Though y is a proper fraction, it contains a repeated linear factor (1 − x) 2
in its
denominator. Hence, we write the function as
A B C
y= + + in terms of partial fractions. The constants
(1 − x) (1 − x) 2
1+ x
A, B, C
are found as follows:

1 A B C
y= = + +
(1 − x) (1 + x)
2
(1 − x) (1 − x) 2
1+ x

ie 1 = A(1 − x )(1 + x) + B (1 + x) + C (1 − x) 2 -------------(**)


Putting x = 1 in (**), we get B as B = 1 2

Putting x = -1 in (**), we get C as C = 1 4


Putting x = 0 in (**), we get 1 = A + B + C
∴ A = 1− B − C = 1− 1 − 1 = 1
2 4 4
∴A= 1
4
(1 / 4) (1 / 2) (1 / 4)
Hence, y = + +
(1 − x) (1 − x) 2
(1 + x)
1  (−1) n n ! n  1  (−1) n (2 + n − 1) ! n  1  (−1) n n ! n 
∴ yn =  (1)  +  (1)  +  (1) 
4  (1 − x ) n +1  2  (2 − 1) !(1 − x)
2+ n
 4  (1 + x)
n +1

1  1 1  1  ( −1) n (n + 1) !
= (−1) n n !  n +1
+ +  
4  (1 − x ) (1 + x) n +1  2  (1 − x) n + 2 
x2
(iii) Let y = (VTU July-05)
2x 2 + 7x + 6
This is an improper function. We make it proper fraction by actual division
and later
spilt that into partial fractions.
1 (− 7 x − 3)
i.e x ÷ ( 2 x + 7 x + 6) = + 2 2
2 2

2 2x − 7x + 6
1 −7 x − 3
∴y= + 2
Resolving this proper fraction into partial fractions,
2 ( 2 x + 3)( x + 2)
we get

134
1  A B 
y= + + . Following the above examples for finding A &
2  (2 x + 3) ( x + 2) 
B, we get
1  92 (−4) 
y= + +
2  2 x + 3 x + 2 
9  (−1) n n ! n  ( −1) n n ! 
Hence, y n = 0 +  n +1
( 2 )  − 4  n +1
(1) n 
2  (2 x + 3)   ( x + 2) 
 9 2 ( 2) n 4 
y
i.e n = ( − 1) n
n !  n +1
− 
 (2 x + 3) ( x + 2) n +1 
( x + 2) x
(iv) Let y = + 2
( x + 1) 4 x + 12 x + 9

(i) (ii)
Here (i) is improper & (ii) is proper function. So, by actual division (i)
becomes
 x + 2  1 
  = 1+   . Hence, y is given by
 x +1   x + 1

 1  1
y = 1+  + [ (2 x + 3) 2 = 4 x 2 + 12 x + 9 ]
 x + 1  (2 x + 3)
2

Resolving the last proper fraction into partial fractions, we get


x A B
= + . Solving we get
(2 x + 3) 2
(2 x + 3) (2 x + 3) 2
A = 1 and B = − 3
2 2
 1   12 − 32 
∴ y = 1+  + + 2 
 1 + x   (2 x + 3) (2 x + 3) 
 (−1) n n ! n  1  ( −1) n n ! n 3  (−1) n (n + 1) ! n 
∴ yn = 0 +  (1)  +  n +1
( 2 )  −  (2) 
 (1 + x)  2  (2 x + 3)  2  ( 2 x + 3)n + 2
n

(v) tan x a
−1
( )
Let y = tan x
−1
( a)
1 1 a
∴ y1 =  = 2
1+ x( a) 2
a x +a
2

 a 
y n = D n y = D n −1 ( y1 ) = D n −1  2 2 
x +a 

135
a a
Consider =
x +a
2 2
( x + ai )( x − ai )
A B
= + , on resolving into partial fractions.
( x + ai ) ( x − ai )

=
(
−1
2 i +
) ( )
1
2i , on solving for A & B.
( x + ai ) ( x − ai )
 a  n −1  − 2 i  n −1  
1 1
∴ D n −1  2  = D   + D 
2i

x +a   x + ai   x − ai 
2

 1   (−1) (n − 1) !  1   (−1) (n − 1) !
n −1 n −1
=  −   +     -----------(*)
 2i   ( x + ai )   2i   ( x − ai )
n n

• Since above answer containing complex quantity i we rewrite the answer in terms of
real quantity, We take transformation x = r cos θ a = r sin θ where r = x 2 + a 2 ,
a
θ = tan −1  
 x
x + ai = r ( cos θ + i sin θ ) = re iθ
x − ai = r ( cos θ − i sin θ ) = re − iθ
1 1 e inθ 1 e −inθ
= = , =
( x − ai ) n r n e −inθ r n ( x + ai ) n r n

=
( − 1) ( n − 1)! inθ
n −1
[e − e −inθ ]
now(*) is y n n
2ir

yn =
( − 1) n −1 ( 2 i sin nθ ) ⇒ ( − 1) n−1 ( n − 1)! sin nθ
2 i rn rn

(vi) Let y = tan −1 x .Putting a = 1 in Ex.(v) we get


yn which is same as above with r = x 2 + 1 θ = tan −1 1
x
( )
θ = cot ( x ) or x = cot θ
−1

1 1
∴ r = cot 2 θ + 1 = cos ecθ ⇒ n = = sin n θ
r cos ec θ
n

n
( −1
)
D tan x = ( − 1) ( n − 1) ! sin θ sin nθ where θ = cot −1 x
n −1 n

−1  1 + x 
(vii) Let y = tan  
1− x 
put x = tan θ θ = tan −1 x
1 + tan θ 
∴ y = tan −1 
1 − tan θ 

136
= tan −1 [ tan(π 4 + θ )] (
tan π
4
)  1 + tan θ 
+θ =  
 1 − tan θ 
π
=π +θ = + tan −1 ( x)
4 4
π
y= + tan −1 ( x)
4
y n = 0 + D n (tan −1 x )
 1   (−1) (n − 1) !  1   (−1) (n − 1) !
n −1 n −1
=  −   +   
 2i   ( x + ai )   2i   ( x − ai )
n n

Problem set No. 1.2.1 for practice


Find the nth derivative of the following functions:
6x x x 2 + 4x + 1 x
1. 2. 3. 4.
( x − 1)( x 2 − 4) ( x + 2)( x 2 − 2 x + 1) x3 + 2x 2 − x − 2 4x − x − 3
2

x3  2x   1 + x2 −1
7. tan  
−1 −1
5. 2 6. tan  2  
x − 3x + 2 1− x   x 

137
• Session 4
1. sin( ax + b) .(See Sl. No-7 of Table-1 )
Let y = sin( ax + b) . Differentiating w.r.t x,
π
y1 = cos(ax + b).a As sin( X + ) = cos X
2
We can write
y1 = a sin(ax + b + π / 2).
Again Differentiating w.r.t x,
π
y 2 = a cos(ax + b + π / 2).a Again using sin( X + ) = cos X ,we get
2
y 2 as
y 2 = a sin( ax + b + π / 2 + π / 2).a
i.e. y 2 = a 2 sin( ax + b + 2π / 2).
Similarly, we get
y 3 = a 3 sin(ax + b + 3π / 2).
y 4 = a 4 sin( ax + b + 4π / 2).
……………………………
y n = a n sin( ax + b + nπ / 2).
2. e ax sin ( bx + c ). (See Sl. No-9 of Table-1 )
Let y = e ax sin ( bx + c ).....(1)
Differentiating using product rule ,we get
y1 = e ax cos( bx + c ) b + sin ( bx + c ) ae ax
i.e. y1 = e ax [ a sin ( bx + c ) + b cos( bx + c ) ] . For computation of higher order
derivatives
it is convenient to express the constants ‘a’ and ‘b’ in terms of the
constants r and
θ defined by a = r cos θ & b = r sin θ ,so t r = a 2 + b 2 and
( )
θ = tan −1 b a .thus,
y1 can be rewritten as
y1 = e ax [ ( r cos θ ) sin ( bx + c ) + ( r sin θ ) cos( bx + c ) ]
or y1 = e ax [ r{sin ( bx + c ) cos θ + cos( bx + c ) cos θ }]
i.e. y1 = re ax [ sin ( bx + c + θ ) ]...........(2)
Comparing expressions (1) and (2), we write y 2 as
y 2 = r 2 e ax sin ( bx + c + 2θ )
y 3 = r 3 e ax sin ( bx + c + 3θ )

138
Continuing in this way, we get
y 4 = r 4 e ax sin ( bx + c + 4θ )
y 5 = r 5 e ax sin ( bx + c + 5θ )
…………………………….
y n = r n e ax sin ( bx + c + nθ )
[ ]
∴ D n e ax sin ( bx + c ) = r n e ax sin ( bx + c + nθ ) , where r = a 2 + b 2 &
( )
θ = tan −1 b a

• 1.2.3 Worked examples


1. (i) sin 2 x + cos 3 x (ii) sin 3 cos 3 x (iii) cos x cos 2 x cos 3 x
(iv) sin x sin 2 x sin 3 x (v) e 3 x cos 2 x (
(vi) e 2 x sin 2 x + cos 3 x )
• The following formulae are useful in solving some of the above problems.
1 − cos 2 x 1 + cos 2 x
(i) sin x = (ii ) cos 2 x =
2

2 2
(iii) sin 3 x = 3 sin x − 4 sin 3 x (iv) cos 3 x = 4 cos 3 x − 3 cos x

(v) 2 sin A cos B = sin ( A + B ) + sin ( A − B )


(vi) 2 cos A sin B = sin ( A + B ) − sin ( A − B )
(vii) 2 cos A cos B = cos( A + B ) + cos( A − B )
(viii) 2 sin A sin B = cos( A − B ) − cos( A + B )

 1 − cos 2 x  1
Solutions: (i) Let y = sin x + cos x =   + ( cos 3 x + 3 cos x )
2 3

 2  4
1
2
[
n
( 2
1
)] [ (
∴ y n = 0 − ( 2 ) cos 2 x + nπ + ( 3) cos 3 x + nπ + 3 cos x + nπ
4
n
2 2
) ( )]
3
 sin 2 x  sin 3 2 x 1  − sin 6 x + 3 sin 2 x 
(ii)Let y = sin x cos x = 
3 3
 = =  
 2  8 8 4
1
= [ 3 sin 2 x − sin 6 x ]
32
1  n  nπ  n  nπ 
yn = 3.2 sin  2 x +  − 6 sin  6 x + 
32   2   2 

(iii) )Let y = cos 3 x cos x cos 2 x

139
1
2
[
( cos 4 x + cos 2 x ) cos 2 x = 1 cos 4 x cos 2 x + cos 2 2 x
=
2
]
1 1 1 − cos 4 x 
=  ( cos 6 x + cos 2 x ) + 
2 2 2
1 cos 2 x 1
= cos 6 x + + (1 − cos 4 x )
4 4 4
 nπ   nπ 
2 n cos 2 x +  4 cos 4 x +
n

1 n  nπ   2   2 
∴ y n = 6 cos 6 x + + −
4  2  4 4
(iv) )Let y = sin 3 x sin x sn 2 x
1
= [ sin ( 2 x ) − sin 4 x ] sin 2 x
2
1
[
= sin 2 2 x − sin 4 x sin 2 x
2
]
1 1 − cos 4 x 1 
=  − ( sin 2 x − sin 6 x ) 
2 2 2 
 1 − cos 4 x  1 
=   − ( sin 2 x − sin 6 x ) 
 4  4 
1 n  nπ   nπ   nπ 
yn =  4 cos 4 x +  − 2 sin  2 x +
n
 + 6 sin  6 x +
n

4  2   2   2 

(v) Let y = e 3 x cos 2 x (Refer Sl.No. 10 of Table 1)


∴ y n = re 3 x cos( 2 x + nθ ) where
−1  2 
r = 3 2 + 2 2 = 13 & θ = tan  
3
2x 2
(
(vi) Let y = e sin x + cos x
3
)
[
We know that sin x + cos x =
2 3

2
]
1 − cos 2 x 1
+ [ cos 3 x + 3 cos x ]
4
[ ] 1 − cos 2 x  e
2x
∴ y = e 2 x sin 2 x + cos 3 x = e 2 x   + 4 [ cos 3 x + 3 cos x ]
 2
1
[ ] 1
[
∴ y = e 2 x − e 2 x cos 2 x + e 2 x cos 3 x + 3e 2 x cos x
2 4
]
Hence,
1
[ 1
] [
y n = 2 n e 2 x − r1n e 2 x cos( 2 x + nθ1 ) + r2n e 2 x cos( 3 x + nθ 2 ) + 3r3n e 2 x cos( x + nθ 3 )
2 4
]
where r1 = 2 2 + 2 2 = 8 ; r2 = 2 2 + 3 2 = 13 ; r3 = 2 2 + 12 = 5
2 3 1
θ1 = tan −1   ;θ 2 = tan −1   ;θ 3 = tan −1   ;
2 2 2

140
Problem set No. 1.2.2 for practice
Find nth derivative of the following functions:
1. (sin 3 x + cos 2 x) 2. sin 2 x cos 3 x 3. cos 2 x. sin 3 x 4. cos x cos 2 x
5. sin x sin 2 x 6. e 3 x (sin 3 x + cos 2 x) 7. e x cos 2 x cos 4 x 8. e − x sin 2 x cos 2 x
.9. e −3 x cos 3 x (VTU Jan-04)

LESSON -2 : Leibnitz’s Theorem

• Session - 1
• Leibnitz’s theorem is useful in the calculation of nth derivatives of product of two
functions.
• Statement of the theorem:
If u and v are functions of x, then
D (uv) = D n uv + n C1 D n −1uDv + n C 2 D n −2 uD 2 v + ....+ n C r D n − r uD r v + ...uD n v ,
n

d n n( n − 1) n!
, C1 = n , C 2 = ,........, n C r =
n
where D =
dx 2 r!( n − r )!
• Worked Examples
1. If x = sin t , y = sin pt prove that
( )
1 − x 2 y n + 2 − ( 2n + 1) xy n +1 + p 2 − n 2 y n = 0 ( )
(VTU July-05)
Solution: Note that the function y = f (x) is given in the parametric form with a
parameter t.
So, we consider
dy dy dt p cos pt
= = (p – constant)
dx dx dt cos t
2
 dy  p 2 cos 2 pt p 2 (1 − sin 2 pt ) p 2 (1 − y 2 )
or   = = =
 dx  cos 2 t 1 − sin 2 t 1− x2
(
or 1 − x 2 y12 = p 2 1 − y 2 ) ( )
(
So that 1 − x 2 y12 − p 2 1 − y 2 ) ( )
Differentiating w.r.t. x,
[( )
1 − x ( 2 y1 y 2 ) + y1 ( − 2 x ) − p ( − 2 yy1 ) = 0
2 2 2
]
( )
1 − x 2 y 2 − xy1 + p 2 y = 0 --------------- (1) [ ÷ 2 y1 , throughout]
Equation (1) has second order derivative y 2 in it. We differentiate (1), n times,
term wise,
using Leibnitz’s theorem as follows.
[(
D n 1 − x 2 y 2 − xy1 − p 2 y = 0 ) ]

141
{ }
i.e D n (1 − x 2 ) y 2 − D n { xy1 } − D n p 2 y = 0( ) ---------- (2)

(a) (b) (c)


Consider the term (a):
[( ) ]
D n 1 − x 2 y 2 . Taking u = y 2 and v = (1 − x 2 ) and applying Leibnitz’s theorem
we get
D n [ uv ] = D n uv + n C1 D n −1uDv + n C 2 D n − 2 D 2 v + n C 3 D n −3 uD 3 v + ...
i.e
[ ]
D n y 2 (1 − x 2 ) = D n ( y 2 ).(1 − x 2 )+ n C1 D n −1 ( y 2 ).D(1 − x 2 )+ n C 2 D n −2 ( y 2 ) D 2 (1 − x 2 )+ n C 3 D n −3 ( y 2 ) D 3 (1 − x 2 ) +
n(n − 1) n(n − 1)(n − 2)
= y ( n ) + 2 − x 2 ) + ny ( n −1) + 2 .( −2 x) + y ( n − 2) + 2 .( −2) + . y ( n −3) + 2 .(0) + ...
2! 3!
[( ) ] ( )
D n 1 − x 2 y 2 = 1 − x 2 y n + 2 − 2nxy n +1 − n(n − 1) y n ----------- (3)

Consider the term (b):


D n [ xy1 ] . Taking u = y1 and v = x and applying Leibnitz’s theorem,
we get
D n [ y1 ( x)] = D n ( y1 ).( x )+ n C1 D n −1 y1 .D( x)+ n C 2 D n −2 ( y1 ).D 2 ( x) + ....
n(n − 1)
= y ( n ) +1 .x + ny ( n −1) +1 + y ( n −2 ) + 2 (0) + ....
2!
D n [ xy1 ] = xy n +1 + ny n ---------- (4)
Consider the term (c):

D n ( p 2 y) = p 2 D n ( y) = p 2 yn --------- (5)
Substituting these values (3), (4) and (5) in Eq (2) we get
{( ) }
1 − x 2 y n + 2 − 2nxy n +1 − n(n − 1) y n − { xy n +1 + ny n } + p 2 y n = 0 { }
ie (1 − x ) y − (2n + 1) xy n +1 − n y n + ny n − ny n + p y n = 0
2 2 2
n+2

∴ (1 − x ) y
2
n+2 ( )
− (2n + 1) xy n +1 + p 2 − n 2 y n = 0 as desired.

2. If sin −1 y = 2 log( x + 1) or
[
y = sin[ 2 log( x + 1)] or y = sin log( x + 1) 2 ]
or y = sin log( x 2 + 2 x + 1) , show that
( x + 1) 2 y n+ 2 + ( 2n + 1)( x + 1) y n+1 + ( n 2 + 4) y n
= 0 (VTU Jan-03)
Out of the above four versions, we consider the function as
sin −1 ( y ) = 2 log( x + 1)
Differentiating w.r.t x, we get

142
1  2 
( y1 ) =   ie ( x + 1) y1 = 2 1 − y 2
1− y 2
 x + 1 
Squaring on both sides
( x + 1) 2 y12 = 4(1 − y 2 )
Again differentiating w.r.t x,
( x + 1) 2 ( 2 y1 y 2 ) + y12 ( 2( x + 1) ) = 4(−2 yy1 )
or ( x + 1) 2 y 2 + ( x + 1) y1 = −4 y (÷2 y1 )
or ( x + 1) 2 y 2 + ( x + 1) y1 + 4 y = 0 -----------*
Differentiating * w.r.t x, n-times, using Leibnitz’s theorem,

 n n(n − 1) n − 2 
n −1
 D y 2 ( x + 1) + nD ( y 2 )2( x + 1) +
2

2!
D ( y 2 )(2) + D n ( g1 )( x + 1) + nD n −1 y1 (1) + 4 D n y = 0 { }
 
On simplification, we get
( x + 1) 2 y n+ 2 + ( 2n + 1)( x + 1) y n+1 + n 2 + 4 y n = 0 ( )
3. If x = tan(log y ) , then find the value of
( )
1 + x 2 y n +1 + ( 2nx − 1) y n + n(n − 1) y n −1 (VTU July-04)
Consider
x = tan(log y )
−1
i.e. tan −1 x = log y or y = e tan x
Differentiating w.r.t x,
−1 1 y
y1 = e tan x . =
1+ x 2
1+ x2
(
∴ 1 + x 2 y1 = y )
ie 1 + x 2 y1 − y = 0 -----------* ( )
We differentiate * n-times using Leibnitz’s theorem,
We get
[(
D n 1 + x 2 y1 − D n ( y ) = 0 ) ]
ie.
{ D ( y1 )(1 + x 2 )+ n C1 D n −1 ( y1 ) D(1 + x 2 ) + n C 2 D n − 2 ( y1 ) D 2 (1 + x 2 ) + .... − D n y = 0
n
} { }
 n(n − 1) 
ie  y n +1 (1 + x ) + ny n (2 x) + y n −1 (2) + 0 + .... − y n = 0
2

 2! 
( )
1 + x y n +1 + ( 2nx − 1) y n + n(n − 1) y n −1 = 0
2

4. If y
1
m
+ y−
1
m
= 2x , [
or y = x + x 2 − 1 or y = x − x 2 − 1 ] m
[ ] m

Show that x − 1 y n + 2
2
( ) ( )
+ (2n + 1) xy n +1 + n 2 − m 2 y n = 0 (VTU Feb-02)
Consider
1
+ y− = 2x ⇒ + = 2x
1 1 1
y m m
y m
1
ym
⇒ (y ) 1
m
2
− 2x y ( ) + 1 = 0 Which is quadratic equation in y
1
m
1
m

143
− (−2 x) ± (−2 x ) 2 − 4(1)(1) 2 x ± 4 x 2 − 4
∴ y 1m = =
2(1) 2

=
2x ± 2 x 2 − 1
2
(
= x ± x2 −1 ⇒ y m = x ± x2 −1
1
) ( )
( )
∴ y = x ± x 2 − 1 . So, we can consider
m

y = [ x + x − 1] y = [ x − x − 1]
m m
or 2 2

Let us take y = [ x + x − 1]
m
2

∴ y = m( x + x − 1 ) 1 +
 1  m −1
1 (2 x) 
2

2 x −1

2

y1 = m x + x 2 − 1  (
m −1 


x 2 − 1 + x 
x 2 − 1 
)

or
( )
x 2 − 1 y1 = my . On squaring
( )
x 2 − 1 y12 = m 2 y 2 . Again differentiating w.r.t x,

(x 2
)
− 1 2 y1 y 2 + y12 (2 x) = m 2 (2 yy1 )
or
(x 2
)
− 1 y 2 + xy1 = m 2 y (÷2 y1 )
or
(x )
− 1 y 2 + xy1 − m 2 y = 0
2
------------(*)
Differentiating (*) n- times using Leibnitz’s theorem and simplifying, we get
( )
x 2 − 1 y n + 2 + (2n + 1) xy n +1 + n 2 − m 2 y n = 0 ( )
Problem set 1.3.1
In each of the following, apply Leibnitz’s theorem to get the results.

d n  log x  (−1) n n!  1 1 1
1. show that n   = n +1  log x − 1 − − − .........  Hint: Take
dx  x  x  2 3 n
1
v = log x; u =
x
2. If y = ( x − 1) , Show that y n satisfies the equation
2 n

(1 − x ) ddx y − 2 x dy
2
2
+ n(n + 1) y = 0 Hint : It is required to show that
2
dx
(1 − x ) y − 2 xy + n( n + 1) y = 0
2
n+2 n +1 n

3. If y = a cos(log x) + b sin(log x ),
x 2 y n + 2 + ( 2n + 1) xy n +1 + n 2 + 1 y n = 0 ( )

144
−1
4. If y = e m sin x , Prove That
( ) ( )
1 − x 2 y n + 2 − ( 2n + 1) xy n +1 − n 2 + m 2 y n = 0
n
 y x
5. If cos −1   = log  , Show that
b n
x y n + 2 + ( 2n + 1) xy n +1 + 2n 2 y n = 0
2

(
6. y = sin m sin −1 x , Pr ove That )
( ) ( )
1 − x 2 y n + 2 − ( 2n + 1) xy n +1 − n 2 + m 2 y n = 0
(
7. If y n = D x log x , Prove That
n n
)
 1 1 1
(i) y n = ny n −1 + (n − 1)! (ii) y n = n! log x + 1 + + + ......... + 
 2 3 n
n!
8. If y = x n log x, Show that y n +1 =
x
• Summary:- The idea of successive differentiation was presented. The computation
of nth derivatives of a few standard functions and relevant problems were discussed.
Also, the concept of successive differention was extended for special type of
functions using Liebnitz’s theorem.

145
Chapter – 2 : POLAR CURVES

LESSON -1 : Angle between Polar Curves

• In this chapter we introduce a new coordinate system, where we can understand


the idea of polar curves and their properties.

• Session-1

2.1.0 Introduction:- We are familiar with Cartesian coordinate system for specifying a
point in the xy – plane. Another useful system for similar purpose is Polar coordinate
system, and the curves specified by these coordinates are referred to as polar curves.

• A polar curve by name “three-leaved rose” is displayed below:

ө=

ө= ө=

ө=π ө=0

ө=

• Any point P can be located on a plane with co-ordinates ( r , θ ) called polar


co-ordinates of P where r = radius vector OP,(with pole ‘O’) ; θ =
projection of OP on the
initial axis OA.(See Fig.1) P(r, ө)

• The equation r = f (θ ) is known as a polar curve. r


• Polar coordinates ( r , θ ) can be related with Cartesian coordinates Ө
( x, y ) through the relations Fig.1. Polar coordinate
O
r = f(ө)
A

x = r cos θ & y = r sin θ . system

146
2.1.1 Important results
• Theorem 1: Angle between the radius vector and the tangent.: Y T

L
dθ φ
i.e. With usual notation prove that tan φ = r
dr P(r, ө)
φ
• Proof:- Let “ φ ” be the angle between the radius vector OPL r
ψ
Ө A
and the tangent TPT 1 at the point `P` on the polar O T
r = f(ө)
curve r = f (θ ) . (See fig.2)
From Fig.2, Fig.2. Angle between radius
ψ = θ +φ vector and the tangent
tan θ + tan φ
tanψ = tan (θ + φ ) =
1 − tan θ tan φ
dy tan θ + tan φ
i.e. = ................. (1)
dx 1 − tan θ tan φ
On the other hand, we have x = r cos θ ; y = r sin θ differentiating these,
w.r.t θ ,
dx  dr  dy  dr 
= r ( − sin θ ) + cos θ   & = r ( cos θ ) + sin θ   //NOTE//
dθ  dθ  dθ  dθ 
 dr 
dy r ( cos θ ) + sin θ  
dy d θ  dθ  dr
= = dividing the Nr & Dr by cos θ
dx dx  dr  dθ
dθ r ( − sin θ ) + cos θ  
 dθ 
dy
=
( r dθ
dr
)
+ tan θ
dx − ( rdθ dr ) tan θ + 1
(
dy tan θ + r dr
dθ )
i.e. =
dx 1 − tan θ rdθ ( )
dr
………………….(2)

Comparing equations (1) and (2)


we get tan φ = r dθ dr
 1 dr 
• Note that cot φ =  
 r dθ 
• A Note on Angle of intersection of two polar curves:-
If φ1 and φ 2 are the angles between the common radius vector and the tangents
at the point of intersection of two curves r = f 1 (θ ) and r = f 2 (θ ) then the
angle intersection of the curves is given by φ1 − φ2

147
• Theorem 2: The length “p” of perpendicular from pole to the tangent in a
polar curve i.e.
1 1 1  dr  2
(i) p = r sin φ or (ii) = 2 = 4 
p 2
r r  dθ 
• Proof:- In the Fig.3, note that ON = p, the length of the perpendicular from the
pole to
the tangent at p on r = f (θ ) .from the right angled triangle OPN,
ON
sin φ = ⇒ ON = ( OP ) sin φ
OP

φ
i.e. p = r sin φ ..............(i )
1 1 1 P(r, ө)
Consider = = cos ecφ r φ
p r sin φ r Ψ
O Ө
1 1 1
∴ 2 = 2 cos ec 2φ = 2 (1 + cot 2φ ) r = f (ө)
p r r
P
Fig.3 Length of the perpendicular N
from the pole to the tangent
1   1 dr  
2
1
= 1 +   
p 2 r 2   r dθ  

2
1 1 1  dr 
∴ 2 = 2 + 4  ............(ii )
p r r  dθ 
2
1 1  du 
• Note:-If u = , we get 2 = u 2 +  
r p  dθ 
• Session-2
• In this session, we solve few problems on angle of intersection of polar curves
and pedal equations.
2.1.2 Worked examples:-
• Find the acute angle between the following polar curves
1. r = a (1 + cos θ ) and r = b(1 − cos θ ) (VTU-July-2003)

2 r = ( sin θ + cos θ ) and r = 2 sin θ (VTU-July-2004)

3. ( 2 ) and r = 25 cos ec (θ 2 )
r = 16 sec 2 θ 2

4. r = a log θ and r = a log θ (VTU-July-2005)


aθ a
5. r = and r =
1+θ 1+θ 2

148
Solutions:

1. Consider Consider
r = a (1 + cos θ ) r = b(1 − cos θ )
Diff w.r.t θ Diff w.r.t θ

dr dr
= − a sin θ = b sin θ
dθ dθ
dθ a (1 + cos θ ) dθ b(1 − cos θ )
r = r =
dr − a sin θ dr b sin θ

tan φ1 = −
2 cos( )
2 θ
2 tan φ1 = −
2 sin 2 θ
2
( )
2 sin ( ) ( )
θ
2
cos θ
2
2 sin θ
2
cos θ ( ) ( )
2
= − cot θ = tan θ
2 2
( ) (
i.e tan φ1 = tan π 2 + θ 2 ⇒ φ1 = π 2 + θ 2 ) tan φ1 = tan θ ⇒ φ1 = φ2
2
Angle between the curves
( )
φ1 − φ2 = π 2 + θ 2 − θ 2 = π 2
Hence ,the given curves intersect orthogonally

2. Consider Consider
r = ( sin θ + cos θ ) r = 2 sin θ
Diff w.r.t θ Diff w.r.t θ
dr dr
= cos θ − sin θ = 2 cos θ
dθ dθ
dθ sin θ + cos θ dθ 2 sin θ
r = r =
dr cos θ − sin θ dr 2 cos θ
tan θ + 1
tan φ1 = (÷ Nr & Dr cos θ ) tan φ2 = tan θ
1 − tan θ
i.e tan φ1 =
tan θ + 1
1 − tan θ
(
= tan π + θ
4
) ⇒ φ2 = θ

⇒ φ1 = π + θ
4
∴ Angle between the curves = φ1 − φ2 = π ( 4
)
+θ −θ = π
4

3. Consider Consider
r = 16 sec 2 θ ( 2) ( 2)
r = 25 cos ec 2 θ
Diff w.r.t θ Diff w.r.t θ

149
dr

( ) ( )
= 32 sec 2 θ tan θ . 1
2 2 2
dr

( ) ( )
= −50 cos ec 2 θ cot θ . 1
2 2 2
( ) ( )
= 16 sec θ tan θ
2 2
= −25 cos ec 2 θ
( ) ( )
2
cot θ
2
dθ 16 sec ( )
2 θ
2 dθ ( )
25 cos ec 2 θ
2
r =
dr 16 sec 2 θ
( ) ( )
2
tan θ
2
r =
dr − 25 cos ec 2 θ
( ) ( )
2
cot θ
2
tan φ1 = cot θ
2
= tan( π
2
)−θ
2
tan φ2 = − tan θ
2
(
= tan − θ
2
)
⇒ φ1 = π ( 2
−θ
2
) ⇒ φ2 = −θ
2
( ) ( )
Angle of intersection of the curves = φ1 − φ2 = π 2 − θ 2 − θ 2

2

4. Consider Consider
r = a log θ r= a
log θ
Diff w.r.t θ Diff w.r.t θ
dr a dr
= − a ( log θ ) . 1
2
=
dθ θ dθ θ
 ( log θ ) 2 θ 
r

dr
= a log θ θ ( )
a
r

dr
= − a 

 log θ  a



tan φ1 = θ log θ ..........(i ) tan φ 2 = −θ log θ ..........(ii )
We know that
tan φ1 − tan φ 2
tan ( φ1 − φ 2 ) =
1 + tan φ1 tan φ 2

θ log θ − ( − θ log θ )
=
1 + (ϑ log θ )( − θ log θ )
2θ log θ
i.e tan ( φ1 − φ 2 ) = ............(iii )
1 − (θ log θ )
2

From the data: a log θ = r = a log θ ⇒ ( log θ ) = 1 or log θ = ±1


2

As θ is acute, we take by θ =1 ⇒ θ = e NOTE


Substituting θ = e in (iii), we get

150
2e log e  2e 
tan ( φ1 − φ 2 ) =
1 − ( e log e )
2
= 2 
1− e 
( log ee = 1)

 2e 
∴ φ1 − φ 2 = tan −1  2 
1− e 

5. Consider Consider
aθ aθ
r= as r=
1+θ 1+θ 2
1 1+θ 1 1
r
=

=
a θ
+1 ( ) ∴ (1 + θ 2 ) = a
r
Diff w.r.t θ Diff w.r.t θ
− 2
1 dr 1 1
r dθ a
= −
θ2
( ) 2θ = − a
dr
r 2 dθ
1 dr r − 2 rθ 1 dr
= =
r dθ aθ 2 a r dθ
dθ aθ 2 dθ − a
r = i.e r =
dr r dr 2rθ

aθ 2 a 1+θ 2 
tan φ1 = tan φ2 = −  
aθ 2θ
(1 + θ )  a 

∴ tan φ1 = θ (1 + θ ) tan φ2 = −
1

(1+θ 2 )
Now, we have

⇒ aθ (1 + θ 2 ) = a (1 + θ )
a
=r=
1+θ 1+θ 2

or θ + θ 3 = 1 + θ ⇒ θ 3 = 1 or θ = 1
∴ tan φ1 = 2 & tan φ2 = ( − 1)
tan φ1 − tan φ2
Consider tan ( φ1 − φ2 ) =
1 + ( tan φ1 )( tan φ2 )
2 − ( − 1)
= = −3 = 3
1 + ( 2 )( − 1)
∴ φ1 − φ2 = tan −1 ( 3)
Problem Set No. 2.1.1 for practice.

• Find the acute angle between the curves


1. r n = a n ( cos nθ + sin nθ ) and r n = a n sin nθ (ans: π 4 )

2. r n cos nθ = a n and r n sin nθ = b n (ans: π 2 )

151
3. r = aθ and r = a θ (ans: π 2 )

4. r = a cos θ and r = a 2 (ans:5 π 6 )

5. r m = a m cos mθ and r m = b m sin mθ (ans: π 2 )


LESSON -2 : Pedal Equations
• Session - 1
• 2.2.0 Pedal equations (p-r equations):- Any equation containing only p & r
is
known as pedal equation of a polar curve.

• Working rules to find pedal equations:-

(i) Eliminate r and φ from the Eqs.: (i) r = f (θ ) & p = r sin φ


2
1 1 1  dr 
(ii) Eliminate only θ from the Eqs.: (i) r = f (θ ) & ∴ = 2 + 4 
p 2
r r  dθ 
• 2.2.1 Worked Examples on pedal equations:-

• Find the pedal equations for the polar curves:-


2a
1. = 1 − cos θ
r
2. r = e θc cot α
3. r m = a m sin mθ + b m cos mθ (VTU-Jan-2005)
4. l r = 1 + e cos θ
Solutions:
2a
1. Consider = 1 − cos θ ……….(i)
r
Diff. w.r.t θ
( )
2a − 1 2
r dθ
dr
= sin θ
1 dr − r sin θ
=
r dθ 2a
dθ 2a 1
r =−
dr r sin θ
( 1 − cosθ ) 2 sin 2 θ
tan φ = −
sin θ
=−
2 sin θ
2
cosθ
( )
= − tan θ
2
2 2
( )
tan φ = tan − θ ⇒ φ = − θ
2 2
Using the value of φ is p = r sin φ , we get
( )
p = r sin − θ = − r sin θ .............(ii )
2 2

152
Eliminating “ θ ” between (i) and (ii)
 1 − cosθ  r  2a 
2
p 2 = r 2 sin 2 θ = r 2   =   [See eg: - (i)]
2  2  2 r 
p 2 = ar.
This eqn. is only in terms of p and r and hence it is the pedal equation of the polar
curve.
2. Consider r = eθ cot α
Diff. w.r.t θ
= eθ cot α ( cot α ) = r cot α ( r = eθ cot α )
dr

We use the equation
2
1 1 1  dr 
= +  
p 2 r 2 r 4  dθ 
1 1
= 2 + 4 ( r cot α )
2

r r
= 2 + 4 ( cot 2 α ) = 2 (1 + cot 2 α ) = 2 cos ec 2α
1 1 1 1
r r r r
1 1
2
= 2 cos ec 2α
p r
2
p2 = r or r 2 = p 2 cosec 2α is the required pedal equation
cosec 2α

3.Consider r m = a m sin mθ + b m cos mθ


Diff. w.r.t θ
dr
mr m −1 = a m ( m cos mθ ) + b m ( −m sin mθ )

r m dr
= a m cos mθ − b m sin mθ
r dθ
1 dr a m cos mθ −b m sin mθ
= m
r dθ a sin mθ +b m cos mθ

a m cos mθ − b m sin mθ
cot φ =
a m sin mθ + b m cos mθ
1 1
Consider p = r sin φ , = cos ecφ
p r
1 1
2
= 2 cos ec 2φ
p r

= 2 (1 + cot 2 φ )
1
r

153
1   a m cos mθ − b m sin mθ  
2

= 2 1 +  m  
r   a sin mθ + b m cos mθ  
 
1  ( a m sin mθ + bm cos mθ ) + ( a m cos mθ − bm sin mθ ) 
2 2

= 2 
r 
 ( a m
sin m θ + b m
cos m θ ) 2


1 1  a 2m + b 2m 
=   Note
p 2 r 2  r 2m 
r 2 ( m + 1)
⇒ p2 = 2m is the required p-r equation
a + b2m

4. Consider l r = (1 + cosθ )
Diff w.r.t θ
 1 dr   dr 
l − 2  = − e sin θ ⇒ l r  1 r  = e sin θ
 r dθ   dθ 
l ( cot φ ) = e sin θ
r
( )
∴ cot φ = r e sin θ
l

We have 2 = 2 (1 + cot φ ) (see eg: 3 above)


1 1 2
Now
p r
1 1  l 2 + e 2 r 2 sin 2 θ 
=
p 2 r 2  
l2 
1
( 2 2
= 2 1 + e r 2 sin 2 θ
r l
) 1 + e cosθ = l
r

l−r
e cosθ =
r
 2   l − r  2  
2 2
 l + e r 1 −   
1 1   re    l −r
= cosθ =  
p2 r2  l2 
 re 
 
 
 
sin 2 θ = 1 − cos 2 θ
1  e2 − 1  2 l −r
2

On simplification 2 =  2  + = 1−  
p  e  lr  re 

• Problem Set No. 2.2.1 for practice.

Find the pedal equations of the following polar curves

154
1. r n cos nθ = a n and r n sin nθ = b n
2. r = aθ and r = a θ

3. r = a cos θ and r = a 2
4 . r m = a m cos mθ and r m = b m sin mθ
Chapter-3: PARTIAL DIFFERENTIATION
Lesson-1: Euler’s Theorem
Session-1

3.1.0 Introduction: We often come across qualities which depend on two or more
variables. For e.g. the area of a rectangle of length x and breadth y is given by
Area = A(x,y) = xy. The area A(x, y) is, obliviously, a function of two variables.
Similarly, the distances of the point (x, y, z) from the origin in three-dimensional space is
an example of a function of three variables x, y, z.

• 3.1.1 Partial derivatives: Let z = f(x, y) be a function of two variables x and y.


∂z ∂f
The first order partial derivative of z w.r.t. x, denoted by or or zx or fx is defined
∂x ∂x
∂z f ( x + δx, y ) − f ( x, y )
as = lim
∂x δx →0 δx
 ∂z 
• From the above definition, we understand that   is the ordinary derivative of z
 ∂x 
w.r.t x, treating y as constant.
∂z ∂f
The first order partial derivative of z w.r.t y, denoted by or or zy or fy is defined as
∂y ∂y
∂z f ( x, y + δy ) − f ( x, y )
= lim
∂y δy →0 δy
 ∂z 
• From the above definition, we understand that   is the ordinary derivative of z
 ∂y 
w.r.t y, treating x as constant
∂  ∂z  ∂ 2 z ∂2 f
• The partial derivatives   = or or zxx or fxx;
∂x  ∂x  ∂x 2 ∂x 2
∂  ∂z  ∂ 2 z ∂2 f
  = 2 or or zyy or fyy;
∂y  ∂y  ∂y ∂y 2
∂  ∂z  ∂ 2 z
 = or zyx or fyx
∂y  ∂y  ∂x∂y
∂  ∂z  ∂ 2 z
and = or zxy or fxy
∂y  ∂y  ∂x∂y
are known as second order Partial derivatives.

In all ordinary cases, it can be verified that

155
∂2z ∂2z
=
∂x∂y ∂y∂x
• The third and higher order partial derivatives of f(x,y) are defined in an analogous
way Also, the second and higher order partial derivatives of more than two
independent variables are defined similarly.

• A note on rules of partial differentiation:-


All the rules of differentiation applicable to functions of a single independent
variable are applicable for partial differentiation also; the only difference is that
while differentiating partially w.r.t one independent variable all other independent
variables are treated as constants.

• 3.1.2 Worked examples: -

In solving the following examples, we learn the partial differentiation


of so called explicit functions of more than one variables.

( )
1. Evaluate ∂z ∂x and ( ∂x ∂y ) , if
z = x 2 y − x sin xy
(a)
x + y + z = log z
(b)

Solution: - (a) Consider z = x y − x sin xy Differentiating z w.r.t x, keeping y as a


2

constant, we get the partial derivative,


∂z
= ( 2 x ) y − { x cos( xy )( y ) + sin ( xy )(1) }
∂x
z x = 2 xy − xy cos xy − sin xy = xy ( 2 − cos xy ) − sin xy
i.e.
Similarly, differentiating z w.r.t y keeping x as a constant, we get the partial derivative
∂z
= x 2 (1) − { xcoxy ( x ) + sin ( xy )( 0 ) }
∂y
z y = x 2 − x 2 cos xy = x 2 (1 − cos xy )
i.e

(b) Consider x + y + z = log z


log z − z = x + y − − − − − * Differentiating * partially w.r.t x treating y as
i.e.

1 ∂z ∂z  ∂z  1 
− = 1 + 0 ⇒   − 1 = 1
Constant, z ∂x ∂x  ∂x  z 
∂z  1 − z  ∂z  z 
  =1 = 
i.e.
∂x  z  or
∂x  1 − z 

Similarly, differentiating * partially w.r.t y, treating x as constant,

156
1 ∂z ∂z  ∂z  1 
− = 1 + 0 or   − 1 = 1
z ∂x ∂x  ∂x  z 
∂z  z 
= 
⇒ ∂x  1 − z 
∂z ∂z  z 
∴ = = 
∂x ∂y  1 − z 
1 ∂  2 ∂θ  ∂θ

r2
r = ?
2. .If θ = t n
e 4t
, what value of n will make
r 2
∂r  ∂r  ∂t
r2

Solution: Consider θ = t e ------------------------------------------------- (*)
n 4t

Differentiating * w.r.t. r, treating t as constant, we get


r2 −r 2
∂θ −  − 2 r  − r −
= t e 4t  =
n n 1
t e 4t
∂r  4t  2
−r 2
∂θ r3
∴r = − t n −1e
2 4t
∂r 2 and hence
∂  2 ∂θ  − 3r n −1 − r 2 4t  r 3  n −1 − r 2 4t  2r 
2
r = t e  − t e − 
∂r  ∂r  2  2  4t 
1 ∂  2 ∂θ   3 n −1 r 2 n −2  −r 2 4t
∴ r  =  − t + t e
r 2 ∂r  ∂r   2 4  ------------------- (1)
Also, differentiating (*) w.r.t t, treating r as constant, we get
2
r
∂θ −r 2  r 
− 2
= nt n −1 e 4t + t n e 4t  2 
∂t  4t 
 1  −r 2
=  nt n −1 + r 2 t n − 2 e 4t
 4  ------------------------ (2)

1 ∂  2 ∂θ  ∂θ
r = ,
Since, r 2
∂ r  ∂r  ∂ t equation (1) and (2) yield,
−r 2
 − 3 n −1 r n −2  4t  n −1 r 2 n − 2  −r 2 4t
2
 t + t e =  nt + t e
2 4 4
   

i.e.
(n + 3 2 ) t n −1
e
−r 2
4t
=0⇒ n+ 3 ( 2
) = 0 or n = − 3 2
∂z ∂z
+a = 2abz b
3. If z = e f ( ax − by ) , prove that ∂x
ax + by
∂y (VTU-Jan-2004)
Solution: Consider z = e
ax + by
f ( ax − by ) ,
------------------- (*)
Differentiating (*) w.r.t x using product and chain rules, we get
∂z
∂y
{ } {
= e ax +by f 1 ( ax − by )( − b ) + f ( ax − by ) e ax +by ( b ) }

157
∂z
= be ax +by { f ( ax − by ) − f 1 ( ax − by ) }
∂y ------------------- (2)
Multiplying eq (1) by b and eq(2) by a, and adding we get

b
∂z
∂x
+a
∂z
∂y
[
= abe ax +by f 1 ( ax + by ) + f ( ax − by ) + f ( ax − by ) − f 1 ( ax − by ) ]
= abe ax +by [ 2 f ( ax − by ) ]

[
= 2ab e ax +by f ( ax − by ) ]
= 2abz ,as desired
∂u 1 ∂v
=
4.Given u = e
r cosθ
cos( r sin θ ) , v = e r cosθ
sin ( r sin θ ) prove that ∂r r ∂θ and
,
∂v 1 ∂u
=−
∂r r ∂θ
Solution: Consider u = e
r cosθ
cos( r sin θ ) , Differentiating u w.r.t. r and θ partially, we
get,
∂u
= e r cosθ { − sin ( r sin θ )( sin θ )} + cos( r sin θ ) {e r cosθ ( cos θ ) }
∂r
∂u r cosθ
e {cos( r sin θ ) cos θ − sin( r sin θ ) sin θ }
i.e. ∂r
∂u
= e r cosθ {cos( r sin θ + θ ) }
or
∂r --------------(1)
∂u
= e r cosθ { − sin ( r sin θ )( r cos θ ) } + cos( r sin θ ) {e r cosθ ( − r sin θ ) }
and ∂θ
= − re r cosθ {sin ( r sin θ ) cos θ + cos( r sin θ ) sin θ }
1 ∂u
− = e r cosθ {sin ( r sin θ + θ ) }
i.e. r ∂ θ ---------------------(2)
Next consider,
v = e r cos θ sin(r sin θ )
Differentiating v w.r.t r & θ , Partially we get
∂v
∂r
{
= e r cos θ { cos( r sin θ ) sin θ } + sin ( r sin θ ) e r cos θ ( cos θ )}
= e r cosθ {sin ( r sin θ ) cos θ + cos( r sin θ ) sin θ }
∂v
= e r cosθ {sin ( r sin θ + θ ) }
i.e ∂r --------------------(3);
∂v
= e r cosθ {cos( r sin θ )( r cos θ ) } + sin ( r sin θ ) {e r cosθ ( − r sin θ ) }
∂θ
= re r cosθ {cos( r sin θ ) cos θ − sin ( r sin θ ) sin θ }

158
1 ∂v
= e r cos θ { cos( r sin θ + θ )}
i.e. r ∂θ ----------------------------- (4)
∂u 1 ∂v
Thus, from eqs (1) and (4), we obtain = , and from eqs (2) and (3) we obtain
∂r r ∂θ
∂v 1 ∂u
=− .
∂r r ∂θ

Session-2
• 3.1.3 Worked Examples:- In each of following examples, we try to
∂ 2u ∂ 2u
show that = , where u = f ( x, y ) is a function of two variables.
∂x∂y ∂y∂x
1. u = x y ; 2. u = e x ( x sin y − y sin y )

3. u = sin  x 
−1 y

 
Solution: - 1. Differentiating u = x y partially w.r.t y, we get
∂u
∂y
= x y log x
 d x
 ( a ) = a x log x 

 dx 
differentiating the above partially w.r.t x, we get
∂  ∂u  ∂ y
 =
∂x  ∂y  ∂x
( x log x )
∂ 2u
= x y   + log x ( yx y −1 )
1
i.e ∂x∂y  x
∂ 2u
= x y −1 (1 + y log x )
or ∂x∂y ---------------(1)
∂u
Next, differentiating u = x y partially w.r.t. x, we get = yx y −1 so that
∂x
∂  ∂u  ∂
 =
∂y  ∂x  ∂y
( yx y −1 )

∂ 2u
= y ( x y −1 log x ) + x y −1 (1)
i.e ∂y∂x
∂2u
= x y −1 (1 + y log x )
or
∂y∂x ---------------(2)

159
∂ 2u ∂ 2u
=
from (1) and (2), we get ∂x∂y ∂y∂x

2. Differentiating u = sin  x  partially w.r.t. x, we get


−1 y

 
∂u 1  − y  = −y
= 2  2
∂x  x  x x2 − y2
1 −  y 
 x
   
x x 2 − y 2 (1) − y  x  1 ( − 2 y ) 
  2 x − y 
2 2
∂  ∂u  
∴  =−
∂y  ∂x  x2 (x2 − y2 )
∂ 2u  x ( x 2 − y 2 ) + xy 2  x
= −  =−
∂y∂x  x 2 ( x 2 − y 2 ) 2  ( )
3 3

i.e. x 2
− y 2 2
----------------(1)

Next, differentiating u = sin  x  partially w.r.t. y, we get


−1 y

 
∂u
∂y
=
1
2
( )
1 =
x
1
x2 − y2
1 −  y 
 x
1
x 2 − y 2 ( 0) − (1) ( 2x)
∂  ∂u  2 x − y2
2
∴   = −
∂x  ∂y  x2 − y2
∂ 2u x
=−
i.e.
∂x∂y ( x 2 − y 2 ) 32 ------------- (2)

∂ u ∂ 2u 2
=
From (1) and (2), we get ∂x∂y ∂y∂x

3. Differentiating u = e ( x sin y − y sin y ) w.r.t y, we get


x

∂u
= e x ( x cos y − y cos y − sin y )
∂y
Differentiating w.r.t x, we get
∂  ∂u 
  = e x ( − x sin y + cos y − 0 − 0) + ( x cos y − y cos y − sin y ) e x
∂x  ∂y 
∂ 2u
= e x ( − x sin y + cos y + x cos y − y cos y − sin y )
i.e. ∂x ∂y
∂ 2u
= e x { (1 + x − y ) cos y − (1 + x ) sin y}
Or ∂x∂y ----------------- (1)

160
Next, differentiating w.r.t x, we get
∂u
= e x { x cos y + sin y} + ( x sin y − y sin y ) e x
∂x
∂u
= e x { x cos y + (1 + x − y ) sin y}
∂x
Differentiating this w.r.t. y, we get
∂  ∂u 
  = e { x ( − sin y ) + 0 + (1 + x − y ) cos y + sin y ( − 1) }
x

∂y  ∂x 
= e x { (1 + x − y ) cos y − (1 + x ) sin y}
----------------- (2)
from (1)and (2),we get
∂ 2u ∂ 2u
=
∂x∂y ∂y∂x
3.1.4 In the following few examples involve equations where partial derivatives of
higher order occur. These equations frequently appear is engineering applications.
−1  2 xy  ∂ 2u ∂ 2u
1. If u = tan  2 
2  , prove that + =0
x −y  ∂x 2 ∂y 2
−1  2 xy 
Solution:- Differentiating u = tan  2 
2 
x −y 
partially w.r.t.x, we get
∂u 1  ( x 2 − y 2 ) ( 2 y ) − 2 xy ( 2 x ) 
= 2  
∂x  2 xy  
1 +  2 
( x2 − y2 )
2

2 
x −y 
∂u − 2y
= 2
i.e. ∂x ( x + y ) Differentiating Partially this w.r.t. x, we get
2

∂ 2 u ∂  − 2 y  ( x 2 − y 2 ) ( 0) − ( 2 y )( 2 x )
=  =
∂x 2 ∂x  x 2 + y 2  ( x2 + y 2 )2
∂ 2u 4 xy
=
i.e.
∂x 2
( x 2
+ y )
2 2
-------------------------- (1)

−1  2 xy 
Next differentiating u = tan  2 
2 
x −y 
partially w.r.t. y, we get
∂u 1  ( x 2 − y 2 ) ( 2 x ) − 2 xy ( − 2 y ) 
= 2  
∂y  2 xy  
1 +  2 
( x2 − y2 )
2

2 
x −y 
∂u 2x
= 2
∂y ( x + y 2 ) Differentiating partially this w.r.t.y, we get

161
∂ 2 u ∂  2 x  ( x 2 − y 2 ) ( 0) − ( 2 x )( 2 y )
=  =
∂x 2 ∂x  x 2 + y 2  ( x2 + y 2 )2
∂ 2u − 4 xy
=
i.e.
∂y 2
( x 2
+ y )
2 2
-----------------------(2)

∂ u ∂ 2u 2
Adding eqs (1) and (2) we get + = 0 , as desired
∂x 2 ∂y 2

Note:
∂ 2u ∂ 2u
(a) The equation + = 0 is known as Laplace’s equation in two dimension
∂x 2 ∂y 2
which has variety of applications in potential theory.

∂ 2u ∂ 2u ∂ 2u
(b) A similar equation viz, + + = 0 is known as Laplace’s equation in
∂x 2 ∂y 2 ∂z 2
three- dimensions where u = u ( x, y , z )
∂ 2u 2 ∂ u
2
2. If u = f ( x + ay ) + g ( x − ay ) , show that = a
∂y 2 ∂x 2
Solution:- Differentiating u = f ( x + ay ) + g ( x − ay ) , partially w.r.t.x, we get
∂u
= f 1 ( x + ay )(1) + g 1 ( x − ay )(1)
∂x
Again differentiating partially w.r.t.x, we see that
∂ 2u
= f 11 ( x + ay )(1) + g 11 ( x − ay )(1)
∂x 2
∂ 2u
[ ]
a 2 2 = a 2 f 11 ( x + ay ) + g 11 ( x − ay ) -----------(1)
∂x
Next, differentiating u = f ( x + ay ) + g ( x − ay ) , partially w.r.t.y
∂u
∂y
( )
= f 1 ( x + ay ) a 2 + g .1 ( x − ay ) a 2 ( )
Again differentiating partially w.r.t.y, we see that
∂ 2u
= f 11 ( x + ay ) ( a 2 ) + g 11 ( x − ay ) ( a 2 )
∂x 2

∂ 2u
∂y
[2 11 11
]
i.e 2 = a f ( x + ay ) + g ( x − ay ) ------------------(2)

162
from eqs (1) and (2), we see that
∂ 2u 2 ∂ u
2
= a
∂y 2 ∂x 2
∂ 2u 2 ∂ u
2
∂ 2u 2 ∂ u
2
Note:- (a). The equation = a or the equation = a is known as
∂y 2 ∂x 2 ∂t 2 ∂x 2
one-dimensional wave equation
∂ 2u 2 ∂ u ∂ 2u ∂ 2u 
2

(b). A similar equation viz, 2 = a 


 ∂x 2 ∂y 2 + ∂z 2 
+
∂t  
is known as three- dimensional wave equation

∂u ∂ 2u
3. If u = e −2 t cos 3x , find the value of ‘c’ such that = c2 2
∂t ∂x
−2 t
Solution:- Differentiating u = e cos 3x , partially w.r.t. t we get
∂u
= −2e −2 t cos 3x --------------- (1)
∂t
Next, differentiating u = e −2 t cos 3x partially w.r.t.x,
∂u
= −3e −2 t sin 3x
∂x
Again differentiating partially w.r.t x,
∂ 2u −2 t
= − 9 e cos 3 x
∂x 2

∂u
∂t
∂ 2u
∂x
( )
= c 2 2 ⇒ −2e −2 t cos 3x = 9e −2 t cos 3x ⇒ c 2 = 9 or c = 9 2
2

∂u ∂ 2u
Note (a) The equation = c 2 2 is called one-dimensional heat equation
∂t ∂x
∂u 2 ∂ u ∂ 2u ∂ 2u 
2

(b) Similarly, the equation viz, = c  2 + 2 + 2  is known as



∂t  ∂x ∂y ∂z 
three-dimensional heat equation

4. If u = log( x 3 + y 3 + z 3 − 3xyz ) , prove that


∂u ∂u ∂u  3 
(a) + + =  
∂x ∂y ∂z  x + y + z 
2
∂ ∂ ∂ 9
(b)  + +  u = − (VTU Feb-2005)
 ∂x ∂y ∂z  ( x + y + z) 2
Solution:- (a) Differentiating u partially w.r.t. x,y,z we get respect or

163
∂u 3x 2 − 3 yz
= 3
∂x x + y 3 + z 3 − 3 xyz
∂u 3 y 2 − 3zx
=
∂y x 3 + y 3 + z 3 − 3xyz
∂u 3z 2 − 3xy
= 3
And ∂z x + y + z − 3xyz adding these partial derivatives we get
3 3

∂u ∂u ∂u 3( x 2 + y 2 + z 2 − xy − yz − zx ) 3( x 2 + y 2 + z 2 − xy − yz − zx )
+ + = =
∂x ∂y ∂z x 3 + y 3 + z 3 − 3xyz ( x + y + z ) ( x 2 + y 2 + z 2 − xy − yz − zx )

 3 
=  
x+ y+z

(b) By definition, we have


2
∂ ∂ ∂ ∂ ∂ ∂ ∂ ∂ ∂
 + +  u =  + +   + +  u
 ∂x ∂y ∂z   ∂x ∂y ∂z   ∂x ∂y ∂z 

∂ ∂ ∂  ∂u ∂u ∂u 
=  + +   + + 
 ∂x ∂y ∂z   ∂x ∂y ∂z 

∂ ∂ ∂  3 
=  + +   
 ∂x ∂y ∂z   x + y + z 
∂  3  ∂  3  ∂  3 
  +   +  
∂x  x + y + z  ∂y  x + y + z  ∂z  x + y + z 
=

 −3   −3   −3  9
=   
2 +
 
2 +
 −
2 =
 ( x + y + z)   ( x + y + z)   ( x + y + z)  ( x + y + z)2

∂ 2u ∂ 2u ∂ 2u 2
5. If u = f ( r ) , where r = x 2 + y 2 + z 2 , show that + 2 + 2 = f 11 ( r ) + f 1 ( r )
∂x 2
∂y ∂z r
Solution:- By data r = x + y + z Differentiating this partially w.r.t x, we get
2 2 2 2

∂r ∂r  x  ∂r  y  ∂r  z 
2 r = 2 x or =   Similarly, we get =   and = 
∂x ∂x  r  ∂y  r  ∂z  r 
Now, differentiating u = f ( r ) , partially w.r.t.x,
∂u ∂r ∂r  x 
= f 1( r) As =   , we get
∂x ∂x ∂x  r 
∂u  x ∂u  f 1 ( r) 
= f 1 ( r )   Differentiating = x  partially w.r.t.x, we get
∂x r ∂x  r 

164
∂ 2u  1 11 ∂r  1  ∂r  f ( r )
1
= x  f ( r ) + f 1
( r )  −   + (1)
∂x 2 r ∂x  r  ∂x 
2
r
∂ 2u  1 11  x   1  x  f ( r )
1

i.e. 2 = x  f ( r )   + f ( r )  − 2   +
1

∂x r r  r  r  r
∂ 2u  x  f ( r ) f ( r )  f ( r )
11 1 1

or = x    −  +
∂x 2  r  r r2  r

∂2 u x 2 11 f 1(r)  x2 
= f ( r ) + 1 − 
…………. (1)

∂x 2 r2 r  r2 
Similarly, we get

(r ) + f (r ) 1 − y2 ....(2 ) and
∂2u y2 1
 2

= 2 f 11

∂y 2
r r  r 

∂2 u z 2 11 f 1 (r)  z2 
= f ( r ) + 1 − .....( 3)
∂z 2 r2 r  r2 
Adding eqs (1) ,(2) and (3), we get
∂ 2u ∂ 2u ∂ 2u f 11 ( r ) 2 f 1( r )  x2 + y2 + z 2 
+ +
∂x 2 ∂y 2 ∂z 2
=
r2
( x + y 2
+ z 2
) +
r 
3 −
r2


f 1( r )
= f 11 ( r ) + { 3 − 1}
r
f 1(r)
= f (r) + 2
11

Problem Set No.3.1.1 for practice

∂2 z ∂2z
1. Verify = where
∂x∂y ∂y∂x
( )
(i) z = x 3 + y 3 − 3axy (ii) z = tan −1 x 2 + y 2 (iii) z = log( x sin y + y sin x )
−1  y 
2. If u = x tan  x  − y tan x y , prove that
2 2 −1

 
∂u2
x −y
2 2
∂ 2u ∂  ∂ 
= 2 Hint =  
∂y∂x x + y 2
∂y∂x ∂y  ∂x 
2
x2 + y2  ∂z ∂z   ∂z ∂z 
3. If, z = , show that  −  = 41 − −  (VTU-Jan-2005)
x +y  ∂x ∂y   ∂x ∂y 

165
∂u ∂u ∂u
4. If u = ( x − y )( y − z )( z − x ) , show that + + =0
∂x ∂y ∂z
∂ 2 z ∂ 2 z 1  ∂z   ∂z  
2 2

5. If z = e , prove that
xy + =   +   
∂x 2 ∂y 2 z  ∂x   ∂y 2  
 
y z ∂u ∂u ∂u
6. If u = + , prove that x + y +z =0
z x ∂x ∂y ∂z
7. If x = r cosθ , y = r sin θ , show that
∂ 2 r ∂ 2 r 1  ∂r   ∂r  
2 2

+ 2 =   +    (Hint: By data x 2 + y 2 = r 2 )
∂x 2
∂y r  ∂x   ∂y  
 

Session –3
• 3.1.5: Introduction to homogenous functions:-
n −1 n−2 2
An expression of the form a 0 x + a1 x y + a 2 x y + ............ + a n y
n n

In which every term is of the nth degree, is called homogenous function of order n. This
can be rewritten as
 2 n

x n a 0 + a1  y  + a 2  y  + ............. + a n  y  
 x   x   x  

= x nφ  y  .
 x
n y 
Thus any function f ( x, y ) which can be expressed in the below x φ  x  , is called
 
homogenous function of order n (or derivative n) in x and y

3 
2 3

For example: (i) f ( x, y ) = x + x y − xy + y = x 1 +  x  −  x  −  x  
3 2 2 3 y y y
       

= x nφ  y  is a homogenous function of degree 3.


 x
x1 + y  1
x+ y  x 2 y
(ii) f ( x , y ) = = = x φ  x  is a homogenous function
x+ y    
x 1 + y 
 x
of degree ½
y  y  0 y 
(iii) f ( x, y ) = sin  x  = x sin  x  = x φ  x  is a homogenous function of degree 0
0

     
n  
Note: - (1) Any function, which can be expressed as, y φ  x y  is also a homogenous
 
function of degree n.(or order n)
(2) In general, a function f ( x, y, z, t ,.........) is said to be a homogenous function

166
of order n in x, y, z, t… if it can be expressed as
x nφ  y , z , t ,................ 
 x x x 
3.1.6: Euler’s Theorem:-
The following theorem, named in honour of the great Swiss mathematician
L. Euler (1707-1783) gives a very useful formula for a particular combination of
partial derivatives of homogenous functions.

• Statement of Euler’s theorem:- If u is a homogenous function of x and y with


∂u ∂u
degree n, then x +y = nu (VTU Feb-2005)
∂x ∂y
Proof: - Since u is a homogenous function of degree n, we can put u is the form
u = x nφ  y  .
 x
Differentiating this w.r.t x partially, we get
∂u
∂x  x  x   x
{
= x nφ 1  y  − y 2  + φ  y  nx n −1 }
∂u
or = − x n −2 yφ 1  y  + nx n −1φ  y  ----------(1)
∂x  x  x
n y 
Again differentiating u = x φ  x  partially w.r.t y, we get
 
∂u
∂y
( )
= x nφ 1  y  1 + 0
 x x
∂u
= x n −1φ 1  y  …………. (2)
∂y  x
Consider
∂u ∂u
x +y = x − x n − 2 yφ 1  y  + nx n −1φ  y  + y  x n −1φ 1  y 
∂x ∂y   x  x    x 

= − x n −1 yφ 1  y  + nx nφ  y  + x n −1 yφ 1  y 
 x  x  x

= nx φ  x 
n y
 
Since u = x φ  x 
= nu . n y
 

Note: - In general, if u is a homogenous function of order x, y, z, t… then


∂u ∂u ∂u ∂u
x +y +z +t + ............ = nu
∂x ∂y ∂z ∂t

167
• Corollary to Euler’s theorem:- If u is homogenous function of x and y with
2 ∂ u ∂ 2u 2 ∂ u
2 2

degree n, then x + 2 xy +y = n( n − 1) u (VTU Feb-2005)


∂x 2 ∂x∂y ∂y 2
Proof: - As u is a homogenous function of degree, we have, by Euler’s theorem
∂u ∂u
x +y = nu -----------(*)
∂x ∂y
Differentiating (*) partially w.r.t x,
∂ 2 u ∂u ∂ 2u ∂u
x 2 + +y =n
∂x ∂x ∂x∂y ∂x
∂ 2u ∂ 2u ∂u
x + y = ( n − 1) ………….(1)
∂x 2
∂x∂y ∂x
Differentiating (*) Partially w.r.t y
∂ 2u ∂ 2 u ∂u ∂u
x +y 2 + =n
∂y∂x ∂y ∂y ∂y
∂ 2u ∂ 2u ∂u
or y + x = ( n − 1) ………….. (2)
∂y 2
∂y∂x ∂y
Multiplying esq. (1) by x and esq. (2) by y we get
 2 ∂ 2u ∂ 2u   2 ∂ 2u ∂ 2u   ∂u ∂u 
x + xy  +  y + xy  = ( n − 1)  x +y 
 ∂x
2
∂x∂y   ∂y 2
∂y∂x   ∂x ∂y 
∂ 2u ∂ 2u ∂ 2u
x 2 2 + 2 xy + y 2 2 = ( n − 1)( nu )
∂x ∂x∂y ∂y
 ∂ 2u ∂ 2u 
or 2 ∂ 2 u ∂ 2u ∂ 2
u  = 
x + 2 xy + y 2 2 = n( n − 1) u  ∂x∂y ∂x∂y 
∂x 2
∂x∂y ∂y

• Session - 4
• 3.1.7: Worked Examples
1. Verify Euler’s theorem for
 xy 
i). u =   ii) u = ax 2 + 2hxy + by 2
 x + y 
xy x 2 ( y x)  y x 
Solution:- (i) Consider u = = = x  = xφ  y  = x 1φ  y 
x + y x (1 + y x )  1 + y x   x  x
xy
This shows that u = is a homogenous function of degree1 (1=n).
x+ y
Hence, Euler’s theorem when applied to u becomes
∂u ∂u
x +y = (1)u ………….(*) (n=1)
∂x ∂y
we verify the equation (*) by showing actually LHS=RHS

168
 xy  ∂u y2 ∂u x2
now as u =   , = & =
 x + y  ∂x ( x + y ) ∂y ( x + y ) 2
2

∂u ∂u
∴ LHS of (*) = x +y
∂x ∂y
 y2   x2  xy ( x + y ) xy
= x 
2 
+ y 
 ( x + y) 2
=
 ( x + y ) 2 = ( x + y ) =RHS
 ( x + y )   
This verifies the result(*)
2 y2 
(ii) Consider u = ax + 2hxy + by = x  a 2 + 2h x  + b
2 2 y

 x   x2 

= x 2φ  y  ,which means u is a
 x
homogenous function of degree 2. Hence, on applying Euler’s theorem
to u, we get
∂u ∂u
x +y = 2u ………….(*)
∂x ∂y
∂u
Now, as u = ax 2 + 2hxy + by 2 , we see that = 2( ax + hy ) &
∂x
∂u
= 2( ay + hx )
∂y
∂u ∂u
Consider, LHS of (*) x +y
∂x ∂y
= x( 2( ax + hy ) ) + y ( 2( ay + hx ) )
(
= 2ax 2 + 2hxy + 2hxy + 2ay 2 = 2 ax 2 + 2hxy + by 2 )
= 2u = RHS
This verifies the result (*)
x y 
2 2
∂u ∂u
(2) If sin u =   , show that x +y = 3 tan u (VTU July-2005)
x+ y ∂x ∂y
 x2 y2 
Solution:- Consider sin u =   = f ,say We note that
 x + y 
2
x  y 
4
 x2 y2  x
f =   =  = x 3φ  y  .Thus, `f` is a homogenous function of degree 3.
 x + y  x1 + y   x
 x
Applying Euler’s theorem to f, we get
∂f ∂f
x +y = 3 f As f = sin u ,we see that
∂x ∂y
∂ ∂
x ( sin u ) + y ( sin u ) = 3( sin u )
∂x ∂y

169
 ∂u   ∂u 
i.e. x ( cos u )  + y ( cos u )  = 3 sin u
 ∂x   ∂y 
∂u ∂u
or we get x +y = 3 tan u as desired
∂x ∂y
 x3 + y3  ∂u ∂u
2. If  
 3 x + 4 y  , show that x +y = 2u log u (VTU july-2004)
u=e  
∂x ∂y
 x 3 +y 3


  x3 + y3 
Solution: - Consider  3 x +4 y  ∴ log u =   = f ,say
u =e  
 3x + 4 y 
3 y3 
x  1 + 
x3 + y3  x3 
∴f = = = x 2φ  y  .Thus, f is a homogenous function of degree 2.
3x + 4 y  x
x 3 + 4 y  
  x 
Applying Euler's theorem to f, we get
∂f ∂f
x +y =2f
∂x ∂y
∂ ∂
i.e x ( log u ) + y ( log u ) = 2( log u )
∂x ∂y
 1 ∂u   1 ∂u 
or x  + y   = 2 log u
 u ∂x   u ∂y 

∂u ∂u
or x +y = 2u log u
∂x ∂y
x+ y ∂u ∂u 1
4. If cos u = Prove that x +y = − cot u (VTU- Jan-2004)
x+ y ∂x ∂y 2

x1 + y  1
x+ y  x
Solution:- Let cos u = = = x φ  y  = f ,say Hence, f is a
2

x+ y    x
x 1 + y 
 x
homogenous function of degree ½
On applying Euler's theorem to f, we get
∂f ∂f 1
x +y = f Since f = cos u
∂x ∂y 2
∂ ∂ 1
x ( cos u ) + y ( cos u ) = cos u
∂x ∂y 2
 ∂u   ∂u  1 ∂u ∂u 1 cos u
or x sin u  + y − sin u  = cos u or x +y =−
 ∂x   ∂y  2 ∂x ∂y 2 sin u
∂u ∂u 1
or x +y = − cot u
∂x ∂y 2

170
−1  x + y 
3 3

5. If u = tan   , show that xu x + yu y = sin 2u .Hence to evaluate


 x+ y 
x 2 u xx + 2 xyu xy + y 2 u yy (VTU July-2005)

 x3 + y3 
Solution: - Consider u = tan  
−1
 x+y 
 
x3 + y3
∴ tan u = = f , say
x+ y
3 y3 
x 
 1 + 
x3 + y 3  x 3 
Now, ∴ f = = = x 2φ  y  .So, f is a homogenous function of
x+ y  y  x
x1 + 
 x
degree2. Applying Euler’s theorem to f we get
∂f ∂f
x +y = 2 f As f = tan u , we see that
∂x ∂y
∂ ∂
x ( tan u ) + y ( tan u ) = 2 tan u
∂x ∂y
 ∂u   ∂u 
x sec 2 u  + y sec 2 u  = 2 tan u
 ∂x   ∂y 

∂u ∂u 2 tan u
x +y = = 2 sin u cos u
∂x ∂y sec 2 u
∂u ∂u
x +y = sin 2u …………….(*),as required.
∂x ∂y
To get the value of x u xx + 2 xyu xy + y u yy we proceed as follows Differentiating (*)
2 2

partially w.r.t.x & y we get, respectively


∂ 2 u ∂u ∂ 2u ∂u
x 2 + +y = ( 2 cos 2u ) …………..(1)
∂x ∂x ∂x∂y ∂x
∂ 2u ∂ 2 u ∂u ∂u
x +y 2 + = ( 2 cos 2u ) …………….(2)
∂y∂x ∂y ∂y ∂y
Multiplying eq (1) by x, and eq(2) by y and adding thereafter
we get
∂ 2u ∂ 2u ∂ 2u  ∂u ∂u 
x2 + 2 xy + y 2 2 = ( 2 cos 2u − 1)  x + y 
∂x ∂x∂y ∂y  ∂x ∂y 
= ( 2 cos 2u − 1)( sin 2u )
x 2 u xx + 2 xyu xy + y 2 u yy = sin 4u − sin 2u

171
−1  y  −1  
6. If u = x tan  x  − y tan  x y  , show that
2 2

   
∂ u
2
∂ u2
∂ u
2
x2 + 2 xy + y 2 2 = 2u
∂x ∂x∂y ∂y
−1  y  −1  
Solution:- We note that u = v − w where v = x tan  x  & w = y tan  x y 
2 2

   
So that v and w are homogenous functions of degree 2. Applying the corollary to the
Euler’s theorem to v and w, we obtain
2 ∂ v ∂ 2v 2 ∂ v
2 2
x + 2 xy +y = 2( 2 − 1) v = 2v
∂x ∂x∂y ∂y 2
∂2w ∂2w 2 ∂ w
2
y 2
+ 2 xy +y = 2( 2 − 1) w = 2 w
∂x ∂x∂y ∂y 2
Taking the difference of these two expressions, we get
∂ 2u ∂ 2u ∂ 2u
x2 + 2 xy + y 2 2 = 2u
∂x ∂x∂y ∂y

Problem set 3.1.2 for practice

−1  x y  ∂u ∂u
2 2

1. If u = sin 
 x + y  ,show that x ∂x + y ∂y = tan u
 
 x2 y2  ∂z ∂z
2. If z = log  ,show that x + y =1
x+ y ∂x ∂y
−1 x+ y ∂z ∂z 1
3. If u = cot ,prove that x + y = − sin 2u
x+ y ∂x ∂y 4
−1  x y  ∂u ∂u 3
3 3

4. If z = tan 
 x 3 + y 3  ,prove that x ∂x + y ∂y = 2 sin 2u
 

172
Chapter - 3 : PARIAL DIFFERENTIATION

LESSON-2: Total derivatives, Differentiation of Composite


and Implicit functions

• In this lesson we learn the concept of total derivatives of functions of two


or more
Variables and, also rules for differentiation of composite and implicit
functions.

Session-1

3.2.0: a) Total differential and Total derivative:-


For a function z = f ( x, y ) of two variables, x and y the total differential
(or
exact differential ) dz is defined by:
∂f ∂f
dz = dx + dy --------------------------(1)
∂x ∂x
Further, if z = f ( x, y ) where x = x(t ) , y = y (t ) i.e. x and y are themselves
functions of an independent variable t, then total derivative of z is given by

dz ∂f dx ∂f dy
= + -------------------------(2)
dt ∂x dt ∂y dt

Similarly, the total differential of a function u = f ( x, y , z ) is defined by

∂f ∂f ∂f
du = dx + dy + dz -----------------(3)
∂x ∂y ∂z

Further, if u = f ( x, y , z ) and if x = x(t ) , y = y (t ) , z = z (t ) , then the total


derivative of u is given by

du ∂f dx ∂f dy ∂f dz
= + + ---------------(4)
dt ∂x dt ∂y dt ∂z dt

(b) Differentiation of implicit functions:-

An implicit function with x as an independent variable and y as the dependent


 dz   df 
variable is generally of the form z = f ( x, y ) = 0 . This gives   =   = 0 .
 dx   dx 
Then, by virtue of expression (2) above, we get

173
dz ∂f ∂f dy df ∂f ∂f dy
= + or = + , and hence
dx ∂x ∂y dx dx ∂x ∂y dx
 ∂f 
∂f ∂f dy dy ∂x 
0= + , so that we get = − ----------- (5)
∂x ∂y dx dx  ∂f 
 ∂y 
(c) Differentiation of composite functions:-

Let z be an function of x and y and that x = φ (u, v ) and y = ϕ (u, v ) are functions
of u and
v then,

∂z ∂f ∂x ∂f ∂y
= +
∂u ∂x ∂u ∂y ∂u
∂z ∂f ∂x ∂f ∂y
& = + -------------------(6)
∂v ∂x ∂v ∂y ∂v

Similarly, if z = f (u, v ) are functions of u and v and if u = φ ( x, y ) and v = ϕ ( x, y )


are functions of x and y then,

∂z ∂f ∂u ∂f ∂v
= +
∂x ∂u ∂x ∂v ∂x
∂z ∂f ∂u ∂f ∂v
& = + --------------------------(7)
∂y ∂u ∂y ∂v ∂y

Note:-1) The above formulae can be extended to functions of three are more
variables and formulas (6) and(7) are called Chain rule for partial differentiation.

2) The second and higher order partial derivatives of z = f ( x, y ) can be obtained


by repeated applications of the above formulas

3.2.1: Worked examples:-


1. Find the total differential of
(i) e x [ x sin y + y cos y ] (ii) e xyz
Ans:- (i) Let z = f ( x, y ) = e x [ x sin y + y cos y ] Then
∂z
= e x [ (1 + x ) sin y + y cos y ]
∂x
∂z
and = e x [ (1 + x ) cos y − y sin y ] Hence, using formula (1), we get
∂y
∂z ∂z
dz = dx + dy
∂x ∂y
i.e dz = e [ (1 + x ) sin y + y cos y ] dx + e x [ (1 + x ) cos y − y sin y ] dy
x

174
(ii) Let z = f ( x, y, z ) = e xyz Then
∂u ∂u ∂u
= ( yz ) e xyz ; = ( xz ) e xyz ; = ( xy ) e xyz
∂x ∂y ∂z
∴ Total differential of z = f ( x, y, z ) is (see formula (3) above)
∂u ∂u ∂u
du = dx + dy + dz
∂x ∂y ∂z
= e xyz ( yzdx + zxdy + xydz )
dz
2. Find if
dt
(i) z = xy 2 + x 2 y ,where x = at 2 , y = 2at
−1  y 
(ii) u = tan  x  ,where x = e t − e −t , y = e t + e −t (VTU-Jan 2003)
 
Ans:- (i) Consider z = xy 2 + x 2 y
∂z ∂z
= y 2 + 2 xy & = 2 xy + x 2
∂x ∂y
dx dy
Since x = at 2 & y = 2at , We have = 2at , = 2a
dt dt
Hence, using formula (2), we get
dz ∂z dx ∂z dy
= +
dt ∂x dt ∂y dt
( ) ( )
= y 2 + 2 xy ( 2at ) + 2 xy + x 2 ( 2a )
( ) ( )
= y 2 + 2 xy y + 2a 2 xy + x 2 , Using y = 2at
dz
= y 3 + 2 xy 2 + 4axy + 2ax 2
dt
 dz 
To get   explicitly in terms of t, we substitute
 dt 
x = at 2 & y = 2at , to get
 dz 
(
  = 2a 8t + 5t
3 3
)
4

 
dt

(ii) Consider
u = tan −1  y 
 x
∂u −y ∂u x
= 2 2 ,
= 2
∂x x + y ∂y x + y 2
Since x = e t − e −t & y = e t + e −t ,we have

175
dx dy
= e t + e −t = y = e t − e −t = x
dt dt
du ∂u dx ∂u dy
Hence = + ( see Eqn (2))
dt ∂x dt ∂y dt
 −y   x   x2 − y2 
=  2 
2 
y + 
 x2 + y2  x =  2 
2 
x +y    x +y 
Substituting x = e t − e −t & y = e t + e −t , we get
du −2
= 2t
dt e + e − 2t
Session-2
 dy 
3. Find   if (i) x y + y x =Constant
 dx 
(ii) x + e y = 2 xy
Ans: - (i) Let z = f ( x, y ) = x y + y x =Constant. Using formula (5)
∂f
dy
= − ∂x -----------------(*)
dx ∂f
∂y
∂f ∂f
But = yx y −1 + y x log y and = x y log x + xy x−1 Putting those in(*), we get
∂x ∂y
dy  yx y −1 + y x log y 
= − y x −1 
dx  x log x + xy 
(ii) Let z = f ( x, y ) = e x + e y − 2 xy =Constant
∂f ∂f
Now, = ex − 2y ; = e y − 2 x Using this in (8),
∂x ∂y
 ∂f 
dy   e x − 2 y 
= − ∂x  = − y 
dx  ∂f   e − 2x 
 ∂y 
4. (i) If z = f ( x, y ) ,where x = r cos θ , y = r sin θ show that
2 2 2 2
 ∂z   ∂z   ∂z  1  ∂z 
  +   =   + 2   (VTU July-2005)
 ∂x   ∂y   ∂r  r  ∂θ 

(ii) If z = f ( x, y ) ,where x = e u + e − v & y = e −u − e v ,Show that


∂z ∂z ∂z ∂z
− =x −y
∂u ∂v ∂x ∂y
Ans: - As x = r cos θ and y = r sin θ , we have

176
∂x ∂x ∂y ∂y
= cos θ , = −r sin θ ; = sin θ & = r cos θ . Using Chain rule (6) & (7) we
∂r ∂θ ∂r ∂θ
have
 ∂z  ∂z ∂x ∂z ∂y ∂z ∂z
 = + = ( cos θ ) + ( sin θ )
 ∂r  ∂x ∂r ∂y ∂r ∂x ∂y

 ∂z  ∂z ∂x ∂z ∂y ∂z ∂z
 = + = ( − r sin θ ) + ( r cos θ )
 ∂θ  ∂x ∂θ ∂y ∂θ ∂x ∂y
Squaring on both sides, the
above equations, we get
2 2 2
 ∂z   ∂z   ∂z   ∂z  ∂z 
  =   cos θ +   sin θ + 2   sin θ cos θ
2 2

 ∂r   ∂x   ∂y   ∂x  ∂y 

2 2 2
1  ∂z   ∂z   ∂z   ∂z  ∂z 
 =   cos θ +   sin θ − 2   sin θ cos θ
2 2
2 
r  ∂θ   ∂x   ∂y   ∂x  ∂y 

Adding the above equations , we get


 ∂z 
2
1  ∂z 
2  ∂z  2  ∂z  2 
  + 2   =   +    cos θ + sin θ
2 2
( )
 ∂r  r  ∂θ   ∂x   ∂y  
2 2
 ∂z   ∂z 
=   +   as desired.
 ∂x   ∂y 
(ii) As x = e + e & y = e −u − e v , We have
u −v

∂x ∂x ∂y ∂y
= eu , = −e − v , = −e − u & = −e v
∂u ∂v ∂u ∂v
Using Chain rule (6) we get
 ∂z  ∂z ∂x ∂z ∂y ∂z u ∂z −v
 = + = ( )
e − e ( )
 ∂u  ∂x ∂u ∂y ∂u ∂x ∂y
 ∂z  ∂z ∂x ∂z ∂y ∂z ∂z
 = + =( )
− e −v − − ev( )
 ∂v  ∂x ∂v ∂y ∂v ∂x ∂y
 ∂z   ∂z  ∂z u ∂z −u
∴  −  = ( e + e −v −) e − ev ( )
 ∂u   ∂v  ∂x ∂y
∂z ∂z
= x− y
∂x ∂y
∂u ∂u ∂u
5. (i) If u = f ( x, z, y / z ) Then show that x −y −z = 0 (VTU-July-2004)
∂x ∂y ∂z
(ii) If H = f ( x − y, y − z , z − x ) , show that
∂H ∂H ∂H
+ + =0 (VTU-July-2003)
∂x ∂y ∂z

177
Ans: - (i) Let u = f (v, w) , where v = xz and w = y z
∂v ∂v ∂v ∂w ∂w 1 ∂w − y
=z, =0, =x & = 0, =
∂x ∂y ∂z ∂x ∂y z , ∂z = z2
Using Chain rule,
∂u ∂u ∂v ∂u ∂w ∂u
= + = ( z ) + ∂u ( 0) = z ∂u
∂x ∂v ∂x ∂w ∂x ∂v ∂w ∂v

∂u ∂u ∂v ∂u ∂w ∂u
= +
∂y ∂v ∂y ∂w ∂y ∂v
=
∂w
( )
( 0) + ∂u 1 z = 1 ∂u
z ∂w
∂u ∂u ∂v ∂u ∂w ∂u
= + = ( x ) + ∂u  − y z 2  = x ∂u − y2 ∂u
∂z ∂v ∂z ∂w ∂z ∂v ∂w   ∂v z ∂w
From these, we get
∂u ∂u ∂u ∂u y ∂u  ∂u y ∂u 
x −y −z = xz − − z x − 
∂x ∂y ∂z ∂v z ∂w  ∂v z 2 ∂w 
=0
(ii) Let H = f (u , v, w) Where u = x − y, v = y − z, w = z − x
∂u ∂u ∂u
Now, = 1, = −1, =0
∂x ∂y ∂z
∂v ∂v ∂v
= 0, = 1, = −1
∂x ∂y ∂z
∂w ∂w ∂u
= −1, = 0, = 1 Using Chain rule,
∂x ∂y ∂z
∂H ∂H ∂u ∂H ∂v ∂H ∂w ∂H
= + + = (1) + ∂H ( 0) + ∂H ( − 1)
∂x ∂u ∂x ∂v ∂x ∂w ∂x ∂u ∂v ∂w
∂H ∂H ∂u ∂H ∂v ∂H ∂w ∂H
= + + = ( − 1) + ∂H (1) + ∂v ( 0)
∂y ∂u ∂y ∂v ∂y ∂w ∂y ∂u ∂v ∂w
∂H ∂H ∂u ∂H ∂v ∂H ∂w ∂H
= + + = ( 0) + ∂H ( − 1) + ∂H (1)
∂z ∂u ∂z ∂v ∂z ∂w ∂z ∂u ∂v ∂w
Adding the above equations, we get
∂H ∂H ∂H
+ + = 0 , as desired.
∂x ∂y ∂z
3.2.2: Problem Set No: 3.2.1

1. Find the total differentials of


(i) xyz + ( xyz ) −1 (ii) x 2 y + y 2 z + z 2 x
 du 
2. Find   If
 dt 
(i) u = x 2 − y 2 , x = e t cos t , y = e t sin t
(ii) u = sin xy 2 , x = log t , y = e t

178
(iii) u = xy + yz + zx, x = 1 t , y = e , z = e
t −t

(iv) u = log( x + y + z ) , x = e − t , y = sin t , z = cos t


 dy 
3. Find   is each of the following cases:-
 dx 
(i) x sin ( x − y ) = ( x + y ) (ii) ( cos x ) y = ( sin y ) x
4. If z = f ( x, y ) and x = u − v, y = uv Prove that
∂z ∂z ∂z ∂z ∂z ∂z
(i) ( u + v ) =u −v (ii) ( u + v ) = +
∂x ∂x ∂v ∂y ∂u ∂v
5. If u = x 2 − y 2 , z = 2r − 3s + 4, y = − r + 8s − 5, Prove that
∂u
= 4x + 2 y
∂r
6. If z = f (r , s) , where r = x + at , y = y + bt Prove that
∂u ∂u ∂u
=a +b
∂t ∂x ∂y

Chapter - 3 : PARIAL DIFFERENTIATION

LESSON-3: Applications to Jacobians, Errors and


Approximations
In this lesson, we study Jacobians, errors and approximations using the
concept of
partial differentiation.

Session - 1

3.3.0 Jacobians:-
Jacobians were invented by German mathematician C.G. Jacob Jacobi (1804-
1851),
who made significant contributions to mechanics, Partial differential equations and
calculus
of variations.

Definition:- Let u and v are functions of x and y, then Jacobian of u and v w.r.t x and
y,
denoted by

179
 u, v  ∂( u, v )
J or J   or
 x, y  ∂ ( x, y )

is defined by
∂u ∂u
 u, v  ∂x ∂y
J
 x, y 
 = ∂v
  ∂v
∂x ∂y
Similarly, if u, v, w are functions of three independent variables of x, y, z, then

∂u ∂u ∂u
∂x ∂y ∂z
 u , v, w  ∂v ∂v ∂u
J = J   =
 x, y, z  ∂x ∂y ∂z
∂w ∂w ∂w
∂x ∂y ∂z
Remark:- In a similar way, Jacobian of n functions in n-variables can be defined

∂( u, v )
Note:- (i) If J = , then the "inverse Jacobian" of the Jacobian J,
∂ ( x, y )
denoted by J ′ ,is defined as

∂( x, y )
J =
∂( u , v )
∂ ( u , v, w ) ∂ ( x, y , z )
(ii) Similarly, "inverse Jacobian" of J = is defined as J ′ =
∂ ( x, y , z ) ∂ ( u , v, w )

3.3.1: Properties of Jacobians :-

∂( u, v ) ∂ ( x, y )
Property 1:- If J = and J ′ = then JJ ′ = 1
∂ ( x, y ) ∂( u, v )

Proof:- Consider
∂u ∂u ∂x ∂x
∂ ( u , v ) ∂ ( x, y ) ∂x ∂y ∂u ∂v
JJ 1 = × = ×
∂ ( x, y ) ∂ ( u , v ) ∂v ∂v ∂y ∂y
∂x ∂y ∂u ∂v

180
∂u ∂x ∂u ∂y ∂u ∂x ∂u ∂y
+ +
∂x ∂u ∂y ∂u ∂x ∂v ∂y ∂v 1 0
= = =1
∂v ∂x ∂v ∂y ∂v ∂x ∂v ∂y 1 0
+ +
∂x ∂u ∂y ∂u ∂x ∂v ∂y ∂v

Property 2:- (Chain rule for Jacobians):- If u and v are functions of r&s and r,s
are functions x&y,then
 u, v   u, v   r , s 
J =   = J   × J  
 x, y   r , s   x, y 
Proof:- Consider
∂u ∂u ∂r ∂r
 u , v   r , s  ∂r ∂s ∂x ∂y
J  × J   = ×
 r , s   x, y  ∂v ∂v ∂s ∂s
∂r ∂s ∂x ∂y
∂u ∂r ∂u ∂s ∂u ∂r ∂u ∂s
+ +
∂r ∂x ∂s ∂x ∂r ∂y ∂s ∂y
=
∂v ∂r ∂v ∂s ∂v ∂r ∂v ∂s
+ +
∂r ∂x ∂s ∂x ∂r ∂y ∂s ∂y
∂u ∂u
∂x ∂y  u, v 
= = J  
∂v ∂v  x, y 
∂x ∂y

3.3.2:-Jacobians in various co-ordinate systems:-

1. In Polar co-ordinates, x = r cos θ , y = r sin θ


∂( u, v )
we have =r
∂ ( r ,θ )
2. In spherical coordinates, x = ρ cos φ , y = ρ sin φ , z = z , we have
∂ ( x, y , z )

∂( ρ ,φ , z )

3. In spherical polar co-ordinates, x = r sin θ cos φ , y = r sin θ sin φ , z = r cos θ


∂x ∂y
Proof of 1:- we have, = cos θ and = sin θ
∂r ∂r
∂x ∂y
= −r sin θ and = r cos θ
∂θ ∂θ

181
∂x ∂x
∂ ( x, y ) ∂r ∂θ cos θ − r sin θ
∴ = =
∂ ( r , θ ) ∂y ∂y sin θ r cos θ
∂r ∂θ
( )
= r cos 2 θ + r sin 2 θ = r cos 2 θ + sin 2 θ = r

∂x ∂y ∂z
Proof of 2 :-we have = cos φ , = sin φ , =0
∂ρ ∂ρ ∂ρ
∂x ∂y ∂z
= − ρ sin φ , = ρ cos φ , =0
∂φ ∂φ ∂φ
∂z ∂z ∂z
=0, = 0, = 0
∂ρ ∂φ ∂z
∂x ∂x ∂x
∂ρ ∂φ ∂z
cos φ − ρ sin φ 0
∂ ( x, y , z ) ∂y ∂y ∂y
∴ = = sin φ ρ cos φ 0 =ρ
∂ ( ρ , φ , z ) ∂ρ ∂φ ∂z
0 0 1
∂z ∂z ∂z
∂ρ ∂φ ∂z

Proof of 3:- We have


∂x ∂x ∂x
= sin θ cos θ , = r cos θ cos φ , = −r sin θ sin φ
∂r ∂θ ∂φ
∂y ∂y ∂y
= sin θ sin φ , = r cos θ sin φ , = r sin θ cos φ
∂r ∂θ ∂φ
∂z ∂z ∂z
= cos θ , = −r sin θ , =0
∂r ∂θ ∂φ
sin θ cos θ r cos θ cos φ − r sin θ sin φ
∂ ( x, y , z )
∴ = sin θ sin φ r cos θ sin φ r sin θ cos φ
∂( r ,θ ,φ )
cos θ − r sin θ 0
= r 2 sin θ

Session – 2

3.3.3:-Worked Examples:-

1. If u = x 2 − 2 y 2 , v = 2 x 2 − y 2 , where
x = r cos θ , y = r sin θ show that

182
∂( u, v )
= 6r 3 sin 2θ (VTU-Jan-2006)
∂ ( r ,θ )
Consider u = x 2 − 2 y 2 = r 2 cos 2 θ − 2r 2 sin 2 θ
v = 2 x 2 − y 2 = 2r 2 cos 2 θ − r 2 sin 2 θ
∂u ∂v
∴ = 2r cos 2 θ − 4r sin 2 θ , = 4r cos 2 θ − 2r sin 2 θ
∂r ∂r
∂u
= −2r 2 cos θ sin θ − 4r 2 sin θ cos θ
∂θ
∂v
= −4r 2 cos θ sin θ − 2r 2 sin θ cos θ
∂θ
∂u ∂u
∂ ( u , v ) ∂r ∂θ 2r cos 2 θ − 4r sin 2 θ − 2r 2 cos θ sin θ − 4r 2 sin θ cos θ
= =
∂ ( r ,θ ) ∂v ∂v 4r cos 2 θ − 2r sin 2 θ − 4r 2 cos θ sin θ − 2r 2 sin θ cos θ
∂r ∂θ
( )(
= 2r cos 2 θ − 4r sin 2 θ − 4r 2 cos θ sin θ − 2r 2 sin θ cos θ )
( )(
= − 2r cos θ sin θ − 4r 2 sin θ cos θ 4r cos 2 θ − 2r sin 2 θ)
= 6r 3 sin 2θ

 x, y  1  x , y 
2. If x = u (1 − v ) , y = uv, Prove that J  × J   = 1 (VTU-2001)
 u, v   u, v 
∂x ∂x
Consider = 1 − v , = −u
∂u ∂v
∂y ∂y
= v, =u
∂u ∂v
∂x ∂x
 x, y  ∂u ∂v 1 − v − u
∴J  = =
 u , v  ∂y ∂y v u
∂u ∂v
= (1 − v ) u − ( − uv ) = u − uv + uv = u
 x, y 
∴J   = u − − − (1)
 u, v 
Further, as x = u (1 − v ) , y = uv,
= u − uv
We write, x = u − y ∴u = x + y and
y  y   y 
v = =   ∴ v =  
u x+ y x+ y

183
∂u ∂u
∴ = 1, = 1 and
∂x ∂y
∂v y ∂v x
=− , =
∂x ( x + y ) ∂y ( x + y ) 2
2

∂u ∂u
1 1
1 u, v  ∂ x ∂y
∴ J   = = y x
 x, y  ∂ v ∂ v −
( x + y) 2 ( x + y) 2
∂ x ∂y
x y  1  1
= + =   =
( x + y) 2
( x + y) 2
x+ y u
1
∴ JJ 1 = u =1
u

3. If x = e u cos v, y = e u sin v, Prove that


∂ ( x, y ) ∂ ( u , v )
× =1
∂ ( u , v ) ∂ ( x, y )
Consider x = e u cos v y = e u sin v
∂x ∂y
= e u cos v = e u sin v
∂u ∂u
∂x ∂y
= −e u sin v = e u cos v
∂v ∂v
∂x ∂x
∂ ( x, y ) ∂u ∂v e cos v − e sin v
u u

∴ = =
∂ ( u, v ) ∂y ∂y e u sin v e u cos v
∂u ∂v
∂ ( x, y )
i.e = e 2u − − − − − (1)
∂( u, v )
Again Consider x = e u cos v, y = e u sin v,
1
∴ x 2 + y 2 = e 2u or u = log x 2 + y 2
2
( )
y −1  y 
& = tan v or v = tan  x 
x  
∂u x ∂u y
Hence = 2 ,; = 2 ,;
∂x x + y 2
∂y x + y 2
∂v −y ∂v x
& = 2 ; = 2
∂x x + y ∂y x + y 2
2

184
∂u ∂u x y
∂( u, v ) ∂x ∂y x +y x + y2
2 2 2

∴ =
∂ ( x, y ) ∂v ∂v
=
−y x
= 2
1
x + y2
(
= x2 + y 2 ) −1

∂x ∂y x2 + y 2 x2 + y 2
∂( u, v )
i.e = e − 2u − − − − − − − (2)
∂ ( x, y )
∂ ( x, y ) ∂ ( u , v )
∴ × = e2u × e− 2u = 1
∂ ( u , v ) ∂ ( x, y )

yz zx xy ∂ ( u , v, w )
4. If u = , v = , w = , Show that =4
x y z ∂ ( x, y , z )
∂u ∂u ∂u yz z y
− 2
∂x ∂y ∂z x x x
∂ ( u , v, w) ∂v ∂v ∂v z − zx x
Now, = =
∂ ( x, y, z ) ∂x ∂y ∂z y y2 y
∂w ∂w ∂w y x − xy
∂x ∂y ∂z z z z2
∂ ( u , v, w)  yz  − zx  − xy   z  z  − xy   y  x 
=  −    −     −   
∂ ( x, y, z )  x 2  y 2  z 2   x  y  z 2   z  y 
i.e
 y  z  x   y  − zx 
+     −   2 
 x  y  z   z  y 

= 4, as desired.

∂ ( x, y , z )
5. If x = r sin θ cos φ , y = r sin θ sin φ , z = r cosθ ,show that = r 2 sin θ
∂( r , θ , φ )
Now, by definition
∂x ∂x ∂x
∂r ∂θ ∂φ
∂ ( x, y, z ) ∂y ∂y ∂y
=
∂ ( r , θ , φ ) ∂r ∂θ ∂φ
∂z ∂z ∂z
∂r ∂θ ∂φ
sin θ cos φ r cos θ cos φ − r sin θ sin φ
∂ ( x, y , z )
i.e = sin θ sin φ r cos θ sin φ r sin θ cos φ
∂( r , θ , φ )
cos θ − r sin θ 0

185
{ (
= sin θ cos φ 0 − r 2 sin 2 θ cos φ )}
− r cos θ cos φ { 0 − ( r sin θ cos θ cos φ )}
{
− r sin θ sin φ − r sin 2 θ sin φ − r cos 2 θ sin φ }
( )
= r 2 sin 2 θ sin θ cos 2 φ + r 2 sin θ cos 2 θ cos 2 φ
( )
= r 2 sin θ sin 2 θ + cos 2 θ cos 2 φ + r 2 sin θ sin 2 φ
(
= r 2 sin θ cos 2 φ + sin 2 φ )
= r 2 sin θ , as required

3.3.4:-Problem set No: 3.3.1

1. x = r cos θ , y = r sin θ show that


 x, y  1  r , θ 
J  × J   = 1
 r, θ   x, y 
2. If x = eu sec v, y = eu tan v, Show that
∂ ( x, y ) ∂ ( u , v )
× =1
∂ ( u , v ) ∂ ( x, y )
3.If u = xyz, v = xy + yz + zx, w = x + y + z show that
∂ ( u , v, w )
= ( x − y )( y − z )( z − x )
∂ ( x, y , z )
4.If x + y + z = u , y + z = v, z = uvw, find the value of
∂ ( x, y , z )
∂ ( u , v, w )
 u, v 
5.If u + v = e x cos y, u − v = e x sin y find J  
 x, y 
ex ex
[Hint: Start from u = ( cos y + sin y ) and v = ( cos y − sin y )
2 2

3.3.5:-Errors and Approximations


Session-3:-

While doing scientific or engineering computations there is always scope for


computation errors. Obviously, it is going to have an impact on the result which may
be negligible or significant too. In such situations, an account of various types of
errors is necessary to present approximate value of the function under consideration.

3.3.6:-Definition:-

If z = f ( x, y ) is a function of two variables x and y then the error or absolute error


∂z is given by

186
 ∂z   ∂z 
δz =  δx +  δy -------------(1)
 ∂x   ∂y 

Similarly, if u = f ( x, y, z ) is a function of three variables x,y,z then the error or


absolute error δu is given by
 ∂u   ∂u   ∂u 
δu =  δx +  δy +  δz ----------- (2)
 ∂x   ∂y   ∂z 
Remark:- The quantity o is called relative error in z or u respectively. Similarly, the
 δz   δu 
quantity  × 100  or  × 100  is called percentage error in z or u respectively
 z  u 

Note:- The following results similar to that of differentials will also be useful.

(i) δ ( cu ) = cδu (ii) δ ( u ± v ) = δu ± δv


 u  vδu − uδv
(iii) δ ( uv ) = uδv + vδu (iv) δ   =
v v2

3.3.7:-Worked Examples:-

1. Find the percentage error in measuring the area of a traingle if 1% error (each) is
made in measuring its base as well as height.
Area A of a traingle is given by

1
A= ah ---------- (1)
2
where `a`and `h` are base and height of the triangle respectively. Therefore, the
change ∂A due to a change δa in a, and a change δh in h, is given by

1 1
δA = δ ( ah ) = [ aδh + hδa ]
2 2
1 [ aδh + hδa ]
δA
∴ = 2
A 1
ah
2
δA δh δa
∴ = +
A h a
δA  δh   δa 
∴ × 100 =  × 100  +  × 100 
A  h   a 
=1 % +1% (by data)
=2%
The percentage error in A is 2%

187
1 1 1
2. The focal length f of a lens is given by = + where p and q are the
f p q
distances of the lens from the object and the image respectively. For a certain lens
p and q are each 25cms with a possible error of almost 0.5cm. Find the
approximate value of the maximum error in f.

1 1 1
Consider = + . By data p = q = 25 and δp = δq = 0.5
f p q
1  1 1
Now, δ   = δ  + 
f  p q
1 1 1
δ   = δ   + δ  
i.e  f   p q

1 1 1
− 2
δf = − 2 δp − 2 δq
f p q
2  δp δq 
i.e δf = f  2 + 2 
p q 
As p = q, and δp = δq, we have
 2δp  1 1 1
δf = f 2  2  Since = + = 0.08
p  f 25 25
we have f=12.5. Hence,
  0.5 
δf = (12.5) 2 2  = 0.25
2 
  ( 25 ) 
∴ The maximum error in f=0.25

3. The current measured by a tangent galvanometer is given by the relation


c = k tan θ Where θ is the angle of deflection .Show that the relative error in c
due to a given error in θ is maximum where θ = 450 .
Consider c = k tan θ (k=constant)
∴ δc = δ ( k tan θ )
( )
= k sec 2 θ δθ + 0


δc k sec θ 2
= δθ
c k tan θ
δc  2 
or = δθ -----------(*)
c  sin 2θ 
δc
The relation error in c being is maximum when the denominator of RHS of
c
(*) is maximum and the maximum of sine function is unity.
∴ sin 2θ = 1 ⇒ 2θ = 900 ⇒ θ = 450

188
Thus, the relative error in c is minimum when θ = 450 .

1 2
4. If T = mv is the kinetic energy, find approximately the change in T as m
2
changes From 49 to 49.5 and V changes from 1600 to 1590.

By data, m = 49, m + ∂m = 49.5 ⇒ ∂m = 0.5


v = 1600, v + ∂v = 1590 ⇒ ∂v = −10
1 2
As T = mv , δT = δ mv
2
1
2
( 2
)
1
[
= m( 2vδv ) + v 2 ( δm )
2
]
1
[ ]
i.e δT = ( 49 )( 2 )(1600 )( − 10 ) + ( 0.5)(1600 )
2
2

=-1,44,000

5. If the sides of a triangle ABC vary in such a way that its circum radius remains
constant, Prove that
δa δb δc
+ + =0
cos A cos B cos c
If the triangle ABC is inscribed in a circle of radius R and if a,b,c respectively denotes
the sides opposite to the angles A,B,C we have the sine rule given by
a b c
= + = 2R
sin A sin B sin c

⇒ a = 2 R sin A ⇒ δa = δ ( 2 R sin A) = 2 Rδ ( sin A)


∴ δa = 2 R cos AδA;
⇒ b = 2 R sin B ⇒ δb = δ ( 2 R sin B ) = 2 R δ ( sin B )
∴ δb = 2 R cos BδB;
⇒ c = 2 R sin C ⇒ δc = δ ( 2 R sin C ) = 2 R δ ( sin C )
∴ δC = 2 R cos CδC
From the above, we write
δa δb δc
= 2 RδA; = 2 RδB & = 2 RδC
cos A cos B cos C
Adding,
δa δb δc
+ + = 2 RδA + 2 RδB + 2 RδC
cos A cos B cos C
= 2 R ( δA + δB + δC )
= 2 Rδ ( Cons tan t )
A + B + C = π radians = Cons tan t
= 2 R δ ( 0)

189
=0

3.3.8:-Problem set No: 3.3.2

1. Find the percentage error in the evaluation of the area of an ellipse if 1% error is
made while measuring the major and minor axis
.
2. The time T of a Complete oscillation of a simple pendulum is given by the
formula T = ( 2π ) l g , where g is constant .If there is an error of 3% in the value
of l, Find the percentage value of T.

3. The radius of a Sphere is found to be 10cms with possible error of 0.02cm. What
is the relative error in computing the volume?
4
( Hint: Volume of the Sphere= π r )
3

3
4. The pressure p and the volume v of a gas are concentrated by the relation
pv1.4=constant. find the percentage increase in pressure corresponding to a
1
dimension of % in volume
2

190

Das könnte Ihnen auch gefallen